You are on page 1of 118

Arab Academy for Science, Technology & Maritime

Transport

Vector Calculus & Complex Analysis


Math 4 (BA224)

Dr. Mohamed Fathy

Spring 2021
Arab Academy for Science, Technology and Maritime Transport
Collage of Engineering and Technology– Heliopolis
Basic and Applied Sciences Department - Cairo

Mathematics 4 (BA224) – Syllabus


Calculus
Text Book Early Transcendental Functions (Fourth Edition)
R.T T. Smith, R. B. Minton.
Academic Year Spring 2019
Used Tools Time Weight
th
Quiz 5 week 10 %
th 30 %
Evaluation 7 Week examination 7th week 20 %
12th Week examination 12th week 20 % 20 %
Class activities, assignment and attendance weekly 10 %
Final – term Examination 16th week 40 %
Course Policy Make-up Exam: There are NO make-up exams.

Course Outlines and Schedule Plan


Week 1 Vector Algebra – Dot and cross product and its Applications.
Week 2 Lines and Curves in Plane and Space – Vector valued function – Plane.
Week 3 Gradient of scalar field – Divergence of vector field and its Applications
Curl of vector field and its Applications – Line Integrals of a scalar and a vector
Week 4
function.
Line Integrals: Conservative vector fields – Path Independent.
Week 5
Double Integrals in Cartesian.
Week 6 Double Integrals in polar coordinates – Green’s Theorem.
Week 7 7th week Exam
Week 8 Planes, Cylinders and Quadric Surfaces – Triple Integrals in Cartesian.
Week 9 Triple Integrals in Cylindrical and Spherical coordinates.
Surface Integrals – Surface area – Flux of a Vector Field – Divergence Theorem
Week 10
(Gauss’ Theorem).
Week 11 Divergence Theorem (Gauss’ Theorem) – Stokes’ Theorem.
Week 12 12th week Exam
Complex numbers and functions – forms of representation – Roots of complex
Week 13
number – Complex power – Elementary functions.
Week 14 Elementary functions (cont’d) – Analytic functions – Harmonic functions.
Week 15 Revision
Chapter 1 Vectors

Chapter 1

Vectors

Introduction

In science, mathematics, and engineering, we distinguish two important quantities: scalars and vectors.
A scalar is simply a real number or a quantity that has magnitude only. For example, length, temperature,
and blood pressure are represented by numbers such as 80 m, 20°𝐶, and the systolic/diastolic ratio 120/80.
A vector, on the other hand, is usually described as a quantity that has both magnitude and direction.
If we push against an object, the effect is determined not only by the strength of the push, but its
direction. Velocity and acceleration are vectors.
We can include both magnitude and direction in one package by representing a vector as an arrow
from the origin to end point. The choice of the point gives the direction of the vector (when viewed from
the origin), and the length is its magnitude.

y

Vectors in the Plane and 3-Space a

(a1 , a2 )
In general, a vector a in ℝ2 is any ordered pair of real numbers, 
a
𝑎⃑ = 𝑎1 𝐢 + 𝑎2 𝐣 a2
j
i x
𝑎⃑ = 〈𝑎1 , 𝑎2 〉, a1
where 𝐢 and 𝐣 are the standard unit vectors and the numbers 𝑎1 and 𝑎2 are said
to be the components of the vector 𝑎⃑ in the directions 𝑥 and 𝑦.
z 
A vector a in 3-space is any ordered triple of real numbers a
𝑎⃑ = 𝑎1 𝐢 + 𝑎2 𝐣 + 𝑎3 𝐤 (a1 , a2 , a3 )

a
𝑎⃑ = 〈𝑎1 , 𝑎2 , 𝑎3 〉,
k a3
i
where 𝐢, 𝐣 and 𝐤 are the standard unit vectors and the numbers 𝑎1 , 𝑎2 j y
a1
and 𝑎3 are said to be the components of the vector 𝑎⃑ in the directions a2
of 𝑥, 𝑦 and 𝑧 respectively. x
-1-
Chapter 1 Vectors

Vector between Two Points z B( x2 , y2 , z2 )


⃑⃑⃑⃑⃑⃑
𝐴𝐵
The vector that has start point 𝐴(𝑥1 , 𝑦1 , 𝑧1 ) and
end point 𝐵(𝑥2 , 𝑦2 , 𝑧2 ) can be written as ( x2  x1, y2  y1, z2  z1 )
⃑⃑⃑⃑⃑⃑ = (𝑥2 − 𝑥1 )𝐢 + (𝑦2 − 𝑦1 )𝐣 + (𝑧2 − 𝑧1 )𝐤.
𝐴𝐵 A( x1, y1, z1 ) ⃑⃑⃑⃑⃑⃑
𝐴𝐵
⃑⃑⃑⃑⃑⃑ write in another following form
The vector 𝐴𝐵 k y
i
⃑⃑⃑⃑⃑⃑

z2  z1
𝐴𝐵 = 〈𝑥2 − 𝑥1 , 𝑦2 − 𝑦1 , 𝑧2 − 𝑧1 〉. j
x1
The length (also called the magnitude or norm) of x x2
⃑⃑⃑⃑⃑⃑ is the scalar in the form
a vector AB y2  y1

⃑⃑⃑⃑⃑⃑ ‖ = √(𝑥2 − 𝑥1 )2 + (𝑦2 − 𝑦1 )2 + (𝑧2 − 𝑧1 )2


‖𝐴𝐵

𝐀𝐁) in the direction of a vector ⃑⃑⃑⃑⃑⃑


The unit vector (also called the direction of a vector ⃑⃑⃑⃑⃑⃑ AB denoted by 𝑎̂
where

⃑⃑⃑⃑⃑⃑
𝐴𝐵 (𝑥2 − 𝑥1 )𝐢 + (𝑦2 − 𝑦1 )𝐣 + (𝑧2 − 𝑧1 )𝐤
𝑎̂ = =
⃑⃑⃑⃑⃑⃑ ‖ √(𝑥2 − 𝑥1 )2 + (𝑦2 − 𝑦1 )2 + (𝑧2 − 𝑧1 )2
‖𝐴𝐵

Equality of two vectors

If 𝑢
⃑⃑ = 𝑢1 𝐢 + 𝑢2 𝐣 + 𝑢3 𝐤 and 𝑤
⃑⃑⃑ = 𝑤1 𝐢 + 𝑤2 𝐣 + 𝑤3 𝐤 then,
𝑢 ⃑⃑⃑ if and only if 𝑢1 = 𝑤1 , 𝑢2 = 𝑤2 and 𝑢3 = 𝑤3
⃑⃑ = 𝑤

⃑⃑⃑⃑⃑⃑⃑, ‖𝑨𝑩
Example 1 For the given points 𝑨(𝟐, 𝟏, 𝟑) and 𝑩(𝟑, 𝟓, −𝟐), Find 𝑨𝑩 ⃑⃑⃑⃑⃑⃑⃑ ‖ and find a unit vector
⃑⃑⃑⃑⃑⃑⃑.
in direction of 𝑨𝑩
Solution:
⃑⃑⃑⃑⃑⃑
𝐴𝐵 = ⃑⃑⃑⃑⃑⃑
𝑂𝐵 − ⃑⃑⃑⃑⃑⃑ ⃑⃑⃑⃑⃑⃑ ‖ = √12 + 42 + (−5)2 = √42,
𝑂𝐴 = < 1,4, −5 > = 𝐢 + 4 𝐣 − 5 𝐤, ‖𝐴𝐵
⃑⃑⃑⃑⃑⃑
𝐴𝐵 𝐢 + 4𝐣 − 5𝐤
𝑎̂ = = .
⃑⃑⃑⃑⃑⃑ ‖
‖𝐴𝐵 √42

Vector Addition

If you think of the vector ⃑⃑⃑⃑⃑⃑


𝐴𝐵 as representing the displacement of a
particle from the point 𝐴 to the point 𝐵, notice that the end result of
displacing the particle from A to B (corresponding to the vector ⃑⃑⃑⃑⃑⃑
𝐴𝐵),
followed by displacing the particle from 𝐵 to 𝐶 (corresponding to the
⃑⃑⃑⃑⃑⃑ ) is the same as displacing the particle directly from 𝐴 to 𝐶,
vector 𝐵𝐶
which corresponds to the vector ⃑⃑⃑⃑⃑⃑
𝐴𝐶 (called the resultant vector). We call
⃑⃑⃑⃑⃑⃑
𝐴𝐶 the sum of ⃑⃑⃑⃑⃑⃑
𝐴𝐵 and ⃑⃑⃑⃑⃑⃑
𝐵𝐶 and write

⃑⃑⃑⃑⃑⃑
𝐴𝐶 = ⃑⃑⃑⃑⃑⃑
𝐴𝐵 + ⃑⃑⃑⃑⃑⃑
𝐵𝐶 .

-2-
Chapter 1 Vectors

To add two vectors, we locate the initial point of one at the terminal point of
the other and complete the parallelogram, as indicated in the opposite figure.
The vector lying along the diagonal, with initial point at 𝐴 and terminal point
at 𝐶 is the sum

⃑⃑⃑⃑⃑⃑
𝐴𝐶 = ⃑⃑⃑⃑⃑⃑
𝐴𝐵 + ⃑⃑⃑⃑⃑⃑
𝐴𝐷.

Basic Properties of Vector Addition. Familiar laws for real numbers give immediately

(a) 𝑢
⃑⃑ + 𝑤⃑⃑⃑ = 𝑤 ⃑⃑⃑ + 𝑢
⃑⃑. (Commutative Rule)
(b) (𝑢⃑⃑ + 𝑤⃑⃑⃑) + 𝑣⃑ = 𝑢 ⃑⃑ + (𝑤
⃑⃑⃑ + 𝑣⃑) = 𝑤
⃑⃑⃑ + (𝑣⃑ + 𝑢
⃑⃑). (Associative Rule)
⃑⃑⃑ + (−𝑤
(c) 𝑤 ⃑⃑⃑) = 0.

Scalar Multiplication (Multiplication by a Number)

The product 𝑐v ⃑⃑ = 〈v1 , v2 , v3 〉 and any scalar 𝑐


⃑⃑ of any vector v
(real number 𝑐) is the vector obtained by multiplying each
component of v⃑⃑ by 𝑐,

⃑⃑ = 〈𝑐v1 , 𝑐v2 , 𝑐v3 〉.


𝑐v

Basic Properties of Scalar Multiplication. From the definitions


we obtain directly
(a) 𝑐(𝑢
⃑⃑ + 𝑤⃑⃑⃑) = 𝑐𝑢 ⃑⃑ + 𝑐𝑤⃑⃑⃑.
(b) (𝑐 + 𝑑)𝑤 ⃑⃑⃑ = 𝑐𝑤 ⃑⃑⃑ + 𝑑𝑤 ⃑⃑⃑.
(c) 𝑐(𝑘𝑤⃑⃑⃑) = 𝑐𝑘𝑤 ⃑⃑⃑.
Example 2 For the two vectors 𝒗
⃑⃑ = 𝟐𝐢 − 𝐣 + 𝟑𝐤 and 𝒘
⃑⃑⃑⃑ = 𝐢 − 𝟑𝐣 + 𝐤, find 𝟐𝒗
⃑⃑ − 𝒘 ⃑⃑ + 𝟓𝒘
⃑⃑⃑⃑, 𝒗 ⃑⃑⃑⃑.
Solution:
⃑⃑⃑ = 2(2𝐢 − 𝐣 + 3𝐤 )– (𝐢 − 3𝐣 + 𝐤) = 3𝐢 + 𝐣 + 5𝐤.
2𝑣⃑ − 𝑤
𝑣⃑ + 5𝑤
⃑⃑⃑ = 2𝐢 − 𝐣 + 3𝐤 + 5(𝐢 − 3𝐣 + 𝐤) = 7𝐢 − 16𝐣 + 8𝐤.
Example 3 At a certain point during a jump, there are two principal forces acting on a sky diver
gravity exerting a force of 180 pounds straight down and air resistance exerting a force of
180 pounds up and 30 pounds to the right. What is the net force acting on the sky diver?
Solution:
We write the gravity force vector as
⃑g⃑ = 〈0, −180〉,
and the air resistance force vector as
r⃑ = 〈30, 180〉.
The net force on the sky diver is the sum of the two forces,
g⃑⃑ + r⃑ = 〈30, 0〉.

We illustrate the forces in opposite figure. Notice that at this point, the vertical forces are balanced,
producing a “free-fall” vertically, so that the sky diver is neither accelerating nor decelerating vertically.
The net force is purely horizontal, combating the horizontal motion of the sky diver after jumping from the
plane.
-3-
Chapter 1 Vectors

Definition: The two vectors 𝑢


⃑⃑ = 𝑢1 𝐢 + 𝑢2 𝐣 + 𝑢3 𝐤 and 𝑣⃑ = 𝑣1 𝐢 + 𝑣2 𝐣 + 𝑣3 𝐤 are parallel if 𝑢
⃑⃑ = 𝑐 𝑣⃑, where
𝑢1 𝑢2 𝑢3
c is the real number or = = =𝑐
𝑣1 𝑣2 𝑣3

Dot Product (Scalar Product)

The dot product of two vectors 𝑎⃑ = 〈𝑎1 , 𝑎2 , 𝑎3 〉 and 𝑏⃑⃑ = 〈𝑏1 , 𝑏2 , 𝑏3 〉 is denoted by 
a
𝑎⃑ · 𝑏⃑⃑ and is a real number, or scalar, defined in terms of the components of the vectors

as  b
𝑎⃑ ∙ 𝑏⃑⃑ = 𝑎1 𝑏1 + 𝑎2 𝑏2 + 𝑎3 𝑏3
The dot product of two vectors 𝑎⃑ and 𝑏⃑⃑ can also be expressed in terms of the lengths of the vectors and
the angle between them. If the vectors 𝑎⃑ and 𝑏⃑⃑ are positioned in such a manner that their initial points
coincide, then we define the angle between 𝑎⃑ and 𝑏⃑⃑ as the angle 𝜃 that satisfies 0 ≤ 𝜃 ≤ 𝜋.

𝑎⃑ ∙ 𝑏⃑⃑ = ‖𝑎⃑ ‖‖𝑏⃑⃑‖ cos 𝜃

Properties of Dot Product


(i) 𝑎⃑ ∙ 𝑏⃑⃑ = 𝑏⃑⃑ ∙ 𝑎⃑ (commutative law)
(ii) 𝑎⃑ ∙ (𝑏⃑⃑ + 𝑐⃑) = 𝑎⃑ ∙ 𝑏⃑⃑ + 𝑎⃑ ∙ 𝑐⃑ (distributive law)
(iii) 𝑎⃑ ∙ 𝑎⃑ = ‖𝑎‖2 ≥ 0
(iv) 0 ∙ 𝑎⃑ = 0

Angle between Two Vectors


 
The angle 𝜃 between vectors 𝑎⃑ and 𝑏⃑⃑ is given by a a
𝑎⃑ ∙ 𝑏⃑⃑  
cos 𝜃 = with 0 ≤ 𝜃 ≤ 𝜋.  b  b
‖𝑎⃑ ‖‖𝑏⃑⃑‖

Note that: To calculate the angle 𝜽, the vectors must start from the same point or the vectors must
end to the same point.

 
 b b
b
  
a  
a a
     
a b  0 a b  0 a b  0

Orthogonal Vectors (perpendicular vectors)


Two nonzero vectors 𝑎⃑ and 𝑏⃑⃑ are orthogonal (perpendicular) if and only if 𝑎⃑ ∙ 𝑏⃑⃑ = 0.

Example 4 For the two vectors 𝒖 ⃑⃑ = 𝐢 − 𝟐𝐣 − 𝟐𝐤 and 𝒗


⃑⃑ = 𝟔𝐢 + 𝟑𝐣 + 𝟐𝐤. Find 𝒖
⃑⃑ ∙ 𝒗
⃑⃑ and the angle
between them.
Solution:
⃑⃑ ∙ 𝑣⃑ = (1)(6) + (−2)(3) + (−2)(2) = 6 − 6 − 4 = −4.
𝑢
-4-
Chapter 1 Vectors
‖𝑢
⃑⃑‖ = √(1)2 + (−2)2 + (−2)2 = 3, ‖𝑣‖ = √(6)2 + (3)2 + (2)2 =7
𝑢
⃑⃑ ∙ 𝑣⃑ −4
𝜃 = cos−1 ( ) = cos −1 ( ) = 100.9°
‖𝑢
⃑⃑‖‖𝑣⃑‖ 3×7

Example 5 Determine whether the following pairs of vectors are orthogonal:


⃑⃑ = 〈𝟏, 𝟑, −𝟓〉 and ⃑𝒃⃑ = 〈𝟐, 𝟑, 𝟏𝟎〉
(a) 𝒂
⃑⃑ = 〈𝟒, 𝟐, −𝟏〉 and ⃑𝒃⃑ = 〈𝟐, 𝟑, 𝟏𝟒〉.
(b) 𝒂
Solution:
For (a), we have:
𝑎⃑ · 𝑏⃑⃑ = 2 + 9 − 50 = −39 ≠ 0,
so that a and b are not orthogonal.
For (b), we have
𝑎⃑ · 𝑏⃑⃑ = 8 + 6 − 14 = 0,
so that 𝑎⃑ and 𝑏⃑⃑ are orthogonal, in this case.

Example 6 Find the interior angles of the triangle whose vertices 𝑨(𝟏, 𝟐, 𝟎), 𝑩(𝟐, 𝟏, 𝟓) and 𝑪(𝟑, 𝟒, 𝟏).
Solution:
B(2,1, 5)
⃑⃑⃑⃑⃑⃑ = < 1, −1, 5 >, 𝑏⃑⃑ = 𝐴𝐶
To get 𝜃1 , 𝑎⃑ = 𝐴𝐵 ⃑⃑⃑⃑⃑⃑ = < 2, 2, 1 > 3 
c

𝑎⃑ ∙ 𝑏⃑⃑ 2−2+5 5 a
cos 𝜃1 = = ⇒ 𝜃1 = cos −1 ( ) = 71.29°
‖𝑎⃑‖ ‖𝑏⃑⃑‖ √27√9 √27√9 2
1 
𝐵𝐶 = < 1, 3, −4 >, 𝑏⃑⃑ = ⃑⃑⃑⃑⃑⃑
To get 𝜃2 , 𝑐⃑ = ⃑⃑⃑⃑⃑⃑ 𝐴𝐶 = < 2, 2, 1 > b C(3, 4,1)
A(1, 2, 0)
𝑐⃑ ∙ 𝑏⃑⃑ 2+6−4 4
cos 𝜃2 = = ⇒ 𝜃2 = cos−1 ( ) = 74. 84°
‖𝑐⃑‖ ‖𝑏⃑⃑‖ √26√9 √26√9

Finally, 𝜃3 = 180° − (𝜃1 + 𝜃2 ) = 33.86°

Work done

When a constant force of magnitude ‖𝐹⃑ ‖ moves an


object a distanced in the same direction of the force, the work
done is simply = ‖𝐹⃑ ‖‖𝑑⃑‖ .
However, if a constant force 𝐹⃑ applied to a body acts at
an angle 𝜃 to the direction of motion, then the work done by
𝐹⃑ is defined to be the product of the component of 𝐹⃑ in the
direction of the displacement and the distance ‖𝑑⃑‖ that the
body moves

𝑊 = 𝐹⃑ ∙ 𝑑⃑ = ‖𝐹⃑ ‖‖𝑑⃑‖cos 𝜃

-5-
Chapter 1 Vectors
Example 7 Find the work done by a constant force ⃑𝑭⃑ = 𝟑𝐢 − 𝐣 + 𝐤 acting on a particle that moves
along the vector ⃑⃑⃑⃑⃑⃑⃑
𝑨𝑩 where 𝑨(−𝟏, 𝟑, 𝟐) and 𝑩(𝟏, 𝟏, −𝟏).
Solution:
𝑑⃑ = ⃑⃑⃑⃑⃑⃑
𝐴𝐵 = 𝐵 ⃑⃑ − 𝐴⃑ = 2𝐢 − 2𝐣 − 3𝐤, 𝐹⃑ = 3𝐢 − 𝐣 + 𝐤,
𝑊 = 𝐹⃑ ∙ 𝑑⃑ = 6 + 2 − 3 = 5.

Example 8 You exert a constant force of 40 pounds in the direction of the handle of the wagon. If the
handle makes an angle of 𝝅/𝟒 with the horizontal and you pull the wagon along a flat
surface for 1 mile (5280 feet), find the work done.
Solution:
First, the magnitude of the force is 40, the force vector must be
𝜋 𝜋
𝐹⃑ = 〈40 cos , 40 sin 〉 = 〈20√2, 20√2〉
4 4
and the displacement vector is
𝑑⃑ = 〈5280, 0〉
Here, this gives us
𝑊 = 〈20√2, 20√2〉 · 〈5280, 0〉 = 20√2 (5280) ≈ 149,341 𝑓𝑜𝑜𝑡 − 𝑝𝑜𝑢𝑛𝑑𝑠.

Cross Product

If 𝑢
⃑⃑ and 𝑣⃑ are not parallel, they determine a plane. We select a unit vector
𝑛̂ perpendicular to the plane by the right-hand rule. This means that we
choose 𝑛̂ to be the unit (normal) vector that points the way your right thumb
points when your fingers curl through the angle from 𝑢 ⃑⃑ to 𝑤
⃑⃑⃑. Then the cross
product 𝑢 ⃑⃑ × 𝑣⃑ (“𝑢
⃑⃑ cross 𝑣⃑”) is the vector defined as follows.
⃑⃑ × 𝑣⃑ = ‖𝑢
𝑢 ⃑⃑‖ ‖𝑣⃑‖ sin 𝜃 𝑛̂

Properties of Cross Product

(i) 𝑢
⃑⃑ × 𝑣⃑ = − 𝑣⃑ × 𝑢 ⃑⃑
(ii) ⃑⃑ × (𝑣⃑ + 𝑤
𝑢 ⃑⃑⃑) = 𝑢 ⃑⃑ × 𝑣⃑ + 𝑢
⃑⃑ × 𝑤 ⃑⃑⃑ distributive law
(iii) (𝑣⃑ + 𝑤 ⃑⃑⃑) × 𝑢 ⃑⃑ = 𝑣⃑ × 𝑢⃑⃑ + 𝑤
⃑⃑⃑ × 𝑢 ⃑⃑ distributive law
(iv) ⃑⃑ × (𝑘𝑣⃑) = (𝑘𝑢
𝑢 ⃑⃑) × 𝑣⃑ = 𝑘(𝑢 ⃑⃑ × 𝑣⃑), 𝑘 is a scalar
(v) 𝑢
⃑⃑ × 𝑢 ⃑⃑ = 0
(vi) 𝑢 (𝑢
⃑⃑ ∙ ⃑⃑ × 𝑣⃑) = 0
(vii) 𝑣⃑ ∙ (𝑢
⃑⃑ × 𝑣⃑) = 0

Parallel Vectors

Two nonzero vectors 𝑢


⃑⃑ and 𝑤 are parallel if and only if 𝑢 ⃑⃑⃑ = ⃑0⃑.
⃑⃑ × 𝑤

-6-
Chapter 1 Vectors
Calculating the Cross Product as a Determinant

If 𝑢
⃑⃑ = 𝑢1 𝐢 + 𝑢2 𝐣 + 𝑢3 𝐤 and 𝑤
⃑⃑⃑ = 𝑤1 𝐢 + 𝑤2 𝐣 + 𝑤3 𝐤, then
𝐢 𝐣 𝐤
𝑢 ⃑⃑⃑ = | 𝑢1
⃑⃑ × 𝑤 𝑢2 𝑢3 | = (𝑢2 𝑤3 − 𝑢3 𝑤2 )𝐢 − (𝑢1 𝑤3 − 𝑢3 𝑤1 )𝐣 + (𝑢1 𝑤2 − 𝑢2 𝑤1 )𝐤.
𝑤1 𝑤2 𝑤3
The vector 𝑢
⃑⃑ × 𝑤
⃑⃑⃑ is orthogonal to both 𝑢
⃑⃑ and 𝑤
⃑⃑⃑.

Area of the Triangle and Parallelogram


Assume 𝑢 ⃑⃑ and 𝑣⃑ are to vectors represented the two
adjacent sides of triangle, 𝜃 is the angle between them and
ℎ is the height of the triangle, then
ℎ = ‖𝑣⃑‖ sin 𝜃,
1
The area of the triangle = × base × height
2
1
= ‖𝑢
⃑⃑‖ ‖𝑣⃑‖ sin 𝜃
2

∵ ‖𝒖 ⃑⃑‖ = ‖𝒖
⃑⃑ × 𝒗 ⃑⃑‖‖𝒗
⃑⃑‖ 𝒔𝒊𝒏 𝜽
1
∴ The area of the triangle = ‖𝑢
⃑⃑ × 𝑣⃑‖.
2

∴ The area of the parallelogram = ‖𝑢


⃑⃑ × 𝑣⃑‖.
Example 9 Find the cross product between the two vectors ⃑𝒖⃑ = 𝐢 − 𝐣 + 𝟑𝐤 and 𝒘 = 𝟐𝐢 − 𝟒𝐣 + 𝟓𝐤.
Solution:
𝐢 𝐣 𝐤
𝑢
⃑⃑ × 𝑤
⃑⃑⃑ = |1 −1 3 | = 7𝐢 − 𝐣 − 𝟐𝐤.
2 −4 5

Example 10 For the given points 𝑨(𝟏, 𝟐, −𝟏), 𝑩(𝟑, −𝟏, 𝟓) and 𝑪(𝟒, 𝟑, 𝟒). Find
(i) The unit normal vector to the plane containing the points 𝑨, 𝑩 and 𝑪.
(ii) The area of the triangle 𝑨𝑩𝑪.
(iii) Is the triangle 𝑨𝑩𝑪 acute, right or obtuse triangle?
Solution:
(i) ⃑⃑⃑⃑⃑⃑ = 𝑂𝐵
𝐴𝐵 ⃑⃑⃑⃑⃑⃑ − 𝑂𝐴
⃑⃑⃑⃑⃑⃑ = 〈2, −3, 6〉, 𝐴𝐶 ⃑⃑⃑⃑⃑⃑ = 𝐶⃑ − 𝐴⃑ = 〈3, 1, 5〉
𝐢 𝐣 𝐤 ⃑⃑
𝑁 −21𝐢 + 8𝐣 + 11𝐤
⃑𝑁⃑ = ⃑⃑⃑⃑⃑⃑ ⃑⃑⃑⃑⃑⃑
𝐴𝐵 × 𝐴𝐶 = |2 ̂=
−3 6| = −21𝐢 + 8𝐣 + 11𝐤, 𝑁 =
⃑⃑‖ √(−21)2 + (8)2 + (11)2
‖𝑁
3 1 5
1 1 1
(ii) The area of the triangle = ⃑⃑⃑⃑⃑⃑ × ⃑⃑⃑⃑⃑⃑
‖𝐴𝐵 𝐴𝐶 ‖ = √(−21)2 + (8)2 + (11)2 = √626.
2 2 2

(iii) To check on ∠𝐴, ⃑⃑⃑⃑⃑⃑


𝐴𝐵 = < 2, −3, 6 > and ⃑⃑⃑⃑⃑⃑
𝐴𝐶 = < 3, 1, 5 >, then
⃑⃑⃑⃑⃑⃑
𝐴𝐵 ∙ ⃑⃑⃑⃑⃑⃑
𝐴𝐶 = 6 − 3 + 30 = 33 > 0.
∴ ∠𝐴 is an acute angle.
To check on ∠𝐵, ⃑⃑⃑⃑⃑⃑
𝐵𝐴 = < −2,3, −6 > and ⃑⃑⃑⃑⃑⃑
𝐵𝐶 = < 1, 4, −1 >, then
⃑⃑⃑⃑⃑⃑
𝐵𝐴 ∙ ⃑⃑⃑⃑⃑⃑
𝐵𝐶 = −2 + 12 + 6 = 16 > 0.
∴ ∠𝐵 is an acute angle.

-7-
Chapter 1 Vectors
Finally, To check on ∠𝐶, ⃑⃑⃑⃑⃑⃑
𝐶𝐴 = < −3, −1, −5 > and ⃑⃑⃑⃑⃑⃑
𝐶𝐵 = < −1, −4, 1 >, then.
⃑⃑⃑⃑⃑⃑ ∙ 𝐶𝐵
𝐶𝐴 ⃑⃑⃑⃑⃑⃑ = 3 + 4 − 5 = 2 > 0.
∴ ∠𝐶 is an acute angle.
So, the triangle 𝐴𝐵𝐶 is acute triangle.

Sheet 1

1. If 𝑎⃑ = 2 𝐢 − 3 𝐣 + 4 𝐤, 𝑏⃑⃑ = 𝐢 + 2 𝐣 − 3 𝐤 and 𝑐⃑ = 3 𝐢 + 4 𝐣 − 𝐤, find


(i) ‖𝑎⃑ − 2𝑏⃑⃑‖ (ii) 𝑐⃑ + 𝑏⃑⃑ − 5𝑎⃑ (iii) 𝑐⃑ ∙ 𝑏⃑⃑ (iv) 𝑎 × 𝑏 (v) 𝑏⃑⃑ ∙ (𝑐⃑ × 𝑎⃑)

(vi) 𝑏⃑⃑ × (𝑐⃑ × 𝑎⃑) (vii) ‖𝑐⃑‖ 𝑎⃑ (viii) 𝑏̂ × 𝑏⃑⃑ (ix) (𝑏⃑⃑ ∙ 𝑎⃑)𝑐⃑ (x) ‖𝑎⃑ × 𝑐⃑‖

2. Determine whether the following points are colinear (on the same line)
(i) 𝑃(2, 3, 1), 𝑄(4, 2, 2) and 𝑅(8, 0, 4).
(ii) 𝑃(2, 3, 1), 𝑄(0, 4, 2) and 𝑅(4, 1, 4).
3. Use vectors to determine whether the following points form an equilateral triangle
(i) (0, 1, 1), (2, 4, 2) and (3, 1, 4).
(ii) (2, 1, 0), (4, 1, 2) and (4, 3, 0).
4. Use vectors and the Pythagorean Theorem to determine whether the following points form a right
triangle
(i) (3, 1,−2), (1, 0, 1) and (4, 2, −1).
(ii) (1, −2, 1), (4, 3, 2) and (7, 1, 3).

5. In the accompanying figure, two ropes are attached to a 500-pound crate.


Rope A exerts a force of 〈10, −130, 200〉 pounds on the crate, and rope B
exerts a force of 〈−20, 180, 160〉 pounds on the crate. If no further ropes
are added, find the net force on the crate and the direction it will move. If
a third rope 𝐶 is added to balance the crate, what force must this rope exert
on the crate? Find the tension (magnitude of force) in each rope.

6. Find the angles in the quadrilateral 𝐴𝐵𝐶𝐷 with vertices 𝐴(2, 0, 1), 𝐵(2, 1, 4), 𝐶(4, −2, 5) and
𝐷(4, 0, 2).

7. A weight of 500 pounds is supported by two ropes that exert forces of 𝑎 = 〈−100, 200〉 pounds and
𝑏⃑⃑ = 〈100, 300〉 pounds. Find the angles 𝛼, 𝛽 and 𝜃 between the ropes.

-8-
Chapter 1 Vectors

8. Find the work done by a force 𝐹⃑ = 8 𝐢 − 6 𝐣 + 9 𝐤 that moves an object from the point (0, 10, 8) to
the point (6, 12, 20) along a straight line. Note: The distance is measured in meters and the force in
Newtons.
𝜋 𝜋
9. Find the unit vector 𝑎⃑ makes angles with 𝐢, with 𝐣 and an acute angle with 𝐤.
3 4

10. Find 𝜆 and 𝜇 if (2 𝐢 + 6 𝐣 + 27 𝐤) × (𝐢 + 𝜆 𝐣 + 𝜇 𝐤) = ⃑⃑


0

11. Find the area of the triangle with vertices (0, 0, 0), (0, 2, 1) and (1, 3, 0).

12. Find the area of the parallelogram with vertices 𝐾(3, 3, 3), 𝐿(3, 6, 4), 𝑀(6, 12, 4), and 𝑁(6, 9, 3).

13. Find two unit vectors orthogonal to the two given vectors 𝑎⃑ = 〈2, −1, 0〉 and 𝑏⃑⃑ = 〈1, 0, 3〉 using (i) dot
product. (ii) cross product.

Homework 1
1. Find 𝑢 ⃑⃑, ‖𝑢
⃑⃑ − 𝑣⃑, 5𝑢 ⃑⃑‖, the cross product 𝑢
⃑⃑ × 𝑣⃑, the dot product 𝑢
⃑⃑ · 𝑣⃑ and determine whether 𝑢
⃑⃑ and 𝑣⃑
are orthogonal if not find the angle between them:
(i) 𝑢 ⃑⃑ = (1, 2), 𝑣⃑ = (2, −1).
(ii) 𝑢⃑⃑ = (1, 0, 1), 𝑣⃑ = (1, 1, 1).
(iii) 𝑢
⃑⃑ = (𝑘, 0, −𝑘), 𝑣⃑ = (𝑘, 2, 2).
(iv) 𝑢⃑⃑ = (𝑎, 𝑏, 1), 𝑣⃑ = (−𝑏, 2𝑎, 0).

2. Find a vector of magnitude 5 units and parallel to the resultant of the vectors 𝑎⃑ = 2𝐢 + 3𝐣 − 𝐤, 𝑏⃑⃑ =
𝐢− 2𝐣+𝐤
3. Find two-unit vectors orthogonal to each of the vectors 𝑎⃑ + 𝑏⃑⃑ and 𝑎⃑ − 𝑏⃑⃑ if.
(i) 𝑎⃑ = 2 𝐢 + 𝐤, 𝑏⃑⃑ = −𝐢 + 2 𝐣 − 𝐤
(ii) 𝑎⃑ = 3 𝐢 + 𝐣 − 2 𝐤, 𝑏⃑⃑ = 2 𝐢 – 𝐣

4. Show that (𝑎⃑ + 𝑏⃑⃑) × (𝑎⃑ − 𝑏⃑⃑) = −2(𝑎⃑ × 𝑏⃑⃑).


5. Find ‖𝑎⃑ − 𝑏⃑⃑‖, if two vectors 𝑎⃑ and 𝑏⃑⃑ such that ‖𝑎⃑‖ = 2 and ‖𝑏⃑⃑‖ = 3 and 𝑎⃑ ∙ 𝑏⃑⃑ = 4.
6. Find the area of the triangle with vertices 𝐴(1, 1, 2), 𝐵(2, 3, 5) and 𝐶(1, 5, 5).
7. Find the area of the parallelogram with adjacent edges formed by 〈2, 0, 1〉 and 〈0, 1, −3〉.
1
8. Find the area of the rectangle having vertices 𝐴, 𝐵, 𝐶 and 𝐷 with position vectors −𝐢 + 𝐣 + 4 𝐤, 𝐢 +
2
1 1 1
𝐣 + 4 𝐤, 𝐢 − 𝐣 + 4 𝐤, −𝐢 − 𝐣 + 4 𝐤 respectively.
2 2 2
9. Find the work done by a force 𝐹⃑ = − 𝐢 − 3 𝐣 + 2 𝐤 that moves an object from the point (−1, 2, −3) to
the point (5, −1, 7) along a straight line.

-9-
Chapter 1 Vectors
Challenging Problems
1. An airplane has an airspeed of 400 mph. Suppose that the wind
velocity is given by the vector 𝑤⃑⃑⃑ = 〈20, 30〉. In what direction
should the airplane head in order to fly due west (i.e., in the
direction of the unit vector −𝐢 = 〈−1, 0〉)?

2. Write the given vector as 𝑎⃑ + 𝑏⃑⃑, where 𝑎⃑ is parallel to 〈1, 2, 3〉 and 𝑏⃑⃑ is perpendicular to 〈1, 2, 3〉, for
(i) 〈3, −1, 2〉 and (ii) 〈0, 4, 2〉.

3. State whether the following statement are true or false. Explain your answer and if false state a possible
correction
(i) If 𝑎⃑ × 𝑏⃑⃑ = 0 and 𝑎⃑ ∙ 𝑏⃑⃑ = 0 then at least one of the two vectors 𝑎⃑ and 𝑏⃑⃑ is the zero vector.
(ii) If 𝑎⃑ ∙ 𝑏⃑⃑ = 𝑎⃑ ∙ 𝑐⃑ and 𝑎⃑ × 𝑏⃑⃑ = 𝑎⃑ × 𝑐⃑ then 𝑏⃑⃑ = 𝑐⃑ for any vector 𝑎⃑.
√2
4. Find the angle between the two vectors 𝑎⃑ and 𝑏⃑⃑ such that ‖𝑎⃑‖ = 3 and ‖𝑏⃑⃑‖ = and 𝑎⃑ × 𝑏⃑⃑ is an unit
3
vector.
5. Let 𝑎⃑ = 𝐢 + 4 𝐣 + 2 𝐤, 𝑏⃑⃑ = 3 𝐢 − 2 𝐣 + 7 𝐤 and 𝑐⃑ = 2 𝐢 − 𝐣 + 4 𝐤, find a vector 𝑑⃑ which is
perpendicular to both 𝑎⃑ and 𝑏⃑⃑ and 𝑐⃑ ∙ 𝑑⃑ = 15.

- 10 -
Chapter 1 Vectors

Lines and Curves in Plane and Space

Imagine a projectile moving along a path in three-


dimensional space; it could be an electron or a comet, a
soccer ball or a rocket. If you take a snapshot of the object,
its position is described by a static position vector ⃑𝒓⃑ =
〈𝑥, 𝑦, 𝑧〉.
However, if you want to describe the full trajectory of
the object as it unfolds in time, you must use a position
vector such as 𝑟⃑(𝑡) = 〈𝑥(𝑡), 𝑦(𝑡), 𝑧(𝑡)〉 whose
components change in time. The opposite figure illustrates
the position vector 𝑟⃑(𝑡). The goal of this section is to
describe continuous motion by using vector-valued
functions.

Vector-Valued Functions
A function of the form 𝑟⃑(𝑡) = 〈𝑥(𝑡), 𝑦(𝑡), 𝑧(𝑡)〉 may be viewed in two ways.
 It is a set of three parametric equations that describe a curve in space.
 It is also a vector-valued function, which means that the three dependent variables (𝑥, 𝑦, and 𝑧) are
the components of 𝑟⃑, and each component varies with respect to a single independent variable 𝑡 (that
often represents time).

Here is the connection between these two perspectives: As 𝑡 varies, a point (𝑥(𝑡), 𝑦(𝑡), 𝑧(𝑡)) on a
parametric curve is also the head of the position vector 𝑟⃑(𝑡) = 〈𝑥(𝑡), 𝑦(𝑡), 𝑧(𝑡)〉. It is useful to keep both of
these interpretations in mind as you work with vector-valued functions.

Curves in Plane and Space

If the plane curve 𝐶 is defined as the set of ordered pairs (𝑓(𝑡), 𝑔(𝑡)) together with their defining
parametric equations
𝑥 = 𝑓(𝑡) and 𝑦 = 𝑔(𝑡)
where 𝑓(𝑡) and 𝑔(𝑡) are continuous functions of 𝑡 on an interval 𝐼. The vector-valued function of this curve
is given as
𝑟⃑(𝑡) = 𝑓(𝑡) 𝐢 + 𝑔(𝑡) 𝐣 Plane

If the space curve 𝐶 is the set of all ordered triples (𝑓(𝑡), 𝑔(𝑡), ℎ(𝑡)) together with their defining parmetric
equations
𝑥 = 𝑓(𝑡), 𝑦 = 𝑔(𝑡) and 𝑧 = ℎ(𝑡)
where 𝑓(𝑡), 𝑔(𝑡) and ℎ(𝑡) are continuous functions of 𝑡 on an interval 𝐼. The vector-valued function of this
curve is defined as
𝑟⃑(𝑡) = 𝑓(𝑡) 𝐢 + 𝑔(𝑡) 𝐣 + ℎ(𝑡) 𝐤 Space
is a vector-valued function, where the component functions 𝑓(𝑡), 𝑔(𝑡) and ℎ(𝑡) are real-functions of the
parameter 𝑡.
- 11 -
Chapter 1 Vectors

Example 11 Graph the curve traced by the vector function ⃑𝒓⃑(𝒕) = 𝟐 𝐜𝐨𝐬 𝒕 𝐢 + 𝟐 𝐬𝐢𝐧 𝒕 𝐣 + 𝟑𝐤.
Solution: C : x 2  y 2  4, z  3
z
Here,
𝑥 = 2 cos 𝑡 , 𝑦 = 2 sin 𝑡 , 𝑧 = 3
𝑥 2 + 𝑦 2 = 4 cos2 𝑡 + 4 sin2 𝑡 = 4 ⇒ 𝑥 2 + 𝑦 2 = 4, 𝑧 = 3

So, the vector function represents the equation of the circle has radius 2 and placed r (t )
at 𝑧 = 3.
y
x

Example 12 Find the vector function that describes the curve 𝐶 of intersection between the given
surfaces (𝐢) 𝒚 + 𝒙𝟒 = 𝟎, 𝒛 = 𝒙𝟐 . (𝐢𝐢) 𝒙𝟐 + 𝒚𝟐 = 𝟏𝟔, 𝒛 = 𝒚𝟐 . (𝐢𝐢𝐢)𝟏𝟔𝒙𝟐 + 𝒚𝟐 = 𝟏𝟔, 𝒛 = 𝟓.
Solution:
(i) 𝑦 + 𝑥 4 = 0, 𝑧 = 𝑥 2
Assume 𝑥 = 𝑡, 𝑦 = −𝑡 4 and 𝑧 = 𝑡 2 , 𝑟⃑(𝑡) = 𝑡 𝐢 − 𝑡 4 𝐣 + 𝑡 2 𝐤
(ii) 𝑥 2 + 𝑦 2 = 16, 𝑧 = 𝑦 2
Assume 𝑥 = 4 cos 𝑡 , 𝑦 = 4 sin 𝑡 and 𝑧 = 16 sin2 𝑡, 𝑟⃑(𝑡) = 4 cos 𝑡 𝐢 + 4 sin 𝑡 𝐣 + 16 sin2 𝑡 𝐤.
(iii) 16𝑥 2 + 𝑦 2 = 16, 𝑧 = 5
𝑦2 𝑦 2
𝑥2 + = 1, 𝑧 = 5 ⇒ 𝑥 2 + ( ) = 1, 𝑧 = 5.
16 4
𝑦
Assume 𝑥 = cos 𝑡 , = sin 𝑡 and 𝑧 = 5, 𝑟⃑(𝑡) = cos 𝑡 𝐢 + 4 sin 𝑡 𝐣 + 5 𝐤.
4

Derivatives of Vector-Valued Functions


z 
A curve C is defined by 𝑟⃑(𝑡), a vector-valued r ' (t )
function of one variable. Let us imagine that C is   
r (t ) r (t  h)  r (t )
the path taken by a particle and 𝑡 is time. The vector
h
𝑟⃑(𝑡) is the position vector of the particle at time 𝑡 
r (t  h )
and 𝑟⃑(𝑡 + ℎ) is the position vector at a later time
𝑡 + ℎ. The average velocity of the particle in the C O y
time interval x
[𝑡, 𝑡 + ℎ] is then
dicplacement vector 𝑟⃑(𝑡 + ℎ) − 𝑟⃑(𝑡)
=
length of time interval ℎ
𝑟⃑(𝑡 + ℎ) − 𝑟⃑(𝑡)
⃑⃑(𝑡) = 𝑟⃑ ′ (𝑡) = lim
v
ℎ→0 ℎ
𝑥(𝑡 + ℎ) − 𝑥(𝑡) 𝑦(𝑡 + ℎ) − 𝑦(𝑡) 𝑧(𝑡 + ℎ) − 𝑧(𝑡)
= (lim ) 𝐢 + (lim ) 𝐣 + (lim )𝐤
ℎ→0 ℎ ℎ→0 ℎ ℎ→0 ℎ
= 𝑥′(𝑡)𝐢 + 𝑦′(𝑡)𝐣 + 𝑧′(𝑡)𝐤
or
𝑑𝑟⃑ 𝑑𝑥(𝑡) 𝑑𝑦(𝑡) 𝑑𝑧(𝑡)
𝑟⃑ ′ (𝑡) =
= 𝐢+ 𝐣+ 𝐤
𝑑𝑡 𝑑𝑡 𝑑𝑡 𝑑𝑡
where 𝑥(𝑡), 𝑦(𝑡) and 𝑧(𝑡) are differentiable function.

Example 13 If ⃑𝒓⃑(𝒕) = (𝒕𝟑 − 𝟐𝒕𝟐 )𝐢 + 𝟒𝒕 𝐣 + 𝒆−𝒕 𝐤, find ⃑𝒓⃑′(𝒕) and ⃑𝒓⃑′′(𝒕).


Solution:
𝑑𝑟⃑ 𝑑 2 𝑟⃑
𝑟⃑ ′ (𝑡) = = (3𝑡 2 − 4𝑡)𝐢 + 4 𝐣 − 𝑒 −𝑡 𝐤, 𝑟⃑ ′′ (𝑡) = = (6𝑡 − 4)𝐢 + 0 𝐣 + 𝑒 −𝑡 𝐤
𝑑𝑡 𝑑𝑡 2

- 12 -
Chapter 1 Vectors

Lines in Space

Two distinct points in ℝ3


determine a unique line. Alternatively,
one point and a direction also
determine a unique line. We use both of
these properties to derive parametric
equations for lines in space. The result
is an example of a vector-valued
function in ℝ3 .

Let ℓ be the line passing through the


point 𝑃0 (𝑥0 , 𝑦0 , 𝑧0 ) parallel to the
nonzero vector 𝐯⃑⃑ = 〈𝑎, 𝑏, 𝑐〉, where 𝑃0 and 𝐯⃑⃑ are given.
The fixed point 𝑃0 is associated with the position vector
⃑𝒓⃑0 = ⃑⃑⃑⃑⃑⃑⃑⃑
𝑂𝑃0 = 〈𝑥0 , 𝑦0 , 𝑧0 〉.
We let 𝑃(𝑥, 𝑦, 𝑧) be a variable point on ℓ with
⃑𝒓⃑ = ⃑⃑⃑⃑⃑⃑
𝑂𝑃 = 〈𝑥, 𝑦, 𝑧〉
the position vector associated with 𝑃. Because ℓ is parallel to 𝐯⃑⃑, the vector ⃑⃑⃑⃑⃑⃑⃑⃑
𝑃0 𝑃 is also parallel to 𝐯⃑⃑;
therefore,
⃑⃑⃑⃑⃑⃑⃑⃑
𝑃0 𝑃 = 𝑡 𝐯⃑⃑,
where 𝑡 is a real number. By vector addition, we see that ⃑⃑⃑⃑⃑⃑
𝑂𝑃 = ⃑⃑⃑⃑⃑⃑⃑⃑ 𝑃0 𝑃, or ⃑⃑⃑⃑⃑⃑
𝑂𝑃0 + ⃑⃑⃑⃑⃑⃑⃑⃑ 𝑂𝑃 = ⃑⃑⃑⃑⃑⃑⃑⃑
𝑂𝑃0 + 𝑡 𝐯⃑⃑. It follows
that
〈𝑥, 𝑦, 𝑧〉 = 〈𝑥0 , 𝑦0 , 𝑧0 〉 + 𝑡〈𝑎, 𝑏, 𝑐〉 or ⃑𝒓⃑(𝒕) = 𝒓𝟎 + 𝑡 𝐯⃑⃑
Equating the components, the line is described by the parametric equations
𝑥 = 𝑥0 + 𝑎𝑡, 𝑦 = 𝑦0 + 𝑏𝑡, 𝑧 = 𝑧0 + 𝑐𝑡, for −∞ < 𝑡 < ∞.

Summary
The Vector Valued Function for a Line
The vector valued function for the line ℓ through 𝑃0 (𝑥0 , 𝑦0 , 𝑧0 ) and parallel to the vector 𝐯⃑⃑ is
⃑𝒓⃑(𝑡) = ⃑⃑⃑⃑⃑
𝒓0 + 𝑡𝐯⃑⃑ , −∞<𝑡 <∞
where ⃑𝒓⃑ is the position vector of any point (𝑥, 𝑦, 𝑧) on ℓ and ⃑⃑⃑⃑⃑
𝒓0 is the position vector of 𝑃0 (𝑥0 , 𝑦0 , 𝑧0 ).

The Parametric Equations for a Line


The parametric equation of the line through 𝑃0 (𝑥0 , 𝑦0 , 𝑥0 ) and parallel to the vector 𝐯⃑⃑ is
𝑥 = 𝑥0 + 𝑎𝑡, 𝑦 = 𝑦0 + 𝑏𝑡, 𝑧 = 𝑧0 + 𝑐𝑡, − ∞ < 𝑡 < ∞.

The Standard Equations for a Line

The standard equation of the line through 𝑃0 (𝑥0 , 𝑦0 , 𝑥0 ) parallel to the vector 𝐯⃑⃑ is
𝑥 − 𝑥0 𝑦 − 𝑦0 𝑧 − 𝑧0
= =
𝑎 𝑏 𝑐

- 13 -
Chapter 1 Vectors
Example 14 Find the equation of the line satisfies the given conditions:
(a) The line passes through 𝑷(𝟑, −𝟒, −𝟏) and parallel to the vector 𝐢 + 𝐣 + 𝐤.
(b) The line that passes through 𝑷(−𝟐, 𝟎, 𝟑) and 𝑸 (𝟑, 𝟓, −𝟐).
(c) The line through the point (𝟑, −𝟐, 𝟏) and parallel to the line ⃑𝒓⃑ =< 𝟏, 𝟐, 𝟎 >
+ 𝒕 < 𝟐, −𝟏, 𝟑 >
Solution:
(a) The line passes through 𝑃(3, −4, −1) and parallel to the vector 𝐢 + 𝐣 + 𝐤.
The direction vector 𝑎⃑ = 𝐢 + 𝐣 + 𝐤, the initial point 𝑃(3, −4, −1). The equation of the line in parametric
form is 𝑥 = 3 + 𝑡, 𝑦 = −4 + 𝑡, 𝑧 = −1 + 𝑡.

(b) The line that passes through 𝑃(−2, 0,3) and 𝑄 (3, 5, −2).
The direction vector 𝑎⃑ = ⃑⃑⃑⃑⃑⃑
𝑃𝑄 = 5𝐢 + 5𝐣 − 5𝐤, the initial point 𝑃(−2, 0, 3). The equation of the line in
parametric form is 𝑥 = −2 + 5𝑡, 𝑦 = 0 + 5𝑡, 𝑧 = 3 − 5𝑡.

(c) The line through the point (3, −2,1) and parallel to the line 𝑟 =< 1,2,0 > +𝑡 < 2, −1,3 >
The direction vector 𝑎⃑ = 2𝐢 − 𝐣 + 3𝐤, the initial point 𝑃(3, −2, 1). The equation of the line in
parametric form is 𝑥 = 3 + 2𝑡, 𝑦 = −2 − 𝑡, 𝑧 = 1 + 3𝑡.

Example 15 Find the equation of the straight line through the point (𝟑, 𝟏, 𝟐) and parallel to the
vector −𝐢 + 𝟐𝐣 − 𝟑𝐤. Show that this straight line passes through the point 𝑸(𝟏, 𝟓, −𝟒).
Solution:
The direction vector 𝑎⃑ = − 𝐢 + 2𝐣 − 3𝐤, the initial point 𝑃(3, 1,2). The equation of the line in parametric
form is 𝑥 = 3 − 𝑡, 𝑦 = 1 + 2𝑡, 𝑧 = 2 − 3𝑡.

𝑥= 1⇒𝑡=2
At 𝑄(1,5, −4) ⇒ 𝑦 = 5 ⇒ 𝑡 = 2 ⇒ ∴ the point 𝑄(1,5, −4) lies on the line.
𝑧 = −4 ⇒ 𝑡 = 2

Example 16 Find the equation of the straight line through the point (𝟏, 𝟏, 𝟏) and perpendicular to
the two vectors 𝐢 − 𝟓𝐣 + 𝟐𝐤 and 𝟐𝐢 + 𝐣 − 𝐤.
Solution:
The initial point is (1, 1, 1) and the direction vector 𝑎⃑ is perepenticular to the vectors 𝐢 − 5𝐣 + 2𝐤 and 2𝐢 +
𝐣 − 𝐤. So,
𝐢 𝐣 𝐤
𝑎⃑ = |1 −5 2 | = 3𝐢 + 5𝐣 + 11𝐤
2 1 −1
The equation of the line in parametric form is 𝑥 = 1 + 3𝑡, 𝑦 = 1 + 5𝑡, 𝑧 = 1 + 11𝑡.
Example 17 Find the equation of the straight line through the point (−𝟐, 𝟑, −𝟓) and perpendicular
to the two lines:
⃑𝒓⃑𝟏 =< 𝟑, 𝟏, 𝟐 > + 𝒕𝟏 < 𝟒, 𝟑, 𝟏 > and ⃑𝒓⃑𝟐 =< 𝟑, 𝟔, 𝟑 > + 𝒕𝟐 < −𝟐, 𝟏, 𝟎 >.
Solution:
The initial point is (−2, 3, −5). The direction vector of the first line is 𝑎⃑1 = 4𝐢 + 3𝐣 + 𝐤. The direction
vector of the second line is 𝑎⃑2 = −2𝐢 + 𝐣 + 0𝐤.

𝐢 𝐣 𝐤
The direction vector of the required line is 𝑎⃑ = 𝑎⃑1 × 𝑎⃑2 = | 4 3 1| = −𝐢 − 𝟐𝐣 + 𝟏𝟎𝐤. The equation
−2 1 0
of the line in parametric form is
𝑥 = −2 − 𝑡3 , 𝑦 = 3 − 2𝑡3 , 𝑧 = −5 + 10𝑡3 .

- 14 -
Chapter 1 Vectors
Example 18 Prove that the following lines are parallel
⃑𝒓⃑𝟏 = 〈𝟓, 𝟏, −𝟑〉 + 𝒕〈−𝟑, 𝟐, 𝟒〉 and ⃑𝒓⃑𝟐 = 〈−𝟏, 𝟐, 𝟒〉 + 𝐬〈𝟔, −𝟒, −𝟖〉
Solution:
The direction vectors of the two lines are 𝑎⃑1 = 〈−3,2,4〉 and 𝑎⃑2 = 〈6, −4, −8〉
Because 𝑎⃑1 = −2 𝑎⃑2 ,
Therefore, 𝑎⃑1 and 𝑎⃑2 are parallel, then the two lines are parallel.
Example 19 Prove that the following lines are intersected
⃑𝒓⃑𝟏 = 〈𝟐, 𝟑, 𝟏〉 + 𝒕〈𝟏, −𝟏, −𝟏〉 and ⃑𝒓⃑𝟐 = 〈𝟏, 𝟐, 𝟒〉 + 𝒔〈𝟏, 𝟎, −𝟐〉,
then find the intersection point and the acute angle between them.
Solution:
The direction of the two lines are 𝑎⃑1 = 〈1, −1, −1〉 and 𝑎⃑2 = 〈1,0, −2〉.
The parametric form of the two lines are
ℓ1 : 𝑥 = 2 + 𝑡, 𝑦 = 3 − 𝑡, 𝑧 =1−𝑡
and
ℓ2 : 𝑥 = 1 + 𝑠, 𝑦 = 2, 𝑧 = 4 − 2𝑠
by equating the equations of the two lines to check for the intersection
𝑥1 = 𝑥2 : 2 + 𝑡 = 1 + 𝑠 ---(1)
𝑦1 = 𝑦2 : 3 − 𝑡 = 2 ⇒ 𝑡 = 1 ---(2)
𝑧1 = 𝑧2 : 1 − 𝑡 = 4 − 2𝑠 ---(3)
Substitute equation (2) into equation (3), yields 𝑠 = 2. By substituting the result into equation (1) yield
1−1=4−4
∴ ℓ1 and ℓ2 are intersected lines
at 𝑡 = 1, the intersection point is (3, 2, 0) or at 𝑠 = 2, the intersection point is (3,2,0).
𝑎⃑⃑1 ∙ 𝑎⃑⃑2 1+0+2
the angle between the two lines is 𝜃 = cos −1 ( ) = cos−1 ( ) = 39.23°
||𝑎⃑⃑1 || ||𝑎⃑⃑2 || √3√5

Example 20 Prove that the following lines are skew


⃑𝒓⃑𝟏 = 〈𝟑, 𝟒, 𝟏〉 + 𝒕〈−𝟏, 𝟏, 𝟐〉 and ⃑𝒓⃑𝟐 = 〈𝟏, 𝟓, 𝟕〉 + 𝒔〈𝟏, −𝟏, −𝟏〉.
Solution:
The direction of the two lines are 𝑎⃑1 = 〈−1, 1, 2〉 and 𝑎⃑2 = 〈1, −1, −1〉.
Because 𝑎⃑1 ≠ c 𝑎⃑2 , therefore, 𝑎⃑1 and 𝑎⃑2 are not parallel, then the two lines are not parallel.
The parametric form of the two lines are
ℓ1 : 𝑥 = 3 − 𝑡, 𝑦 = 4 + 𝑡, 𝑧 = 1 + 2𝑡
and
ℓ2 : 𝑥 = 1 + 𝑠, 𝑦 = 5 − 𝑠, 𝑧 =7−𝑠
by equating the equations of the two lines to check for the intersection
𝑥1 = 𝑥2 : 3 − 𝑡 = 1 + 𝑠 ---(1)
𝑦1 = 𝑦2 : 4 + 𝑡 = 5 − s ---(2)
𝑧1 = 𝑧2 : 1 + 2𝑡 = 7 − 𝑠 ---(3)
By adding equation (2) and equation (3), yields 𝑠 = 4 and 𝑡 = −2. By substituting the result into equation
(1) yield
4+4≠5+2
∴ ℓ1 and ℓ2 are not intersected lines.
∴ ℓ1 and ℓ2 are skew lines.

- 15 -
Chapter 1 Vectors

Plane 
n   a, b, c 
A plane in space is determined by knowing a point on
the plane and its orientation. This orientation is defined by
specifying a vector that is normal to the plane. Q ( x, y , z )
Suppose that plane 𝜋 passes through a point
𝑃(𝑥0 , 𝑦0 , 𝑧0 ) and its normal is non zero vector 𝑛⃑⃑ = 𝑎 𝐢 +
𝑏 𝐣 + 𝑐 𝐤. Then 𝜋 is the set of all points 𝑄(𝑥, 𝑦, 𝑧) for which P( x0 , y0 , z0 ) 
⃑⃑⃑⃑⃑⃑
𝑃𝑄 is orthogonal to 𝑛⃑⃑.
Thus, the dot product 𝑛⃑⃑ ∙ ⃑⃑⃑⃑⃑⃑
𝑃𝑄 = 0. This equation equavilent to

(𝑎𝐢 + 𝑏𝐣 + 𝑐𝐤) ∙ [(𝑥 − 𝑥0 )𝐢 + (𝑦 − 𝑦0 )𝐣 + (𝑧 − 𝑧0 )𝐤] = 0


or
𝑎(𝑥 − 𝑥0 ) + 𝑏(𝑦 − 𝑦0 ) + 𝑐(𝑧 − 𝑧0 ) = 0

Equation for a Plane


z
The plane through 𝑃(𝑥0 , 𝑦0 , 𝑧0 ) normal to 𝑛⃑⃑ = 𝑎𝐢 + 𝑏𝐣 + 𝑐𝐤 has (0, 0, C )
Standard Form: 𝑎(𝑥 − 𝑥0 ) + 𝑏(𝑦 − 𝑦0 ) + 𝑐(𝑧 − 𝑧0 ) = 0
General Form: 𝑎𝑥 + 𝑏𝑦 + 𝑐𝑧 = 𝑑, where 𝑑 = 𝑎𝑥0 + 𝑏𝑦0 + 𝑐𝑧0

𝑥 𝑦 𝑧 𝑑 𝑑 𝑑 y
𝐈𝐧𝐭𝐞𝐫𝐜𝐞𝐩𝐭 𝐅𝐨𝐫𝐦: + + = 1, where 𝐴, 𝐵 and 𝐶 are , and (0, B, 0)
𝐴 𝐵 𝐶 𝑎 𝑏 𝑐
recpectivily. ( A, 0, 0)
x
Example 21 Find an equation of the plane 𝝅 has intercepts 3, 5, 7 with 𝒙, 𝒚 and 𝒛 recpectivily.

Solution:
The equation of the plane is
𝑥 𝑦 𝑧
+ + =1
3 5 7
Example 22 Find an equation of the plane 𝝅 containing the point 𝑷(𝟑, 𝟏, 𝟐) and ⃑𝒏⃑ = 𝟐𝐢 + 𝐣 + 𝐤.
Solution:
⃑⃑⃑⃑⃑⃑
𝑃𝑄 = (𝑥 − 3)𝐢 + (𝑦 − 1)𝐣 + (𝑧 − 2)𝐤, 𝑛⃑⃑ = 2𝐢 + 𝐣 + 𝐤
(2𝐢 + 𝐣 + 𝐤 ) ∙ [(𝑥 − 3)𝐢 + (𝑦 − 1)𝐣 + (𝑧 − 2)𝐤] = 0
2(𝑥 − 3) + (𝑦 − 1) + (𝑧 − 2) = 0 ⇒ 2𝑥 + 𝑦 + 𝑧 − 9 = 0

The equation of the plane 𝜋 is


2𝑥 + 𝑦 + 𝑧 − 9 = 0.

Example 23 Find an equation of the plane 𝝅 containing the three point 𝑨(𝟒, −𝟑, 𝟏), 𝑩(𝟔, −𝟒, 𝟕) and
𝑪(𝟏, 𝟐, 𝟐).
Solution:
To write the equation of the plane 𝜋, the normal vector to the plane should be calculated. First, any two
vectors in the plane will calculate. So,

- 16 -
Chapter 1 Vectors
⃑⃑⃑⃑⃑⃑
𝐴𝐵 = 2𝐢 − 𝐣 + 6𝐤, ⃑⃑⃑⃑⃑⃑
𝐴𝐶 = −3𝐢 + 5𝐣 + 𝐤
𝐢 𝐣 𝐤
⃑⃑⃑⃑⃑⃑ ⃑⃑⃑⃑⃑⃑
𝑛⃑⃑ = 𝐴𝐵 × 𝐴𝐶 = | 2 −1 6| = −31𝐢 − 20𝐣 + 7𝐤
−3 5 1
⃑⃑⃑⃑⃑⃑ = (𝑥 − 4)𝐢 + (𝑦 + 3)𝐣 + (𝑧 − 1)𝐤, 𝑛⃑⃑ = −31𝐢 − 20𝐣 + 7𝐤
𝑃𝐴

(−31𝐢 − 20𝐣 + 7𝐤) ∙ [(𝑥 − 4)𝐢 + (𝑦 + 3)𝐣 + (𝑧 − 1)𝐤] = 0

−31(𝑥 − 4) − 20(𝑦 + 3) + 7(𝑧 − 1) = 0 ⇒ −31𝑥 − 20𝑦 + 7𝑧 + 57 = 0

The equation of the plane 𝜋 is


−31𝑥 − 20𝑦 + 7𝑧 + 57 = 0

Example 24 Prove that the two planes 𝝅𝟏 : 𝟓𝒙 + 𝒚 − 𝒛 + 𝟏𝟎 = 𝟎 and 𝝅𝟐 : 𝒙 − 𝟐𝒚 + 𝟑𝒛 + 𝟏 =0 are


perpendicular.
Solution:
The normal vector of the first plane is 𝑛⃑⃑1 = 〈5, 1, −1〉 and the normal vector of the first plane is 𝑛⃑⃑2 =
〈1, −2,3〉.
Moreover, because 𝑛⃑⃑1 ∙ 𝑛⃑⃑2 = 〈5, 1, −1〉 ∙ 〈1, −2,3〉 = 5 − 2 − 3 = 0, 𝑛⃑⃑1 and 𝑛⃑⃑2 are perpendicular
then 𝜋1 and 𝜋2 are perpendicular.

Example 25 Find the angle between the two planes 𝝅𝟏 : 𝟐𝒙 − 𝒚 + 𝟐𝒛 + 𝟓 = 𝟎 and 𝝅𝟐 : 𝟔𝒙 − 𝟕𝒚 −


𝟔𝒛 − 𝟗 =0.
Solution:
The normal vector of the first plane is 𝑛⃑⃑1 = 〈2, −1, 2〉and the normal vector of the first plane is 𝑛⃑⃑2 =
〈6, −7, −6〉.

The angle between the two planes is the angle between the two normal vectors. So,
𝑛⃑⃑1 ∙ 𝑛⃑⃑2 12 + 7 − 12
𝜃 = cos −1 ( ) = cos −1 ( ) = 77.75°
‖𝑛⃑⃑1 ‖ ‖𝑛⃑⃑2 ‖ √9√121
Example 26 Write an equation of the plane that is parallel to the plane 𝒙 − 𝟑𝒚 + 𝟒𝒛 + 𝟐𝟒 = 𝟎 and
passes through the point (𝟐, −𝟏, 𝟑).
Solution:
Because the required plane is parallel to the plane 𝑥 − 3𝑦 + 4𝑧 + 24 = 0, the normal vector of the required
plane is the same normal vector of the given plane.
The normal vector is 𝑛⃑⃑ = 〈1, −3, 4〉 , 𝑃(2, −1, 3) and ⃑⃑⃑⃑⃑⃑
𝑃𝑄 = (𝑥 − 2)𝐢 + (𝑦 + 1)𝐣 + (𝑧 − 3)𝐤.

(𝐢 − 3𝐣 + 4𝐤) ∙ [(𝑥 − 2)𝐢 + (𝑦 + 1)𝐣 + (𝑧 − 3)𝐤] = 0

(𝑥 − 2) − 3(𝑦 + 1) + 4(𝑧 − 3) = 0 ⇒ 𝑥 − 3𝑦 + 4𝑧 − 17 = 0

The equation of the plane 𝜋 is


𝑥 − 3𝑦 + 4𝑧 − 17 = 0.

Example 27 Find the point of intersection and the angle between the plane 𝟐𝒙 − 𝒚 + 𝟑𝒛 = 𝟔 and
the line ⃑𝒓⃑ = 〈𝟏, 𝟎, 𝟏〉 + 𝒕〈𝟏, 𝟑, 𝟏〉Write an equation of the plane that is parallel to the
plane 𝒙 − 𝟑𝒚 + 𝟒𝒛 + 𝟐𝟒 = 𝟎 and passes through the point (𝟐, −𝟏, 𝟑).
Solution:

- 17 -
Chapter 1 Vectors
The direction vector of the line 𝑎⃑ = 〈1, 3, 1〉and the  l
normal vector of the plane is 𝑛⃑⃑ = 〈2, −1, 3〉. n
The angle between the line and the plane is
Intersection Point
𝜃 = |90 − 𝛼|, 
a
where 
𝛼 is the angle between 𝑎⃑ and 𝑛⃑⃑. So, p 

𝑎⃑ ⋅ 𝑛⃑⃑ 2−3+3 
𝛼 = cos −1 = cos−1
‖𝑎⃑‖ ‖𝑛⃑⃑‖ √11√14
∴ 𝛼 = 80.42° and 𝜃 = 9.27°

To get the intersection point, we solve the equation of the line in parametric form and the equation of
the plane. The parametric form of the line is
𝑥 = 1 + 𝑡, 𝑦 = 3𝑡, 𝑧 = 1 + 𝑡.
By substituting the equation of the line into the equation of the plane, yield
1
2 + 2𝑡 − 3𝑡 + 3 + 3𝑡 = 6 ⇒ 𝑡 =
2
3 3 3
Then, the intersection point is 𝑝 ( , , )
2 2 2

Sheet 2

1. Find the vector function that describes the curve 𝐶 of intersection between the given surfaces.
(i) 𝑥 2 + 𝑦 2 = 16, 𝑧 = 5.
(ii) 𝑥 = 𝑦 − 1, 𝑦 = 𝑧 + 3.
(iii) 𝑧 = 𝑦 3 ,𝑦 = 𝑥 2 .
(iv) 𝑧 = 𝑥 2 + 𝑦 2 , 𝑧 = 3.
(v) 𝑧 = 𝑥2 + 𝑦2, 𝑧 = 8 − 𝑥2 − 𝑦2.

2. Find 𝑟⃑′(𝑡), 𝑟⃑′′(𝑡) for the given vector functions

(i) 𝑟⃑(𝑡) = 〈1 − 𝑡 2 , 𝑡 3 , 𝑡〉.


(ii) 𝑟⃑(𝑡) = 2 cos 𝑡 𝐢 + 2 sin 𝑡 𝐣 + 𝑡 𝐤.
(iii) 𝑧 = √𝑥 2 + 𝑦 2 , 𝑧 = 5.

3. Find parametric equations and symmetric equations of the line satisfying the following conditions
(i) the line through (3, 2, 4) and parallel to 〈3, 2, 1〉.
𝑥+1 𝑦
(ii) the line through (1, 0, 0) and parallel to the line = = 𝑧 − 2.
−2 3
(iii) the line through (3, 1, 0) and perpendicular to both 〈0, 3, 1〉 and 〈4, 2, 1〉.

4. State whether the lines are parallel or perpendicular and find the angle between the lines

ℓ1 : 𝑥 = 4 − 2𝑡, 𝑦 = 3𝑡, 𝑧 = −1 + 2𝑡,


ℓ2 : 𝑥 = 4 + 𝑠, 𝑦 = −2𝑠, 𝑧 = −1 + 3𝑠.

5. Find the angle between the following two lines


𝑥−2 𝑧+1 𝑥+7 𝑦 𝑧+1
(i) =𝑦= and =− = .
2 −3 −6 3 9

- 18 -
Chapter 1 Vectors
(ii) ℓ1 : 𝑥 = 1 + 2𝑡, 𝑦 = 1 − 𝑡, 𝑧 = 7 − 2𝑡 and ℓ2 : 𝑥 = 6 + 3𝑡, 𝑦 = 3 − 2𝑡, 𝑧 = 2 + 8𝑡.
6. Which is true for the lines that are described by the parametric equations:

ℓ1 : 𝑥 = 1 + 𝑡, 𝑦 = 3 − 2𝑡, 𝑧 = 1 − 𝑡,
ℓ2 : 𝑥 = −1 − 𝑠, 𝑦 = −3 + 2𝑠, 𝑧 = −1 + 𝑠.
(i) the lines coincide.
(ii) the lines are parallel.
(iii) the lines intersect at exactly one point.

7. Find an equation of line that passes through the point (1, 1, 1) and perpendicular to the line

ℓ: 𝑥 = 3 + 2𝑡, 𝑦 = 4 − 4𝑡, 𝑧 = 5 − 6𝑡.

8. Determine whether the following lines intersect. If so, find their point of intersection and the equation
of the line passes through this point and perpendicular to both

ℓ1 : 𝑥 = −2 + 3𝑡, 𝑦 = 5 − 4𝑡, 𝑧 = 1 + 2𝑡,


ℓ2 : 𝑥 = 1 − 𝑠, 𝑦 = 3 + 2𝑠, 𝑧 = −4 − 3𝑠.

9. Find the equation of the plane satisfying the following conditions


(i) The plane contains the three points 𝑃1 (1, 0, 1), 𝑃2 (0, 1, 1) and 𝑃3 (1, 1, 0).
(ii) The plane contains the line 𝑥 = 1 + 2𝑡, 𝑦 = 𝑡; 𝑧 = 3 − 𝑡 and is parallel to the plane
2𝑥 + 4𝑦 + 8𝑧 = 1.
𝑥+1 𝑦
(iii) The plane passes through the point (1, −2, 7) perpendicular to the line = = 𝑧 − 2.
−2 3
(iv) The plane that passes through the point (6, 0, 0) and contains the line 𝑥 = 4 − 2𝑡, 𝑦 = 2 +
3𝑡, 𝑧 = 3 + 5𝑡.
(v) The plane which passes through (4,1, −3) and perpendicular to each of the planes 𝜋1 : 3𝑥 −
5𝑦 + 2𝑧 + 15 = 0 and 𝜋2 : 3𝑥 − 𝑦 + 7𝑧 − 21 = 0.

Homework 2
1. Find (a) parametric equations and (b) symmetric equations of the line satisfying the following conditions
(i) The line through (2, −1, −3) and (0, 2, −3)
(ii) The line through (−1, 0, 2) and (−3, 0, −2)
𝑥−1 𝑧+2
(iii)The line through (2, −1, 1) and parallel to = 2𝑦 =
2 −3

2. Find the angle between the lines


(i) ℓ1 : 𝑥 = 4 + 𝑡, 𝑦 = 2, 𝑧 = 3 + 2𝑡, ℓ2 : 𝑥 = 4 + 2𝑠, 𝑦 = 2 + 2𝑠, 𝑧 = 3 + 4𝑠.
(ii) ℓ1 : 𝑥 = 3 + 𝑡, 𝑦 = 3 + 3𝑡, 𝑧 = 4 − 𝑡, ℓ2 : 𝑥 = 3 − 𝑠, 𝑦 = 3 − 2𝑠, 𝑧 = 4 + 2𝑠.

3. Find the distance from the point 𝑄 to the given line


(i) 𝑄(1, −1, 0), ℓ: 𝑥 = 1 + 𝑡, 𝑦 = −1 + 2𝑡, 𝑧 = 3.
(ii) 𝑄(0, 1, 0), ℓ: 𝑥 = −1 + 2𝑡, 𝑦 = 4𝑡, 𝑧 = 2 + 3𝑡.

4. Determine whether the lines are parallel, skew or intersect.


(i) ℓ1 : 𝑥 = 2𝑡, 𝑦 = 3 + 𝑡, 𝑧 = −1 + 4𝑡, ℓ2 : 𝑥 = 4, 𝑦 = 4 + 𝑠, 𝑧 = 3 + 𝑠.
(ii) ℓ1 : 𝑥 = 1 − 𝑡, 𝑦 = 2𝑡, 𝑧 = −1 + 4𝑡, ℓ2 : 𝑥 = 3 + 3𝑠, 𝑦 = 2, 𝑧 = 1 − 3𝑠.
- 19 -
Chapter 1 Vectors
5. Find the equation of the plane satisfying the following conditions
(i) The plane contains the three points 𝑃1 (0, 0, 1), 𝑃2 (0, −5, 0) and 𝑃3 (0, 0, 7).
(ii) The plane passes through the point (−3, 7, 1) and parallel to the plane 2𝑥 + 4𝑦 + 8𝑧 = 1.
(iii)The plane passes through the two points (1, −2, 7) and (1, −7, 2) perpendicular to the plane 2𝑥 −
𝑦 + 4𝑧 = −1.
(iv) The plane contains 𝑟⃑1 = 〈5,1, −3〉 + 𝑡〈−3,2,4〉 and 𝑟⃑2 = 〈−1,2,4〉 + s〈6, −4, −8〉.

6. Evaluate the distance between the point (1, 3, 4) and the plane 𝑥 − 2𝑦 + 𝑧 = 10.
7. Find the line of intersection between the two planes 𝑥 − 𝑦 + 𝑧 = 2 and 𝑥 + 4𝑦 + 𝑧 = 7.

Challenging Problems

1. Find the distance from the point 𝑃(1, 1,1) to the line ℓ: 𝑥 = 1 + 𝑡, 𝑦 = −1 − 2𝑡, 𝑧 = 2 + 3𝑡.
2. Prove that the following two lines are skew then find the shortest distance between them

ℓ1 : 𝑥 = 1 + 𝑡, 𝑦 = 3 − 𝑡, 𝑧 = −5𝑡,
ℓ2 : 𝑥 = −2 + 7𝑠, 𝑦 = −1 + 2𝑠, 𝑧 = 5 − 2𝑠.

3. Show that ℓ1 and ℓ2 are parallel and find the shortest distance between them
ℓ1 : 𝑥 = 2 − 𝑡, 𝑦 = 2𝑡, 𝑧 = 1 + 𝑡,
ℓ2 : 𝑥 = 1 + 2𝑠, 𝑦 = 3 − 4𝑠, 𝑧 = 5 − 2𝑠.
then find an equation of the line passes through the point (1, 3, 5) and perpendicular to the plane
containing both ℓ1 and ℓ2 .

4. Prove that the two planes 2𝑥 + 7𝑦 − 𝑧 = 2 and −4𝑥 − 14𝑦 + 2𝑧 = 10 are parallel then find the
distance between them.

5. Find the equation of the plane containing the line of intersection of the two planes 𝑥 + 𝑦 + 𝑧 + 5 =
0 and 3𝑥 + 2𝑦 − 𝑧 + 2 = 0 and perpendicular to the plane 𝑥 − 𝑦 − 𝑧 − 5 = 0.

6. Find the equation of the plane has 3 units from the origin and perpendicular to the line 𝑥 = 3 + 𝑡, 𝑦 =
3 + 3𝑡, 𝑧 = 4 − 𝑡.

- 20 -
Chapter 1 Vectors

Scalar and Vector Fields


A field is a quantity which can be specified everywhere in space as a function of position. The quantity
that is specified may be a scalar or a vector. For instance, we can specify the temperature at every point in
a room. The room may, therefore, be said to be a region of "temperature field" which is a scalar field because
the temperature 𝑇(𝑥, 𝑦, 𝑧) is a scalar function of the position. An example of a scalar field in
electromagnetism is the electric potential.
In a similar manner, a vector quantity which can be specified at every point in a region of space is a
vector field. For instance, every point on the earth may be considered to be in the gravitational force field
of the earth. We may specify the field by the magnitude and the direction of acceleration due to gravity at
every point in space. As another example consider flow of water in a pipe. At each point in the pipe, the
water molecule has a velocity 𝑣⃑(𝑥, 𝑦, 𝑧). The water in the pipe may be said to be in a velocity field. There
are several examples of vector field in electromagnetism, e.g., the electric field 𝐸⃑⃑ , the magnetic flux density
⃑⃑ etc.
𝐵
A vector field in ℝ3 expressed in terms of its component function 𝐹1 , 𝐹2 and 𝐹3 as
𝐹⃑ = 𝐹1 (𝑥, 𝑦, 𝑧)𝐢 + 𝐹2 (𝑥, 𝑦, 𝑧)𝐣 + 𝐹3 (𝑥, 𝑦, 𝑧)𝐤.
A vector field in ℝ2 expressed in terms of its component function 𝐹1 and 𝐹2 as
𝐹⃑ = 𝐹1 (𝑥, 𝑦)𝐢 + 𝐹2 (𝑥, 𝑦)𝐣.

Level Curves and Surfaces


Level Curves
For a function 𝑧 = 𝑓(𝑥, 𝑦), the curves defined by 𝑓(𝑥, 𝑦) = 𝑐, for suitable 𝑐, are called the level
curves of 𝑓. The word level arises from the fact that we can interpret the equation 𝑓 (𝑥, 𝑦) = 𝑐 as the
projection onto the 𝑥𝑦-plane of the curve of intersection, or trace, of 𝑧 = 𝑓 (𝑥, 𝑦) and the (horizontal or
level) plane 𝑧 = 𝑐.
Suppose 𝑇(𝑥, 𝑦) = 1 − 𝑥 2 − 𝑦 2 gives the temperature at any point (𝑥, 𝑦) in a region of the 𝑥𝑦-plane.
Then a level curve or contour of 𝑇 is a curve in the 𝑥𝑦-plane such that at every point on that curve 𝑇 has the
same temperature. This is to say a level curve is the intersection of the surface 𝑧 = 𝑓(𝑥, 𝑦) and the
horizontal plane 𝑧 = 𝑘 projected into the 𝑥𝑦-plane.

Example 28 Graph 𝒇(𝒙 , 𝒚) = 𝟏𝟎𝟎 − 𝒙𝟐 − 𝒚𝟐 and plot the level curves 𝒇(𝒙, 𝒚) = 𝟎, 𝒇(𝒙, 𝒚) = 𝟓𝟏,
and 𝒇(𝒙, 𝒚) = 𝟕𝟓 in the domain of 𝒇 in the plane.
Solution:
The level curve 𝑓(𝑥, 𝑦) = 0 is the set of points in the 𝑥𝑦-plane
at which

𝑓(𝑥, 𝑦) = 100 − 𝑥 2 − 𝑦 2 = 0, or 𝑥 2 + 𝑦 2 = 100,

which is the circle of radius 10 centered at the origin.


Similarly, the level curves 𝑓(𝑥, 𝑦) = 51 and 𝑓(𝑥, 𝑦) = 75
are the circles
𝑓(𝑥, 𝑦) = 100 − 𝑥 2 − 𝑦 2 = 51, or 𝑥 2 + 𝑦 2 = 49,
𝑓(𝑥, 𝑦) = 100 − 𝑥 2 − 𝑦 2 = 75, or 𝑥 2 + 𝑦 2 = 25.

- 21 -
Chapter 1 Vectors
Level Surfaces
Although we cannot draw a graph of a function of three variables 𝑤 = 𝐹(𝑥, 𝑦, 𝑧), we can draw the
surfaces defined by 𝐹(𝑥, 𝑦, 𝑧) = 𝑐 for suitable values of 𝑐. These surfaces are called. level surfaces. This
is an unfortunate, though standard, choice of words, since level surfaces are usually not level.

Example 29 Describe the level surface of the function 𝒇(𝒙, 𝒚, 𝒛) = √𝒙𝟐 + 𝒚𝟐 + 𝒛𝟐 .


Solution:
The value of 𝑓 is the distance from the origin to the point (𝑥, 𝑦, 𝑧).
Each level surface √𝑥 2 + 𝑦 2 + 𝑧 2 = 𝑐, 𝑐 > 0, is a sphere of radius
𝑐 centered at the origin. The opposite figure shows a cutaway view of
three of these spheres. The level surface √𝑥 2 + 𝑦 2 + 𝑧 2 = 0 consists
of the origin alone.

Gradient
If 𝜙(𝑥, 𝑦, 𝑧) is a scalar function, defined at each point (𝑥, 𝑦, 𝑧) in a certain region of space and 𝜙(𝑥, 𝑦, 𝑧)
is differentiable, the gradient of 𝜙(𝑥, 𝑦, 𝑧) is defined as

𝜕 𝜕 𝜕
grad 𝜙 = ∇𝜙(𝑥, 𝑦, 𝑧) = ( 𝐢+ 𝐣 + 𝐤) 𝜙(𝑥, 𝑦, 𝑧)
𝜕𝑥 𝜕𝑦 𝜕𝑧
𝜕𝜙(𝑥, 𝑦, 𝑧) 𝜕𝜙(𝑥, 𝑦, 𝑧) 𝜕𝜙(𝑥, 𝑦, 𝑧)
grad 𝜙 = 𝐢+ 𝐣+ 𝐤
𝜕𝑥 𝜕𝑦 𝜕𝑧
𝜕 𝜕 𝜕
where ∇= 𝐢+ 𝐣+ 𝐤 is a vector differential operator called
𝜕𝑥 𝜕𝑦 𝜕𝑧 ∇𝜙(𝑃0 )
"Nabla" or "del". If 𝜙 defines a scalar field, ∇𝜙 defines a vector field. )

Physical Significance of "grad 𝝓"

1. If 𝜙(𝑥, 𝑦, 𝑧) = 𝑐 (𝑐 being a constant) represents a level surface, then


"grad 𝜙" represents the normal vector to the surface at the point
𝑃0 (𝑥0 , 𝑦0 , 𝑧0 ).

2. If 𝜙(𝑥, 𝑦, 𝑧) represents a scalar field, then


"grad 𝜙" represents the direction at which
𝜙(𝑥, 𝑦, 𝑧) increases (decreases) most
rapidly at 𝑃0 in the direction of ∇𝜙 at 𝑃0 .

- 22 -
Chapter 1 Vectors
Example 30 If 𝒇(𝒙, 𝒚, 𝒛) = 𝒙𝟐 𝒚𝒛, find grad 𝒇 at the point (𝟏, −𝟐, 𝟏).
Solution:
𝜕𝑓 𝜕𝑓 𝜕𝑓
𝑓(𝑥, 𝑦, 𝑧) = 𝑥 2 𝑦𝑧; ∵ ∇𝑓 = 𝐢+ 𝐣+ 𝐤,
𝜕𝑥 𝜕𝑦 𝜕𝑧
𝜕 𝜕 𝜕
grad 𝑓 = (𝑥 2 𝑦𝑧)𝐢 + (𝑥 2 𝑦𝑧)𝐣 + (𝑥 2 𝑦𝑧)𝐤 = (2𝑥𝑦𝑧)𝐢 + (𝑥 2 𝑧)𝐢 + (𝑥 2 𝑦)𝐤
𝜕𝑥 𝜕𝑦 𝜕𝑧

At the point (1, −2,1), grad 𝑓 = −4𝐢 + 𝐣 − 2𝐤.


Example 31 Find the unit normal vector to the surface 𝒙𝒚 + 𝒚𝒛 + 𝒛𝒙 = 𝟑 at the point (𝟏, 𝟏, 𝟏).
Solution:
If 𝜙(𝑥, 𝑦, 𝑧) = 𝑐 is a level surface, ∇𝜙 is the normal to it. The normal vector is
𝜕𝜙 𝜕𝜙 𝜕𝜙
⃑⃑ = ∇𝜙 =
𝑁 𝐢+ 𝐣+ 𝐤
𝜕𝑥 𝜕𝑦 𝜕𝑧
𝜕 𝜕 𝜕
= (𝑥𝑦 + 𝑦𝑧 + 𝑧𝑥)𝐢 + (𝑥𝑦 + 𝑦𝑧 + 𝑧𝑥)𝐣 + (𝑥𝑦 + 𝑦𝑧 + 𝑧𝑥)𝐤
𝜕𝑥 𝜕𝑦 𝜕𝑧
= (𝑦 + 𝑧)𝐢 + (𝑥 + 𝑧)𝐣 + (𝑦 + 𝑥)𝐤
⃑⃑(1, 1, 1) = 2𝐢 + 2𝐣 + 2𝐤 and the unit normal vector to the surface is
∴ The normal vector at (1, 1, 1) is 𝑁
2𝐢 + 2𝐣 + 2𝐤 𝐢+𝐣+𝐤
̂(1, 1, 1) =
𝑁 =
√22 + 22 + 22 √3

Tangent Plane and Normal Line:


Normal Line Equation:
To write an equation of the line, we need a point and vector
determine the direction of this line. Assume the line passes through
(𝑥0 , 𝑦0 , 𝑧0 ) and the direction of this line is 𝑎⃑ = 〈𝑎1 , 𝑎2 , 𝑎3 〉, then the
equation of the line is

𝑥 = 𝑥0 + 𝑎1 𝑡, 𝑦 = 𝑦0 + 𝑎2 𝑡, 𝑧 = 𝑧0 + 𝑎3 𝑡

The equation of the line that passes through (𝑥0 , 𝑦0 , 𝑧0 ) and normal to the level surface 𝜙(𝑥, 𝑦, 𝑧) = 𝑐 is

𝑥 = 𝑥0 + 𝜙𝑥 (𝑥0 , 𝑦0 , 𝑧0 ) 𝑡, 𝑦 = 𝑦0 + 𝜙𝑦 (𝑥0 , 𝑦0 , 𝑧0 ) 𝑡, 𝑧 = 𝑧0 + 𝜙𝑧 (𝑥0 , 𝑦0 , 𝑧0 ) 𝑡

where the direction vector of the normal vector at (𝑥0 , 𝑦0 , 𝑧0 ) is grad𝜙(𝑥0 , 𝑦0 , 𝑧0 ) or


𝑁⃑⃑ = 𝑎⃑ = 𝜙𝑥 (𝑥0 , 𝑦0 , 𝑧0 ) 𝐢 + 𝜙𝑦 (𝑥0 , 𝑦0 , 𝑧0 )𝐣 + 𝜙𝑧 (𝑥0 , 𝑦0 , 𝑧0 )𝐤

Tangent Plane Equation:

To write an equation of the plane, we need a point and normal vector to the plane. Assume the plane
⃑⃑ = 〈𝑛1 , 𝑛2 , 𝑛3 〉, the equation of the plane is
contains the point (𝑥0 , 𝑦0 , 𝑧0 ) and the normal vector is 𝑁

𝑛1 (𝑥 − 𝑥0 ) + 𝑛2 (𝑦 − 𝑦0 ) + 𝑛3 (𝑧 − 𝑧0 ) = 0
The equation of the line that is tangent to the level surface 𝜙(𝑥, 𝑦, 𝑧) = 𝑐 at the point (𝑥0 , 𝑦0 , 𝑧0 ) is

(𝜙𝑥 (𝑥0 , 𝑦0 , 𝑧0 ))(𝑥 − 𝑥0 ) + (𝜙𝑦 (𝑥0 , 𝑦0 , 𝑧0 )) (𝑦 − 𝑦0 ) + (𝜙𝑧 (𝑥0 , 𝑦0 , 𝑧0 ))(𝑧 − 𝑧0 ) = 0

- 23 -
Chapter 1 Vectors
𝟐 𝟐
Example 32 Find the tangent plane and normal line to the surface 𝒛 = 𝒙 + 𝒚 at the point (𝟐, 𝟏, 𝟓).
Solution:
If 𝜙(𝑥, 𝑦, 𝑧) = 𝑐 is a level surface, ∇𝜙 is the normal to it. The equation of the surface is 𝜙 = 𝑧 − 𝑥 2 − 𝑦 2 ,
the normal vector is
𝜕𝜙 𝜕𝜙 𝜕𝜙
⃑⃑ = ∇𝜙 =
𝑁 𝐢+ 𝐣+ 𝐤 = −2𝑥𝐢 − 2𝑦𝐣 + 𝐤
𝜕𝑥 𝜕𝑦 𝜕𝑧
The normal vector at 𝑃(2, 1, 5) is 𝑁(2, 1, 5) = −4𝐢 − 2𝐣 + 𝐤. The direction vector 𝑎⃑ = −4𝐢 − 2𝐣 + 𝐤 the
initial point 𝑃(2, 1, 5). The equation of the normal line in parametric form is
𝑥 = 2 − 4𝑡, 𝑦 = 1 − 2𝑡, 𝑧 = 5 + 𝑡
To find the equation of the tangent plane, we need a point 𝑃(2, 1, 5) and the normal vector 𝑛⃑⃑
−4(𝑥 − 2) − 2(𝑦 − 1) + (𝑧 − 5) = 0 ⇒ −4𝑥 − 2𝑦 + 𝑧 + 5 = 0
The equation of the tangent plane 𝜋 is 4𝑥 + 2𝑦 − 𝑧 − 5 = 0.
𝒙𝟐
Example 33 Find the tangent plane and normal line to the surface 𝒛 = 𝒙𝟑 + 𝒚𝟑 + at the point
𝒚
(𝟐, 𝟏, 𝟏𝟑).
Solution:
If 𝜙(𝑥, 𝑦, 𝑧) = 𝑐 is a level surface, ∇𝜙 is the normal to it. The equation of the surface is 𝜙 = 𝑧 − 𝑥 3 − 𝑦 3 −
𝑥2
, the normal vector is
𝑦
𝜕𝜙 𝜕𝜙 𝜕𝜙 2𝑥 𝑥2
⃑⃑ = ∇𝜙 =
𝑁 𝐢+ 𝐣+ 2 3
𝐤 = (−3𝑥 − ) 𝐢 + (−3𝑦 + 2 ) 𝐣 + 𝐤
𝜕𝑥 𝜕𝑦 𝜕𝑧 𝑦 𝑦
The normal vector at 𝑃(2, 1, 13) is 𝑁(2, 1, 13) = −16𝐢 + 𝐣 + 𝐤. The direction vector 𝑎⃑ = −16𝐢 + 𝐣 + 𝐤
the initial point 𝑃(2, 1, 13). The equation of the normal line in parametric form is
𝑥 = 2 − 16𝑡, 𝑦 = 1 + 𝑡, 𝑧 = 13 + 𝑡
To find the equation of the tangent plane, we need a point 𝑃(2, 1, 13) and the normal vector 𝑛⃑⃑
−16(𝑥 − 2) + (𝑦 − 1) + (𝑧 − 13) = 0 ⇒ −16𝑥 + 𝑦 + 𝑧 + 18 = 0
The equation of the tangent plane 𝜋 is 16𝑥 + 𝑦 + 𝑧 − 18 = 0.

Directional Derivatives

f ( x0 , y0 , z0 )  ( x0 , y0 , z0 )
If 𝜙 = 𝑓(𝑥, 𝑦, 𝑧) is a scalar function of u
𝑥, 𝑦 and 𝑧, then the rate of change of 𝜙 = z ||  ( x0 , y0 , z0 ) || cos 
𝑓(𝑥, 𝑦, 𝑧) at the point (𝑥0 , 𝑦0 , 𝑧0 ) is 
∇𝑓(𝑥0 , 𝑦0 , 𝑧0 ). u
The directional derivative is the rate 
of change of 𝜙 = 𝑓(𝑥, 𝑦, 𝑧) in the
direction of the vector 𝑢 ⃑⃑ at the point y
𝑃(𝑥0 , 𝑦0 , 𝑧0 ). p( x0 , y0 , z0 )

The directional derivative can be   f ( x, y , z )


calculated as the component of
∇𝑓(𝑥0 , 𝑦0 , 𝑧0 ) in the direction of vector 𝑢
⃑⃑. x
So, we can write the directional
derivative in the form
∇𝑓(𝑥0 , 𝑦0 , 𝑧0 ) ⋅ 𝑢
⃑⃑
𝐷𝑢⃑⃑ 𝑓(𝑥0 , 𝑦0 , 𝑧0 ) = ‖∇𝑓(𝑥0 , 𝑦0 , 𝑧0 )‖ cos 𝜃 = ‖∇𝑓(𝑥0 , 𝑦0 , 𝑧0 )‖
‖∇𝑓(𝑥0 , 𝑦0 , 𝑧0 )‖ ‖𝑢⃑⃑‖
∇𝑓(𝑥0 , 𝑦0 , 𝑧0 ) ⋅ 𝑢
⃑⃑ 𝑢
⃑⃑
= = ∇𝑓(𝑥0 , 𝑦0 , 𝑧0 ) ⋅ = ∇𝑓(𝑥0 , 𝑦0 , 𝑧0 ) ⋅ 𝑢̂
‖𝑢⃑⃑‖ ‖𝑢⃑⃑‖
- 24 -
Chapter 1 Vectors

Properties of the Gradient

1. If ∇𝑓(𝑥0 , 𝑦0 , 𝑧0 ) = ⃑0⃑, then 𝐷𝑢⃑⃑ 𝑓(𝑥0 , 𝑦0 , 𝑧0 ) = 0 for all 𝑢


⃑⃑.
2. The direction of maximum increase of 𝑓 is given by ∇𝑓(𝑥0 , 𝑦0 , 𝑧0 ). The maximum value of
𝐷𝑢⃑⃑ 𝑓(𝑥0 , 𝑦0 , 𝑧0 ) = ‖∇𝑓(𝑥0 , 𝑦0 , 𝑧0 )‖.
3. The direction of minimum increase of 𝑓 is given by −∇𝑓(𝑥0 , 𝑦0 , 𝑧0 ). The minimum value of
𝐷𝑢⃑⃑ 𝑓(𝑥0 , 𝑦0 , 𝑧0 ) = −‖∇𝑓(𝑥0 , 𝑦0 , 𝑧0 )‖.

Example 34 Find the directional derivative of 𝒇 = 𝟐𝒆𝟐𝒙−𝒚+𝒛 at (𝟏, 𝟑, 𝟏) in a direction towards the
point (𝟐, 𝟏, 𝟑), then find the maximum value of the directional derivative and its
direction.
Solution:
𝑓 = 2𝑒 2𝑥−𝑦+𝑧 , ∇𝑓 = 𝑒 2𝑥−𝑦+𝑧 (4𝐢 − 2𝐣 + 2𝐤)
∇𝑓(1, 3, 1) = 𝑒 2−3+1 (4𝐢 − 2𝐣 + 2𝐤) = 4𝐢 − 2𝐣 + 2𝐤
𝐢 − 2𝐣 + 2𝐤 𝐢 − 2𝐣 + 2𝐤
Let 𝐴 = (1, 3, 1) and 𝐵 = (2, 1, 3), 𝑢 ⃑⃑⃑⃑⃑⃑ = 𝐢 − 2𝐣 + 2𝐤, 𝒖
⃑⃑ = 𝐴𝐵 ̂= =
√1 + 4 + 4 3

𝐢 − 2𝐣 + 2𝐤 (4 + 4 + 4)
𝐷𝑢⃑⃑ 𝑓(1, 3, 1) = ∇𝑓(1, 3, 1) ⋅ 𝑢̂ = (4𝐢 − 2𝐣 + 2𝐤) ⋅ ( )= =4
3 3
Maximum value of directional derivative = ‖∇𝑓(1, 3, 1)‖ = √16 + 4 + 4 = √24.
The direction of maximum increase of 𝑓 is ∇𝑓(1, 3, 1) = 4𝐢 − 2𝐣 + 2𝐤.

Example 35 Given the function 𝒇 = 𝒙𝒚𝟐 − 𝟒𝒙𝟐 𝒚 + 𝒛𝟐 , point 𝑷(𝟏, −𝟏, 𝟐) and vector 𝒖
⃑⃑ = 𝟔𝐢 + 𝟐𝐣 +
𝟑𝐤. Find
⃑⃑.
(i) The rate of change of the function 𝒇 at 𝑷 in the direction of the vector 𝒖
(ii) The minimum value of the directional derivative and its direction.
Solution:
𝑓 = 𝑥𝑦 2 − 4𝑥 2 𝑦 + 𝑧 2 , ∇𝑓 = (𝑦 2 − 8𝑥𝑦)𝐢 + (2𝑥𝑦 − 4𝑥 2 )𝐣 + 2𝑧𝐤
∇𝑓(1, −1, 2) = 9 𝐢 − 6𝐣 + 4𝐤
6𝐢 + 2𝐣 + 3𝐤 6𝐢 + 2𝐣 + 3𝐤
𝑢
⃑⃑ = 6𝐢 + 2𝐣 + 3𝐤, 𝑢̂ = =
√36 + 4 + 9 7
6𝐢 + 2𝐣 + 3𝐤 (54 − 12 + 12) 54
𝐷𝑢⃑⃑ 𝑓(1, −1, 2) = ∇𝑓(1, −1, 2) ⋅ 𝑢̂ = (9 𝐢 − 6𝐣 + 4𝐤) ⋅ ( )= =
7 7 7
Minimum value of directional derivative = −‖∇𝑓(1, −1, 2)‖ = −√81 + 36 + 16 = −√133.
The direction of minimum increase of 𝑓 is −∇𝑓(1, −1, 2) = −9 𝐢 + 6𝐣 − 4𝐤.

The Divergence of a Vector Function

If 𝐹⃑ = 𝐹1 𝐢 + 𝐹2 𝐣 + 𝐹3 𝐤 is a vector function, defined and differentiable at each point (𝑥, 𝑦, 𝑧) in a certain


region of space (i.e., 𝐹⃑ defines a vector field), then the divergence of 𝐹⃑ (abbreviated as 'div 𝐹⃑ ') is defined
as,
𝜕 𝜕 𝜕 𝜕𝐹1 𝜕𝐹2 𝜕𝐹3
div 𝐹⃑ = ∇ 𝐹⃑ = ( 𝐢 + 𝐣 + 𝐤) ⋅ (𝐹1 𝐢 + 𝐹2 𝐣 + 𝐹3 𝐤) = + +
𝜕𝑥 𝜕𝑦 𝜕𝑧 𝜕𝑥 𝜕𝑦 𝜕𝑧

- 25 -
Chapter 1 Vectors
Physical significance of the divergence "div ⃑𝑭⃑" :

1. Divergence is an operation on a vector field that tells us


how the field behaves toward or away from a point. If 𝐹⃑
represents the velocity of fluid in a fluid flow, then div 𝐹⃑
at 𝑃 represents the rate of fluid flow through unit volume
with respect to time of the amount of fluid flowing away
from 𝑃. In particular, if the amount of fluid flowing
into 𝑃 is the same as the amount flowing out, then the
divergence at 𝑃 is zero. i.e. If div ⃑𝑭⃑ = 𝟎, then the fluid
is said to be incompressible.

2. If 𝐹⃑ represents the Electric flux or heat flux, div 𝐹⃑


represents the amount of electric flux or heat flux that
⃑⃑ = 𝟎, then
diverges per unit volume in unit time. If div 𝑭
𝐹⃑ is called solenoidal vector field.

Example 36 If ⃑𝑭⃑ = (𝒙𝟐 𝒚)𝐢 + (𝒙𝒚𝟐 𝒛)𝐣 + (𝒙𝒚𝒛)𝐤, find div ⃑𝑭⃑ at the point (𝟏, −𝟏, 𝟐).
Solution:

𝐹⃑ = (𝑥 2 𝑦)𝐢 + (𝑥𝑦 2 𝑧)𝐣 + (𝑥𝑦𝑧)𝐤


𝜕𝐹1 𝜕𝐹2 𝜕𝐹3 𝜕 𝜕 𝜕
div 𝐹⃑ = + + = (𝑥 2 𝑦) + (𝑥𝑦 2 𝑧) + (𝑥𝑦𝑧) = 2𝑥𝑦 + 2𝑥𝑦𝑧 + 𝑥𝑦 = 𝑥𝑦(2𝑧 + 3)
𝜕𝑥 𝜕𝑦 𝜕𝑧 𝜕𝑥 𝜕𝑦 𝜕𝑧

At (1, −1, 2), div 𝐹⃑ = (1)(−1)(4 + 3) = −7.

Example 37 If 𝑭
⃑⃑ = (𝟐𝒙𝒚)𝐢 + (𝟑𝒙𝟐 𝒚)𝐣 − (𝟑𝒑𝒚𝒛)𝐤 is solenoidal at the point (𝟏, 𝟏, 𝟏), find 𝒑.
Solution:
𝐹⃑ = (2𝑥𝑦)𝐢 + (3𝑥 2 𝑦)𝐣 − (3𝑝𝑦𝑧)𝐤, div 𝐹⃑ = 0 (solenoidal)
𝜕𝐹 𝜕𝐹 𝜕𝐹 𝜕 𝜕 𝜕
div 𝐹⃑ = 1 + 2 + 3 = (2𝑥𝑦) + (3𝑥 2 𝑦) − (3𝑝𝑦𝑧) = 2𝑦 + 3𝑥 2 − 3𝑝𝑦
𝜕𝑥 𝜕𝑦 𝜕𝑧 𝜕𝑥 𝜕𝑦 𝜕𝑧
5
At (1, 1, 1), div 𝐹⃑ = 5 − 3𝑝 = 0 ⇒ 𝑝 = .
3

Example 38 Is ⃑𝑭⃑ = (𝟑𝒚𝟒 𝒛𝟐 )𝐢 + (𝟒𝒙𝟑 𝒛𝟐 )𝐣 + (𝟔𝒙𝟐 𝒚𝟑 )𝐤 incompressible?


Solution:

𝜕𝐹1 𝜕𝐹2 𝜕𝐹3 𝜕 𝜕 𝜕


div 𝐹⃑ = + + = (3𝑦 4 𝑧 2 ) + (4𝑥 3 𝑧 2 ) + (6𝑥 2 𝑦 3 ) = 0.
𝜕𝑥 𝜕𝑦 𝜕𝑧 𝜕𝑥 𝜕𝑦 𝜕𝑧
Yes, 𝐹⃑ is an incompressible vector field (solenoidal).

- 26 -
Chapter 1 Vectors

Curl of a vector function

If 𝐹⃑ is a differential vector function, then curl 𝐹⃑ is defined as,


curl 𝐹⃑ = ∇ × 𝐹⃑ . If 𝐹⃑ = 𝐹1 𝐢 + 𝐹2 𝐣 + 𝐹3 𝐤, then
𝐢 𝐣 𝐤
𝜕 𝜕 𝜕 𝜕𝐹3 𝜕𝐹2 𝜕𝐹3 𝜕𝐹1 𝜕𝐹2 𝜕𝐹1
curl 𝐹⃑ = ∇ × 𝐹⃑ = | | =( − )𝐢 − ( − )𝐣 + ( − )𝐤
𝜕𝑥 𝜕𝑦 𝜕𝑧 𝜕𝑦 𝜕𝑧 𝜕𝑥 𝜕𝑧 𝜕𝑥 𝜕𝑦
𝐹1 𝐹2 𝐹3

Physical significance of curl

The second operation on a vector field that we examine is the curl,


which measures the extent of rotation of the field about a point. Let 𝑟⃑ =
𝑥 𝐢 + 𝑦 𝐣 + 𝑧 𝐤 be the position vector of a point 𝑃(𝑥, 𝑦, 𝑧) of a solid disk
rotating about 𝑧 axis with a nonzero constant angular velocity 𝜔 ⃑⃑ = 𝜔 𝐤
as shown in the opposite figure. Then the velocity ⃑⃑⃑⃑⃑
𝑉𝑃 of the particle 𝑃 is
given by,

𝐢 𝐣 𝐤
⃑⃑⃑⃑⃑
𝑉𝑃 = 𝜔⃑⃑ × 𝑟⃑ = |0 0 𝜔| = (−𝑦𝜔)𝐢 − (−𝑥𝜔)𝐣 + (0)𝐤
𝑥 𝑦 𝑧

𝐢 𝐣 𝐤
𝜕 𝜕 𝜕
curl ⃑⃑⃑⃑⃑
𝑉𝑃 = ∇ × ⃑⃑⃑⃑⃑
𝑉𝑃 = | | = (0)𝐢 − (0)𝐣 + (𝜔 + 𝜔)𝐤 = 2ω 𝐤 = 2ω
⃑⃑⃑
𝜕𝑥 𝜕𝑦 𝜕𝑧
−𝑦𝜔 𝑥𝜔 0

Thus the curl of velocity vector is twice the angular velocity of rotation.

⃑⃑ whose curl is zero is said to be an irrotational vector field.


Irrotational Vector: A vector field 𝑉
⃑⃑ = ∇ × 𝑉
curl 𝑉 ⃑⃑
⃑⃑ = 0

Suppose that 𝐹⃑ represents the velocity field of a fluid. Then, the


curl of 𝐹⃑ at point 𝑃 is a vector that measures the tendency of particles
near 𝑃 to rotate about the axis that points in the direction of this
vector.
The magnitude of the curl vector at 𝑃 measures how quickly the
particles rotate around this axis. In other words, the curl at a point is a
measure of the vector field’s “spin” at that point. Visually, in the
opposite figure, imagine placing a paddlewheel into a fluid at 𝑃, with
the axis of the paddlewheel aligned with the curl vector.

- 27 -
Chapter 1 Vectors

Important results

For any scalar field 𝜙(𝑥, 𝑦, 𝑧) and any vector field 𝐹⃑ (𝑥, 𝑦, 𝑧) we have
(i) curl(grad 𝜙(𝑥, 𝑦, 𝑧)) = ∇ × (∇𝜙(𝑥, 𝑦, 𝑧)) = ⃑⃑
0.
(ii) div(curl 𝐹⃑ ) = ∇ ⋅ (∇ × 𝐹⃑ ) = 0.

Example 39 If 𝑭
⃑⃑ = (𝒙𝒚)𝐢 + (𝒚𝒛)𝐣 + (𝒛𝒙)𝐤; find
⃑⃑ at (𝟏, 𝟐, −𝟑).
(i) curl 𝑭 ⃑⃑) 𝐚𝐭 (𝟏, 𝟐, −𝟑).
(ii) 𝐜𝐮𝐫𝐥 (𝐜𝐮𝐫𝐥 𝑭
Solution:
𝐢 𝐣 𝐤
𝜕 𝜕 𝜕
(i) curl 𝐹⃑ = ∇ × 𝐹⃑ = | | = (0 − 𝑦)𝐢 − (𝑧 − 0)𝐣 + (0 − 𝑥)𝐤 = −𝑦𝐢 − 𝑧𝐣 − 𝑥𝐤.
𝜕𝑥 𝜕𝑦 𝜕𝑧
𝑥𝑦 𝑦𝑧 𝑧𝑥

curl 𝐹 (1,2, −3) = −2𝐢 + 𝟑𝐣 − 𝐤.
𝐢 𝐣 𝐤
𝜕 𝜕 𝜕
(ii) curl (curl 𝐹⃑ ) = ∇ × (∇ × 𝐹⃑ ) = | | = (0 + 1)𝐢 − (−1 − 0)𝐣 + (0 + 1)𝐤 = 𝐢 + 𝐣 + 𝐤.
𝜕𝑥 𝜕𝑦 𝜕𝑧
−𝑦 −𝑧 −𝑥
curl (curl 𝐹⃑ (1,2, −3)) = 𝐢 + 𝐣 + 𝐤.
Example 40 Show that 𝑭
⃑⃑ = 𝒙𝐢 + 𝒚𝟐 𝐣 + 𝒛𝟑 𝐤 is irrotational
Solution:
𝐢 𝐣 𝐤
𝜕 𝜕 𝜕
curl 𝐹⃑ = ∇ × 𝐹⃑ = |𝜕𝑥 𝜕𝑦 𝜕𝑧
| = (0 − 0)𝐢 − (0 − 0)𝐣 + (0 − 0)𝐤 = ⃑0⃑.
2 3
𝑥 𝑦 𝑧
∴ 𝐹⃑ is irrotational.

Example 41 Find the constants 𝒂, 𝒃 and 𝒄 such that the vector field ⃑𝑭⃑ = (𝟒𝒙 + 𝟑𝒚 + 𝒂𝒛)𝐢 + (𝒃𝒙 −
𝒚 + 𝒛)𝐣 + (𝟐𝒙 + 𝒄𝒚 + 𝒛)𝐤 is irrotational.
Solution:
𝐢 𝐣 𝐤
𝜕 𝜕 𝜕
curl 𝐹⃑ = ∇ × 𝐹⃑ = | 𝜕𝑥 𝜕𝑦 𝜕𝑧
|
4𝑥 + 3𝑦 + 𝑎𝑧 𝑏𝑥 − 𝑦 + 𝑧 2𝑥 + 𝑐𝑦 + 𝑧
= (𝑐 − 1)𝐢 − (2 − 𝑎)𝐣 + (𝑏 − 3)𝐤 = 0𝐢 − 0𝐣 + 0𝐤.

𝑐 − 1 = 0 ⇒ 𝑐 = 1, − (2 − 𝑎) = 0 ⇒ 𝑎 = 2, 𝑏−3=0⇒𝑏 =3

Sheet 3

1. Consider the following functions 𝑓, points 𝑃, and unit vectors 𝑢


⃑⃑.
a) Compute the gradient of 𝑓 and evaluate it at 𝑃.
b) Find the unit vector in the direction of maximum increase of 𝑓 at 𝑃.
c) Find the rate of change of the function in the direction of maximum increase at 𝑃.
d) Find the directional derivative at 𝑃 in the direction of the given vector.

- 28 -
Chapter 1 Vectors
1 1 1
⃑⃑ = 〈
(i) 𝑓(𝑥, 𝑦, 𝑧) = 1 + 4𝑥𝑦𝑧; 𝑃(1, −1, −1); 𝑢 , ,− 〉.
√3 √3 √3
1 1
⃑⃑ = 〈0, −
(ii) 𝑓(𝑥, 𝑦, 𝑧) = 𝑥𝑦 + 𝑦𝑧 + 𝑥𝑧 + 4; 𝑃(2, −2, 1) ; 𝑢 ,− 〉.
√2 √2
𝜋 𝜋 𝜋 1 2 2
⃑⃑ = 〈 , , 〉.
(iii)𝑓(𝑥, 𝑦, 𝑧) = 1 + sin(𝑥 + 2𝑦 − 𝑧); 𝑃 ( , , − ) ; 𝑢
6 6 6 3 3 3
𝑥𝑦𝑧−1 2 2 1
(iv) 𝑓(𝑥, 𝑦, 𝑧) = 𝑒 ⃑⃑ = 〈− , , − 〉.
; 𝑃(0,1, −1); 𝑢
3 3 3
3 2 𝑥
(v) 𝑓 (𝑥, 𝑦, 𝑧) = 𝑥 𝑦𝑧 − tan−1 ( ) ; ⃑⃑ = 〈2, 0, −1〉.
𝑃(1, −1, 2); 𝑢
𝑦
2. Find an equation of the tangent plane and normal vector to the following surfaces at the given point
(i) 𝑥 2 + 𝑦 2 + 𝑧 2 = 3; 𝑃(1, 1, 1).
(ii) 𝑥𝑦𝑧 = 8; 𝑃(2, 2, 2).
(iii) 𝑒 𝑥+𝑦−𝑧 = 1; 𝑃(1, 1, 2).
(iv) 𝑥𝑦 + 𝑥𝑧 − 𝑦𝑧 = 1; 𝑃(1, 1, 1).
(v) 𝑧 = √𝑥 2 + 𝑦 2 ; 𝑃(3, −4, 5).
3. Find the directions of maximum and minimum change of 𝑓 at the given point, and the values of the
maximum and minimum rates of change.
(i) 𝑓(𝑥, 𝑦, 𝑧) = 4𝑥 2 𝑦𝑧 3 ; 𝑃(1, 2, 1).
(ii) 𝑓(𝑥, 𝑦, 𝑧) = √𝑥 2 + 𝑦 2 + 𝑧 2 ; 𝑃(1, 2, −2).
4. If the temperature at the point (𝑥, 𝑦, 𝑧) is given by 𝑇 (𝑥, 𝑦, 𝑧) = 80 + 5𝑒 −𝑧 (𝑥 −2 + 𝑦 −1 ), find the
direction from the point (1, 4, 8) in which the temperature decreases most rapidly.
5. If ⃑⃑⃑⃑
𝑉 = 2𝑥𝑧 2 𝑖 − 𝑦𝑧𝑗 + 3𝑥𝑧 3 𝑘, and 𝑓 = 𝑥 2 𝑦𝑧, find the following at the point (1, 1, 1)
⃑⃑
(i) curl 𝑉 (ii) curl (grad 𝑓) ⃑⃑ )
(iii) div (𝑓𝑉 ⃑⃑ )
(iv) div (curl 𝑉 (v) div (grad 𝑓).
1
6. Let = , prove that ∇ ∙ (∇𝑓) = 0.
√𝑥 2 +𝑦 2 +𝑧 2
4 1 4 𝑧 𝑧
⃑⃑ = √ 𝐢 + ( √𝑦 3 +
7. Show that the vector field 𝑢 ) 𝐣 + (2√𝑦 + 𝑧 + + √𝑧) 𝐤 is irrotational.
𝑥 √𝑦+𝑧 √𝑦+𝑧
8. Show that any vector field of the form 𝐹(𝑥, 𝑦, 𝑧) = 𝑓(𝑦, 𝑧) 𝐢 + 𝑔(𝑥, 𝑧) 𝐣 + ℎ(𝑥, 𝑦) 𝐤 is
incompressible.
9. Determine whether the following vector fields is solenoidal at the given point
(i) 𝐹(𝑥, 𝑦, 𝑧) = 〈𝑥𝑦 2 , 2𝑥𝑧, 4 − 𝑧 2 𝑦〉; 𝑃(1, 1, 2)
〈𝑥,𝑦,𝑧〉
(ii) 𝐹(𝑥, 𝑦, 𝑧) = ; 𝑃(1, 1, 1)
1+𝑥 2 +𝑦 2
𝜋 𝜋 𝜋
(iii)𝐹 = 〈𝑦𝑧 sin 𝑥 , 𝑥𝑧 cos 𝑦 , 𝑥𝑦 cos 𝑧〉; 𝑃 ( , , )
2 2 2

Homework 3

1. Find the gradient vector field of the 𝑓


(a) 𝑓(𝑥, 𝑦, 𝑧) = 𝑥 5 − 4𝑥 3 𝑧 4 + 𝑥𝑦 6 + 𝑧 sin 𝑦
(b) 𝑓(𝑥, 𝑦, 𝑧) = 𝑒 𝑥𝑧 cos 4𝑦 + sin 𝑥𝑦𝑧
2. Find the direction at which 𝑓(𝑥, 𝑦, 𝑧) increases most rapidly and find its direction at the given point
𝜋 𝜋
(a) 𝑓(𝑥, 𝑦, 𝑧) = 𝑥 cos(3𝑥 + 2𝑦) + 2𝑧 2 , ( , − , −1)
6 8
(b) 𝑓(𝑥, 𝑦, 𝑧) = 𝑥𝑦𝑧, (−1, 2, 5)

- 29 -
Chapter 1 Vectors
3. Find the equation of the tangent plane and the normal line to the given surface at the specified point
(a) 4𝑥 2 + 𝑦 2 + 𝑧 2 = 24 , (2,2,2)
(b) 𝑧 + 1 = 𝑥 𝑒 𝑦 cos 𝑧 , (1, 0, 0)
4. Find the directional derivative of the given functions at the given point in the direction of the vector 𝑢
⃑⃑
(a) 𝑓(𝑥, 𝑦, 𝑧) = √𝑥𝑦𝑧, (2,4,2), ⃑⃑ = 〈4, 2 , −4〉
𝑢
𝑦
(b) 𝑓(𝑥, 𝑦, 𝑧) = 𝑥 tan−1 ( ), (−2, 1, 1), ⃑⃑ = 〈1, −2 ,3〉
𝑢
𝑥

5. Find the constants 𝑝 and 𝑞 such that the normal vectors of the two surfaces 𝑝𝑥 2 − 𝑞𝑦𝑧 = (𝑝 + 2) 𝑥
and 4𝑥 2 𝑦 + 𝑧 3 = 4 are orthogonal at the point (1, −1, 2).

6. Compute div 𝐹⃑ and curl 𝐹⃑ for the following vector field


1. 𝐹⃑ (𝑥, 𝑦, 𝑧) = 𝑥 2 𝑦 i − (𝑧 3 − 3𝑥) j + 4𝑦 2 k
2. 𝐹⃑ (𝑥, 𝑦, 𝑧) = 𝑒 𝑥𝑦 i − cos 𝑦 j + sin2 𝑧 k
𝑥 3 𝑦2
3. 𝐹⃑ (𝑥, 𝑦, 𝑧) = (3𝑥 + 2𝑧 2 )i + ( ) j − (𝑧 − 7𝑥) k
𝑧

7. Find ∇ ∙ (∇ × 𝐹⃑ )
1. 𝐹⃑ (𝑥, 𝑦, 𝑧) = sin 𝑥 i + cos(𝑥 − 𝑦) j + 𝑧 k 2. 𝐹⃑ (𝑥, 𝑦, 𝑧) = 𝑒 𝑥𝑧 i + 3𝑥𝑒 𝑦 j − 𝑒 𝑦𝑧 k

8. Find ∇ × (∇ × ⃑F⃑)
1. 𝐹⃑ (𝑥, 𝑦, 𝑧) = 𝑥𝑦 j + 𝑥𝑦𝑧 k 2. 𝐹⃑ (𝑥, 𝑦, 𝑧) = 𝑦 2 𝑥 i − 3𝑦𝑧 j + 𝑥𝑦 k

Challenging Problems
1. Sharks find their prey through a keen sense of smell and an ability to detect small electrical impulses. If
𝑓(𝑥, 𝑦, 𝑧) indicates the electrical charge in the water at position (𝑥, 𝑦, 𝑧) and a shark senses that 𝛻 𝑓 =
〈12, −20, 5〉 , in which direction should the shark swim to find its prey?
6
2. Consider a function 𝑓(𝑥, 𝑦) at the point (2, 6). At that point the function has directional derivatives:
√45
2
in the direction (parallel to) 〈3, 6〉 and in the direction (parallel to) 〈2,7〉. What is the gradient of 𝑓
√53
at the point (2,6)?
3. For the given contour map draw the curves of steepest ascent starting at 𝑃 and at 𝑄.

4. At what point on the surface 𝑦 = 𝑥 2 + 𝑧 2 is the tangent plane parallel to the plane 𝑥 + 2𝑦 + 3𝑧 = 1?
⃑⃑ on ℝ3 such that curl 𝑢
5. Is there a vector field 𝑢 ⃑⃑ = 𝑦𝑧 𝐢 + 𝑥𝑦𝑧 𝐣 + 𝑥𝑦 𝐤? Explain.
6. If the scalar field 𝜙(𝑥, 𝑦, 𝑧), is a harmonic field (𝜙𝑥𝑥 + 𝜙𝑦𝑦 + 𝜙𝑧𝑧 = 0) and the vector field 𝐹⃑ is the
gradient of 𝜙(𝑥, 𝑦, 𝑧), show that 𝐹⃑ is an incompressible and irrotational vector field.
7. For any two scalar field 𝑓(𝑥, 𝑦, 𝑧) and 𝑔(𝑥, 𝑦, 𝑧), show that div(∇𝑓 × ∇𝑔) = 0.

- 30 -
Chapter 2 Line Integral

Chapter 2

Line Integrals

Introduction

In this chapter we define an integral that is similar to a single integral except that instead of integrating
over an interval, we integrate over a curve. Such integrals are called line integrals, although “curve integrals”
would be better terminology. They were invented in the early 19th century to solve problems involving fluid
flow, forces, electricity, and magnetism
Line integrals are used to find the work done by a force in moving an object along a path, find the mass
of a curved wire with variable density and find the arc length of a curve. We need to integrate over a curve
𝐶 rather than over an interval [𝑎, 𝑏].

Line Integral
Line Integral of a Scalar Function
Suppose that 𝑓(𝑥, 𝑦) is a real-valued function we wish to integrate
over the curve 𝐶 lying within the domain of 𝑓 and parametrized by
𝑟⃑ = 𝑔(𝑡) 𝐢 + ℎ(𝑡) 𝐣, 𝑎 ≤ 𝑡 ≤ 𝑏.
The values of 𝑓 along the curve are given by the composite function
𝑓(𝑔(𝑡), ℎ(𝑡)). We are going to integrate this composite with respect to
arc length from 𝑡 = 𝑎 to 𝑡 = 𝑏.
To begin, we first partition the curve 𝐶 into a finite number 𝑛 of sub-
arcs. We define the line integral of a scalar function 𝑓(𝑥, 𝑦) along 𝐶 by:
𝑡2 =𝑏
∫ 𝑓(𝑥, 𝑦)𝑑𝑠 = ∫ 𝑓(𝑥, 𝑦) √(𝑑𝑥)2 + (𝑑𝑦)2
𝐶 𝑡1 =𝑎

Cases:
(1) If 𝑪 is represented in the Cartesian form 𝒚 = 𝒈(𝒙), 𝒙𝟏 ≤ 𝒙 ≤ 𝒙𝟐 then
𝑥2
𝑑𝑔 2
∫ 𝑓(𝑥, 𝑦)𝑑𝑠 = ∫ 𝑓(𝑥, 𝑔(𝑥))√1 + ( ) 𝑑𝑥
𝐶 𝑥1 𝑑𝑥
- 31 -
Chapter 2 Line Integral
(2) If 𝑪 is represented in the Cartesian form 𝒙 = 𝒈(𝒚), 𝒚𝟏 ≤ 𝒚 ≤ 𝒚𝟐 then

𝑦2
𝑑𝑔 2
∫ 𝑓(𝑥, 𝑦)𝑑𝑠 = ∫ 𝑓(𝑔(𝑦), 𝑦)√( ) + 1 𝑑𝑦
𝐶 𝑦1 𝑑𝑦

(3) If C is a curve in the 𝒙𝒚-plane with 𝒓


⃑⃑(𝒕) = 𝒈(𝒕)𝐢 + 𝒉(𝒕)𝐣, 𝒕𝟏 ≤ 𝒕 ≤ 𝒕𝟐 then

𝑡2 =𝑏
𝑑𝑔 2 𝑑ℎ 2
∫ 𝑓(𝑥, 𝑦)𝑑𝑠 = ∫ √
𝑓(𝑔(𝑡), ℎ(𝑡)) ( ) + ( ) 𝑑𝑡
𝐶 𝑡1 =𝑎 𝑑𝑡 𝑑𝑡

z
By generalizing the line integral ∫𝑪 𝒇(𝒙, 𝒚)𝒅𝒔 in 3-space, we get
sk
t
b ta
𝑡2 =𝑏 2 2 2
𝑑𝑥 𝑑𝑦 𝑑𝑧 ( xk , yk , zk )
∫ 𝑓(𝑥, 𝑦, 𝑧)𝑑𝑠 = ∫ 𝑓(𝑥, 𝑦, 𝑧) √( ) + ( ) + ( ) 𝑑𝑡
𝐶 𝑡1 =𝑎 𝑑𝑡 𝑑𝑡 𝑑𝑡 
r (t )
𝑡2 =𝑏
y
=∫ 𝑓(𝑥, 𝑦, 𝑧) ‖𝑟⃑′(𝑡)‖ 𝑑𝑡
𝑡1 =𝑎
x

Notice that:
(1) If 𝑓(𝑥, 𝑦, 𝑧) represents the linear density (mass per unit length) of certain wire then ∫𝐶 𝑓(𝑥, 𝑦, 𝑧)𝑑𝑠
represents the total mass of the wire.

(2) If 𝑓(𝑥, 𝑦, 𝑧) = 1, we get ∫𝐶 𝑑𝑠 represents the arc length of a part of the curve 𝐶.

(3) To compute a line integral ∫𝐶 𝑓(𝑥, 𝑦, 𝑧)𝑑𝑠 when the curve 𝐶 piecewise smooth C1 C2
as shown in figure, that is made of smooth pieces. In this case

∫ 𝑓(𝑥, 𝑦, 𝑧)𝑑𝑠 = ∫ 𝑓(𝑥, 𝑦, 𝑧)𝑑𝑠 + ∫ 𝑓(𝑥, 𝑦, 𝑧)𝑑𝑠 + ∫ 𝑓(𝑥, 𝑦, 𝑧)𝑑𝑠 C3


𝐶 𝐶1 𝐶2 𝐶3

+ ∫ 𝑓(𝑥, 𝑦, 𝑧)𝑑𝑠 C4
𝐶4

Example 1 Evaluate ∫𝑪 𝟑𝒙 𝒅𝒔 along 𝑪 is the curve 𝒚 = 𝒙𝟐 from 𝑨(𝟎, 𝟎) to 𝑩(𝟑, 𝟗).


Solution:
𝐶: 𝑦 = 𝑥 2 assume 𝑥 = 𝑡, 𝑦 = 𝑡 2 , 0 ≤ 𝑡 ≤ 3,
𝑟⃑(𝑡) = 𝑥(𝑡) 𝐢 + 𝑦(𝑡) 𝐣 = 𝑡 𝐢 + 𝑡 2 𝐣, 𝑟⃑ ′ (t) = 𝐢 + 2𝑡 𝐣,

‖𝑟⃑ ′ (𝑡)‖ = √1 + 4𝑡 2 , 𝑓(𝑡) = 3𝑡

3 3
3 3 3 2 3 3
∫ 3𝑥 𝑑𝑠 = ∫ 3𝑡‖𝑟⃑ ′ (𝑡)‖𝑑𝑡 = ∫ 3𝑡√1 + 4𝑡 2 𝑑𝑡 = ∫ 8𝑡 √1 + 4𝑡 2 𝑑𝑡 = ⋅ [(1 + 4𝑡 2 )2 ]
𝐶 0 0 8 0 8 3 0

1
= ((37)3/2 − 1).
4

- 32 -
Chapter 2 Line Integral

Example 2 Evaluate ∫𝑪 𝒚 𝒅𝒔 along 𝑪 is the upper half of the circle 𝒙𝟐 + 𝒚𝟐 = 𝟒 from 𝑨(𝟐, 𝟎) to
𝑩(−𝟐, 𝟎).
Solution:
𝐶: 𝑥 2 + 𝑦 2 = 4 the parametric form: 𝑥 = 2 cos 𝑡 , 𝑦 = 2 sin 𝑡 , 0 ≤ 𝑡 ≤ 𝜋,
𝑟⃑(𝑡) = 𝑥(𝑡)𝐢 + 𝑦(𝑡)𝐣 = 2 cos 𝑡 𝐢 + 2 sin 𝑡 𝐣, 𝑟⃑ ′ (𝑡) = −2 sin 𝑡 𝐢 + 2 cos 𝑡 𝐣,
‖𝑟⃑ ′ (𝑡)‖ = √4sin2 𝑡 + 4 cos2 𝑡 = 2, 𝑓(𝑡) = 2 sin 𝑡

𝜋 𝜋 𝜋
∫ 𝑦 𝑑𝑠 = ∫ 2 sin 𝑡 ‖𝑟⃑ ′ (𝑡)‖𝑑𝑡 = ∫ 4 sin 𝑡 𝑑𝑡 = 4 ∫ sin 𝑡 𝑑𝑡 = 4[−cos 𝑡]𝜋0 = 4(1 + 1) = 8
𝐶 0 0 0

Example 3 Find the mass of a thin wire shaped in the form of a helix with parametric equation
⃑⃑ = 𝐜𝐨𝐬 𝒕 𝐢 + 𝐬𝐢𝐧 𝒕 𝐣 + 𝟐𝒕 𝐤,
𝒓 𝟎≤𝒕≤𝝅
if the density function 𝝆(𝒙, 𝒚, 𝒛) = 𝒂𝒛.
Solution:
𝐶: 𝑥 = cos 𝑡 , 𝑦 = sin 𝑡 , 𝑧 = 2𝑡, 0 ≤ 𝑡 ≤ 𝜋,
𝑟⃑(𝑡) = 𝑥(𝑡)𝐢 + 𝑦(𝑡)𝐣 + 𝑧(𝑡)𝐤 = cos 𝑡 𝐢 + sin 𝑡 𝐣 + 2𝑡 𝐤, 𝑟⃑ ′ (𝑡) = − sin 𝑡 𝐢 + cos 𝑡 𝐣 + 2𝐤,

‖𝑟⃑ ′ (𝑡)‖ = √sin2 𝑡 + cos2 𝑡 + 4 = √5,


𝜋

mass = ∫ 𝜌(𝑥, 𝑦, 𝑧)𝑑𝑠 = ∫ 𝑎𝑧 𝑑𝑠 = 𝑎 ∫ 2𝑡√5 𝑑𝑡 = 𝑎√5[𝑡 2 ]𝜋0 = 𝑎𝜋 2 √5


𝐶 𝐶 0

Example 4 Find the length of the part of the curve 𝒚 = 𝐜𝐨𝐬𝐡 𝒙 from 𝒙 = 𝟎 to 𝒙 = 𝟐.
Solution:
𝐶: 𝑦 = cosh 𝑥, assume 𝑥 = 𝑡, 𝑦 = cosh 𝑡, 0 ≤ 𝑡 ≤ 2,
𝑟⃑(𝑡) = 𝑥(𝑡)𝐢 + 𝑦(𝑡)𝐣 = 𝑡 𝐢 + cosh 𝑡 𝐣, 𝑟⃑ ′ (𝑡) = 𝐢 + sinh 𝑡 𝐣,
‖𝑟⃑ ′ (𝑡)‖ = √1 + sinh2 𝑡 = √cosh2 𝑡 = cosh 𝑡,
2

Arc lenght = 𝐿 = ∫ 1 𝑑𝑠 = ∫ cosh 𝑡 𝑑𝑡 = [sinh 𝑡]20 = sinh 2.


𝐶 0

Another Solution
𝐶: 𝑦 = cosh 𝑥 , 0 ≤ 𝑥 ≤ 2, 𝑦′ = sinh 𝑥

𝑥 𝑑𝑦 2 2 2
Arc lenght = ∫𝑥 2 √1 + (𝑑𝑥 ) 𝑑𝑥 = ∫0 √1 + (sinh 𝑥)2 𝑑𝑥 = ∫0 √cosh2 𝑥 𝑑𝑥
1

2
= ∫0 cosh 𝑥 𝑑𝑥 = [sinh 𝑥]20 = sinh 2.

- 33 -
Chapter 2 Line Integral

z 
Line Integral of a Vector Function F
T
Assume that the vector field t b ta
𝐹⃑ = 𝐹1 (𝑥, 𝑦, 𝑧)𝐢 + 𝐹2 (𝑥, 𝑦, 𝑧) 𝐣 + 𝐹3 (𝑥, 𝑦, 𝑧) 𝐤 
 Component of F in the
has continuous components defined, and that the curve 𝐶 has a smooth r (t ) direction of motion
 
F cos   F  T
parametrization
O y
𝑟⃑ = 𝑔(𝑡) 𝐢 + ℎ(𝑡) 𝐣 + 𝑘(𝑡) 𝐤, 𝑎 ≤ 𝑡 ≤ 𝑏, x

The parametrization 𝑟⃑(𝑡) defines a direction (or orientation) along 𝐶 which we call the forward
direction. At each point along the path 𝐶, the tangent vector 𝐓 is a unit vector tangent to the path and pointing
in this forward direction. Then the line integral of 𝐹⃑ along 𝐶 is

⃑⃑⃑⃑
𝑟 ′ (𝑡) 𝑑𝑟⃑
∫ 𝐹⃑ ⋅ 𝐓 𝑑𝑠 = ∫ (𝐹⃑ ⋅ ) ‖𝑟⃑′(𝑡)‖𝑑𝑡 = ∫ (𝐹⃑ ⋅ ) 𝑑𝑡 = ∫ 𝐹⃑ ⋅ 𝑑𝑟⃑
𝐶 𝐶 ‖𝑟⃑′(𝑡)‖ 𝐶 𝑑𝑡 𝐶

where

𝑑𝑟⃑ = 𝑑𝑥 𝐢 + 𝑑𝑦 𝐣 + 𝑑𝑧 𝐤

Different Forms of Line Integrals of Vector Fields

1. 𝐼 = ∫ 𝐹⃑ ⋅ 𝐓 𝑑𝑠 (The definition)
𝐶
𝑑𝑟⃑
2. 𝐼 = ∫𝐶 (𝐹⃑ (𝑟⃑(𝑡)) ⋅ 𝑑𝑡 ) 𝑑𝑡 (Parametric differential form)

3. 𝐼 = ∫ 𝐹⃑ ⋅ 𝑑𝑟⃑ (Vector differential form)


𝐶

4. 𝐼 = ∫ 𝐹1 (𝑥, 𝑦, 𝑧)𝑑𝑥 + 𝐹2 (𝑥, 𝑦, 𝑧)𝑑𝑦 + 𝐹3 (𝑥, 𝑦, 𝑧)𝑑𝑧 (Scalar differential form)


𝐶

Physical Applications:
The work done by a variable force
If 𝐹⃑ (𝑥, 𝑦, 𝑧) represents the force vector which moves a particle along a curve C then ∫𝐶 𝐹⃑ ⋅ 𝑑𝑟⃑
represents the work done. i.e.

𝑊 = ∫ 𝐹⃑ ⋅ 𝑑𝑟⃑
𝐶
If 𝐹⃑ (𝑥, 𝑦, 𝑧) is the electric field strength, the line integral ∫𝐶 𝐹⃑ ⋅ 𝑑𝑟⃑, is called the flow of A along 𝐶.

Circulation
A line integral of a vector field 𝐹⃑ (𝑥, 𝑦, 𝑧) around a simple closed curve 𝐶 is said to be circulation of
𝐹⃑ (𝑥, 𝑦, 𝑧) around 𝐶 that is,

∮ 𝐹⃑ ⋅ 𝑑𝑟⃑
𝐶
If 𝐹⃑ (𝑥, 𝑦, 𝑧) is the velocity field of a fluid, then the circulation is a measure of the amount by which the fluid
tends to turn the curve 𝐶 by rotating, or circulation around it.
- 34 -
Chapter 2 Line Integral

Example 5 If 𝑭⃑⃑ = (𝒙𝟐 − 𝟐𝒚)𝐢 − 𝟔𝒚𝒛 𝐣 + 𝟖𝒙𝒛𝟐 𝐤, evaluate ∫ 𝑭 ⃑⃑ ⋅ 𝒅𝒓


⃑⃑ from the point (𝟎, 𝟎, 𝟎) to the
𝑪
point (𝟏, 𝟏, 𝟏) along the following paths
(a) 𝒙 = 𝒕, 𝒚 = 𝒕𝟐 , 𝒛 = 𝒕𝟑 .
(b) the straight line from (𝟎, 𝟎, 𝟎) to (𝟏 , 𝟎, 𝟎), then to (𝟏, 𝟏 , 𝟎) and then to (𝟏, 𝟏, 𝟏).
(c) the straight line joining (𝟎, 𝟎, 𝟎) to (𝟏, 𝟏, 𝟏).
Solution:
𝑑𝑟⃑
(a) 𝑥 = 𝑡, 𝑦 = 𝑡 2 , 𝑧 = 𝑡 3 , 𝑟⃑ = 𝑥𝐢 + 𝑦 𝐣 + 𝑧 𝐤 = 𝑡 𝐢 + 𝑡 2 𝐣 + 𝑡 3 𝐤, = 𝐢 + 2𝑡 𝐣 + 3𝑡 2 𝐤
𝑑𝑡
𝐹⃑ (𝑟⃑(𝑡)) = (𝑡 2 − 2𝑡 2 )𝐢 − 6𝑡 5 𝐣 + 8𝑡 7 𝐤
1 1
𝑑𝑟⃑
∫ 𝐹⃑ ⋅ 𝑑𝑟⃑ = ∫ 𝐹⃑ ⋅ 𝑑𝑡 = ∫ ((𝑡 2 − 2𝑡 2 )𝐢 − 6𝑡 5 𝐣 + 8𝑡 7 𝐤) ⋅ (𝐢 + 2𝑡 𝐣 + 3𝑡 2 𝐤)𝑑𝑡
𝐶 0 𝑑𝑡 0
1 1
2 6 9 )𝑑𝑡
𝑡 3 12 24 10 1 12 12 37
= ∫ (− 𝑡 − 12𝑡 + 24𝑡 = − − 𝑡7 + 𝑡 | =− − + =
0 3 7 10 0
3 7 5 105
(b) the straight line from (0, 0, 0) to (1 , 0, 0), then to (1, 1 ,0) and then to (1, 1, 1).

Let 𝑂 = (0,0,0), 𝑃 = (1,0,0), 𝑄 = (1, 1,0), 𝑅 = (1, 1,1).


̅̅̅̅, 𝑦 = 0, 𝑧 = 0, and 𝑥 varies from 0 to 1. Assume 𝑥 = 𝑡, then
Then along 𝑂𝑃
𝑑𝑟⃑
𝑟⃑ = 𝑥𝐢 + 𝑦 𝐣 + 𝑧 𝐤 = 𝑡 𝐢 + 0 𝐣 + 0𝐤, = 𝐢
𝑑𝑡
𝐹⃑ (𝑟⃑(𝑡)) = (𝑡 2 − 2(0))𝐢 − 6(0)𝐣 + 8(0)𝐤 = 𝑡 2 𝐢
1 1 1 1
𝑑𝑟⃑ 𝑡3 1
𝐼1 = ∫ 𝐹⃑ ⋅ 𝑑𝑟⃑ = ∫ 𝐹⃑ ⋅ 𝑑𝑡 = ∫ (𝑡 2 𝐢) ⋅ (𝐢)𝑑𝑡 = ∫ ( 𝑡 2 )𝑑𝑡 = | =
𝐶1 0 𝑑𝑡 0 0 3 0 3
̅̅̅̅, 𝑥 = 1, 𝑧 = 0, and 𝑦 varies from 0 to 1. Assume 𝑦 = 𝑡, then
Along 𝑃𝑄
𝑑𝑟⃑
𝑟⃑ = 𝑥 𝐢 + 𝑦 𝐣 + 𝑧 𝐤 = 𝐢 + 𝑡 𝐣 + 0𝐤, = 𝐣
𝑑𝑡
𝐹⃑ (𝑟⃑(𝑡)) = (1 − 2(𝑡))𝐢 − 6(0)𝐣 + 8(0)𝐤 = (1 − 2(𝑡))𝐢
1 1
𝑑𝑟⃑
𝐼2 = ∫ 𝐹⃑ ⋅ 𝑑𝑟⃑ = ∫ 𝐹⃑ ⋅ 𝑑𝑡 = ∫ (1 − 2(𝑡))𝐢 ⋅ (𝐣)𝑑𝑡 = 0
𝐶2 0 𝑑𝑡 0
̅̅̅̅, 𝑥 = 1, 𝑦 = 1, and 𝑧 varies from 0 to 1. Assume 𝑧 = 𝑡, then
Along 𝑄𝑅
𝑑𝑟⃑
𝑟⃑ = 𝑥𝐢 + 𝑦 𝐣 + 𝑧 𝐤 = 𝐢 + 𝐣 + 𝑡 𝐤, = 𝐤
𝑑𝑡
𝐹⃑ (𝑟⃑(𝑡)) = −𝐢 − 6𝑡𝐣 + 8𝑡 2 𝐤
1 1 1 1
𝑑𝑟⃑ 8𝑡 3 8
𝐼3 = ∫ 𝐹⃑ ⋅ 𝑑𝑟⃑ = ∫ 𝐹⃑ ⋅ 𝑑𝑡 = ∫ (−𝐢 − 6𝑡𝐣 + 8𝑡 2 𝐤) ⋅ (𝐤)𝑑𝑡 = ∫ (8𝑡 2 )𝑑𝑡 = | =
𝐶3 0 𝑑𝑡 0 0 3 0 3
1 8
𝐼 = ∫ 𝐹⃑ ⋅ 𝑑𝑟⃑ = 𝐼1 + 𝐼2 + 𝐼3 = + 0 + = 3.
𝐶 3 3
(c) The equation of the straight line joining (0, 0, 0) to (1, 1, 1) is
𝑥 = 𝑡, 𝑦 = 𝑡, 𝑧 = 𝑡
𝑑𝑟⃑
𝑟⃑ = 𝑥 𝐢 + 𝑦 𝐣 + 𝑧 𝐤 = 𝑡 𝐢 + 𝑡 𝐣 + 𝑡 𝐤, = 𝐢+𝐣+𝐤
𝑑𝑡
𝐹⃑ (𝑟⃑(𝑡)) = (𝑡 2 − 2(𝑡))𝐢 − 6(𝑡 2 )𝐣 + 8(𝑡 3 )𝐤

1 1
𝑑𝑟⃑
𝐼 = ∫ 𝐹⃑ ⋅ 𝑑𝑟⃑ = ∫ 𝐹⃑ ⋅ 𝑑𝑡 = ∫ ((𝑡 2 − 2(𝑡))𝐢 − 6(𝑡 2 )𝐣 + 8(𝑡 3 )𝐤) ⋅ ( 𝐢 + 𝐣 + 𝐤)𝑑𝑡
𝐶 0 𝑑𝑡 0
1 1 1
2 2 3 )𝑑𝑡 2 3 )𝑑𝑡
5𝑡 3 8𝑡 4
2
2
= ∫ ( 𝑡 − 2𝑡 − 6𝑡 + 8𝑡 = ∫ ( −2𝑡 − 5𝑡 + 8𝑡 = −𝑡 − + | =−
0 0 3 4 0 3

- 35 -
Chapter 2 Line Integral
𝟐
Example 6 Evaluate ∫𝑪 (𝒙 + 𝟐𝒚)𝒅𝒙 − 𝒙𝒚𝒅𝒚 along the path 𝑪: 𝒚 = 𝟑𝒙 from the point 𝑨(𝟎, 𝟎) to the
point 𝑩(𝟏, 𝟑).
Solution:
𝑑𝑟⃑
𝐶: 𝑦 = 3𝑥 2 , assume 𝑥 = 𝑡, then 𝑦 = 3𝑡 2 , 𝑟⃑ = 𝑥𝐢 + 𝑦 𝐣 + 𝑧 𝐤 = 𝑡 𝐢 + 3𝑡 2 𝐣, = 𝐢 + 6𝑡 𝐣
𝑑𝑡

∫ (𝑥 + 2𝑦)𝑑𝑥 − 𝑥𝑦𝑑𝑦 = ∫ ((𝑥 + 2𝑦)𝐢 − 𝑥𝑦𝐣) ⋅ (𝑑𝑥𝐢 + 𝑑𝑦𝐣) = ∫ ((𝑥 + 2𝑦)𝐢 − 𝑥𝑦𝐣) ⋅ 𝑑𝑟⃑
𝐶 𝐶 𝐶
𝑑𝑟⃑
= ∫ ((𝑡 + 6𝑡 2 )𝐢 − 3𝑡 3 𝐣) ⋅ 𝑑𝑡 = ∫ ((𝑡 + 6𝑡 2 )𝐢 − 3𝑡 3 𝐣) ⋅ (𝐢 + 6𝑡 𝐣)𝑑𝑡
𝐶 𝑑𝑡 𝐶
1
𝑡2 18 1 18 11
∴ ∫ (𝑥 + 2𝑦)𝑑𝑥 − 𝑥𝑦𝑑𝑦 = ∫ (𝑡 + 6𝑡 − 18𝑡 ) 𝑑𝑡 = + 2𝑡 3 − 𝑡 5 | = + 2 −
2 4
=−
𝐶 𝐶 2 5 0
2 5 10

Another Solution
𝑑𝑦
𝐶: 𝑦 = 3𝑥 2 , 𝑑𝑥 = 6𝑥 ⇒ 𝑑𝑦 = 6𝑥𝑑𝑥 and 𝑥 varies from 0 to 1.
1 1
11
∫ (𝑥 + 2𝑦)𝑑𝑥 − 𝑥𝑦𝑑𝑦 = ∫ (𝑥 + 2(3𝑥 2 ))𝑑𝑥 − 𝑥(3𝑥 2 )6𝑥𝑑𝑥 = ∫ (𝑥 + 6𝑥 2 − 18𝑥 4 )𝑑𝑥 = −
𝐶 0 0 10

Example 7 Find the work done by a force ⃑𝑭⃑ = 𝟐𝒙𝒚 𝐢 – 𝟒𝒛 𝐣 + 𝟓𝒙 𝐤 along the curve 𝒙 = 𝒕𝟐 ,
𝒚 = 𝟐𝒕 + 𝟏, 𝒛 = 𝒕𝟑 , from the points 𝒕 = 𝟏 to 𝒕 = 𝟐.
Solution:
𝑑𝑟⃑
𝑥 = 𝑡 2 , 𝑦 = 2𝑡 + 1, 𝑧 = 𝑡 3 , 𝑟⃑ = 𝑥𝐢 + 𝑦 𝐣 + 𝑧 𝐤 = 𝑡 2 𝐢 + (2𝑡 + 1) 𝐣 + 𝑡 3 𝐤, = 2𝑡 𝐢 + 2 𝐣 + 3𝑡 2 𝐤
𝑑𝑡
𝐹⃑ (𝑟⃑(𝑡)) = (4𝑡 3 + 2t 2 ) 𝐢 − 4𝑡 3 𝐣 + 5𝑡 2 𝐤
2 2
𝑑𝑟⃑
𝑊 = ∫ 𝐹⃑ ⋅ 𝑑𝑟⃑ = ∫ 𝐹⃑ ⋅ 𝑑𝑡 = ∫ ((4𝑡 3 + 2t 2 ) 𝐢 − 4𝑡 3 𝐣 + 5𝑡 2 𝐤) ⋅ (2𝑡 𝐢 + 2 𝐣 + 3𝑡 2 𝐤)𝑑𝑡
𝐶 1 𝑑𝑡 1
2 2 2
23 5 638
= ∫ (8𝑡 4 + 4𝑡 3 − 8𝑡 3 + 15𝑡 4 )𝑑𝑡 = ∫ (23𝑡 4 − 4𝑡 3 )𝑑𝑡 = 𝑡 − 𝑡4| =
1 1 5 1 5

Example 8 Find the work done by a force 𝑭 ⃑⃑ = (𝒙 + 𝒚 + 𝒛)𝐢 + (𝟐𝒙 + 𝒚)𝐣 + (𝟐𝒙 − 𝒚 + 𝒛)𝐤 in
moving a particle once around the circle 𝑪 in 𝒙𝒚 - plane, if the center of the circle is the
origin and has radius equal 2.
Solution:
𝐶: 𝑥 2 + 𝑦 2 = 4, 𝑥 = 2 cos 𝑡 , 𝑦 = 2 sin 𝑡 , 𝑧 = 0, 0 ≤ 𝑡 ≤ 2𝜋
𝑑𝑟⃑
𝑟⃑ = 𝑥𝐢 + 𝑦 𝐣 + 𝑧 𝐤 = 2 cos 𝑡 𝐢 + 2 sin 𝑡 𝐣 + 0 𝐤, = −2 sin 𝑡 𝐢 + 2 cos 𝑡 𝐣 + 0 𝐤
𝑑𝑡
𝐹⃑ (𝑟⃑(𝑡)) = (2 cos 𝑡 + 2 sin 𝑡 + 0) 𝐢 + (4 cos 𝑡 + 2 sin 𝑡) 𝐣 + (4 cos 𝑡 − 2 sin 𝑡 + 0)𝐤

2𝜋
𝑑𝑟⃑
𝑊 = ∫ 𝐹⃑ ⋅ 𝑑𝑟⃑ = ∫ 𝐹⃑ ⋅ 𝑑𝑡
𝐶 0 𝑑𝑡
2𝜋
= ∫ ((2 cos 𝑡 + 2 sin 𝑡 + 0) 𝐢 + (4 cos 𝑡 + 2 sin 𝑡) 𝐣 + (4 cos 𝑡 − 2 sin 𝑡 + 0)𝐤)
0
⋅ (−2 sin 𝑡 𝐢 + 2 cos 𝑡 𝐣)𝑑𝑡
2𝜋 𝜋
= ∫ (−4 cos 𝑡 sin 𝑡 − 4 sin2 𝑡 + 8 cos 2 𝑡 + 4 sin 𝑡 cos 𝑡)𝑑𝑡 = ∫ (−4 sin2 𝑡 + 8 cos 2 𝑡)𝑑𝑡
0 0
2𝜋 2𝜋
= ∫ (−2(1 − cos 2𝑡) + 4(1 + cos 2𝑡))𝑑𝑡 = ∫ (2 + 6 cos 2𝑡)𝑑𝑡 = 2𝑡 + 3 sin 2𝑡|2𝜋
0 = 4𝜋
0 0

- 36 -
Chapter 2 Line Integral

⃑⃑ = (𝒙 − 𝒚)𝐢 + 𝒙 𝐣 around the circle


Example 9 Find the circulation of the field 𝑭
⃑⃑(𝒕) = 𝐜𝐨𝐬 𝒕 𝐢 + 𝐬𝐢𝐧 𝒕 𝐣,
𝒓 𝟎 ≤ 𝒕 ≤ 𝟐𝝅.
Solution:
𝐶: 𝑥 = cos 𝑡 , 𝑦 = sin 𝑡 , 𝑧 = 0, 0 ≤ 𝑡 ≤ 2𝜋
𝑑𝑟⃑
𝑟⃑ = 𝑥𝐢 + 𝑦 𝐣 + 𝑧 𝐤 = cos 𝑡 𝐢 + sin 𝑡 𝐣 + 0 𝐤, 𝑑𝑡 = − sin 𝑡 𝐢 + cos 𝑡 𝐣 + 0 𝐤
𝐹⃑ (𝑟⃑(𝑡)) = (cos 𝑡 − sin 𝑡) 𝐢 + (cos 𝑡)𝐣 + (0)𝐤

2𝜋 2𝜋
𝑑𝑟⃑
∮ 𝐹⃑ ⋅ 𝑑𝑟⃑ = ∮ 𝐹⃑ ⋅
𝑑𝑡 = ∮ ((cos 𝑡 − sin 𝑡) 𝐢 + (cos 𝑡)𝐣 + (0)𝐤) ⋅ (− sin 𝑡 𝐢 + cos 𝑡 𝐣 + 0 𝐤)𝑑𝑡
𝐶 0 𝑑𝑡 0
2𝜋 2𝜋 2𝜋
1
= ∮ (− cos 𝑡 sin 𝑡 + sin2 𝑡 + cos 2 𝑡)𝑑𝑡 = ∮ (− cos 𝑡 sin 𝑡 + 1)𝑑𝑡 = ( cos 2 𝑡 + 𝑡)
0 0 2 0
= 2𝜋

Conservative Vector Field


The vector field 𝐹⃑ (𝑥, 𝑦, 𝑧) = 𝑓(𝑥, 𝑦, 𝑧)𝐢 + 𝑔(𝑥, 𝑦, 𝑧) 𝐣 + ℎ(𝑥, 𝑦, 𝑧) 𝐤 is a conservative vector field if
there is exist a potential function 𝜙(𝑥, 𝑦, 𝑧) such that 𝐹⃑ = ∇𝜙 or curl 𝐹⃑ = 0

Path Independence

If the value of a line integral is the same for every path in a region connecting the initial point 𝐴 and
terminal point 𝐵, then the integral is said to be independent of the path. In other words, a line integral
∫ 𝐹⃑ ⋅ 𝑑𝑟⃑ of 𝐹⃑ along 𝐶 is independent of the path if ∫ 𝐹⃑ ⋅ 𝑑𝑟⃑ = ∫ 𝐹⃑ ⋅ 𝑑𝑟⃑ for any two paths 𝐶1 and 𝐶2
𝐶 𝐶1 𝐶2
between 𝐴 and 𝐵.

Fundamental Theorem for Conservative Vector Fields


Assume that 𝐹⃑ is conservative vector field or 𝐹⃑ = ∇𝜙 on a domain 𝐷.
(1) If 𝐶 is any smooth curve from 𝐴(𝑥1 , 𝑦1 , 𝑧1 ) to 𝐵(𝑥2 , 𝑦2 , 𝑧2 ) in 𝐷, then

∫ 𝐹⃑ ⋅ 𝑑𝑟⃑ = 𝜙(𝐵) − 𝜙(𝐴).


𝐶
Proof:
𝑡2 𝑡2
𝜕𝜙 𝜕𝜙 𝜕𝜙 𝑑𝑥 𝑑𝑦 𝑑𝑧
∫ 𝐹⃑ ⋅ 𝑑𝑟⃑ = ∫ ∇𝜙 ⋅ 𝑟⃑′ 𝑑𝑡 = ∫ ( 𝐢+ 𝐣+ 𝐤) ⋅ ( 𝐢+ 𝐣+ 𝐤) 𝑑𝑡
𝐶 𝑡1 𝑡1 𝜕𝑥 𝜕𝑦 𝜕𝑧 𝑑𝑡 𝑑𝑡 𝑑𝑡
𝑡2 𝑡2 𝑡2
𝜕𝜙 𝑑𝑥 𝜕𝜙 𝑑𝑦 𝜕𝜙 𝑑𝑧 𝑑𝜙
=∫ + + 𝑑𝑡 = ∫ 𝑑𝑡 = 𝜙(𝑥(𝑡), 𝑦(𝑡), 𝑧(𝑡)) |
𝑡1 𝜕𝑥 𝑑𝑡 𝜕𝑦 𝑑𝑡 𝜕𝑧 𝑑𝑡 𝑡1 𝑑𝑡 𝑡1

= 𝜙(𝑥(𝑡2 ), 𝑦(𝑡2 ), 𝑧(𝑡2 )) − 𝜙(𝑥(𝑡1 ), 𝑦(𝑡1 ), 𝑧(𝑡1 )) = 𝜙(𝐵) − 𝜙(𝐴),


𝑑𝜙 𝜕𝜙 𝑑𝑥 𝜕𝜙 𝑑𝑦 𝜕𝜙 𝑑𝑧
where = + 𝜕𝑦 + (chain rule).
𝑑𝑡 𝜕𝑥 𝑑𝑡 𝑑𝑡 𝜕𝑧 𝑑𝑡

(2) The circulation around a closed path 𝐶 (that is 𝐴 = 𝐵) is zero,

∮ 𝐹⃑ ⋅ 𝑑𝑟⃑ = 0.
𝐶
- 37 -
Chapter 2 Line Integral

How to Get the Scalar Potential 𝝓


𝜕𝜙 𝜕𝜙 𝜕𝜙
If 𝐹⃑ = ∇𝜙, then 𝐹⃑ = 〈𝐹1 , 𝐹2 , 𝐹3 〉 = ∇𝜙 = 〈 𝜕𝑥 , 𝜕𝑦 , 𝜕𝑧 〉, then

𝜕𝜙
= 𝐹1 ⇒ 𝜙 = ∫ 𝐹1 𝑑𝑥 + ℎ1 (𝑥, 𝑦) … … … … … (1)
𝜕𝑥
𝜕𝜙
= 𝐹2 ⇒ 𝜙 = ∫ 𝐹2 𝑑𝑥 + ℎ2 (𝑥, 𝑧) … … … … … (2)
𝜕𝑦
𝜕𝜙
= 𝐹3 ⇒ 𝜙 = ∫ 𝐹3 𝑑𝑥 + ℎ3 (𝑦, 𝑧) … … … … … (3)
𝜕𝑧

Comparing (1), (2) and (3) we obtain the scalar potential 𝜙.

Example 10 Show that the following vector field is not conservative ⃑𝑭⃑ = 𝒚𝐢 + (𝒙 + 𝒙𝟐 )𝐣 + 𝟐𝒚𝒛𝐤
Solution:
𝐢 𝐣 𝐤
𝜕 𝜕 𝜕
∇ × 𝐹⃑ = || || = (2𝑧 − 0)𝐢 − (0 − 0)𝐣 + (2𝑥 + 1 − 1)𝐤 ≠ ⃑0⃑
𝜕𝑥 𝜕𝑦 𝜕𝑧
𝑦 𝑥 + 𝑥2 2𝑦𝑧
∴ 𝐹⃑ is not conservative field.
Example 11 If 𝑭
⃑⃑ = 𝒚𝟐 𝐢 + (𝟐𝒙𝒚 + 𝒛𝟐 )𝐣 + 𝟐𝒚𝒛𝐤
(a) Show that is a conservative force field.
(b) Find the work done in moving an object in this field from 𝑨(𝟏, 𝟐, 𝟏) to 𝑩(𝟑, 𝟏, 𝟒).
Solution:
(a)
𝐢 𝐣 𝐤
𝜕 𝜕 𝜕
∇ × 𝐹⃑ = || || = (2𝑧 − 2𝑧)𝐢 − (0 − 0)𝐣 + (2𝑦 − 2𝑦)𝐤 = ⃑⃑
0
𝜕𝑥 𝜕𝑦 𝜕𝑧
𝑦2 2𝑥𝑦 + 𝑧 2 2𝑦𝑧
∴ 𝐹⃑ is conservative force field and the line integral is independent of path.
𝜕𝜙 𝜕𝜙 𝜕𝜙
𝐹⃑ = 𝑦 𝟐 𝐢 + (2𝑥𝑦 + 𝑧 2 )𝐣 + 2𝑦𝑧𝐤 = ∇𝜙 = 〈 , , 〉
𝜕𝑥 𝜕𝑦 𝜕𝑧
𝜕𝜙
= 𝑦 2 ⇒ 𝜙 = ∫ 𝑦 2 𝑑𝑥 = 𝑥𝑦 2 … … … … … (1)
𝜕𝑥
𝜕𝜙
= 2𝑥𝑦 + 𝑧 2 ⇒ 𝜙 = ∫(2𝑥𝑦 + 𝑧 2 ) 𝑑𝑦 = 𝑥𝑦 2 + 𝑦𝑧 2 … … … … … (2)
𝜕𝑦
𝜕𝜙
= 2𝑦𝑧 ⇒ 𝜙 = ∫ 2𝑦𝑧 𝑑𝑧 = 𝑦𝑧 2 … … … … … (3)
𝜕𝑧
To get 𝜙(𝑥, 𝑦, 𝑧), we add all equation but the repeated term is written once. The potential function is
𝜙 = 𝑥𝑦 2 + 𝑦𝑧 2
(b)
∵ 𝐹⃑ conservative field, then the work done can be evaluated as
𝐵 (3, 1, 4)

𝑊 = ∫ 𝐹 ⋅ 𝑑𝑟⃑ = 𝜙(𝐵) − 𝜙(𝐴) = 𝑥𝑦 + 𝑦𝑧 |2 2
= (3 + 16) − (4 + 2) = 13.
𝐴 (1, 2, 1)

- 38 -
Chapter 2 Line Integral
Example 12 Show that the line integral ∫𝑩(𝒚 + 𝒚𝒛)𝒅𝒙 + (𝒙 + 𝟑𝒛𝟑 + 𝒙𝒛)𝒅𝒚 + (𝟗𝒚𝒛𝟐 + 𝒙𝒚 + 𝟏)𝒅𝒛,
𝑨
is independent of the path joining the two points 𝑨(𝟏, 𝟏, 𝟏) and 𝑩(𝟐, 𝟏, 𝟒), then evaluate
it.
Solution: 𝐹⃑ = (𝑦 + 𝑦𝑧) 𝐢 + (𝑥 + 3𝑧 3 + 𝑥𝑧) 𝐣 + (9𝑦𝑧 2 + 𝑥𝑦 + 1) 𝐤
𝐢 𝐣 𝐤
𝜕 𝜕 𝜕
∇ × 𝐹⃑ = || ||
𝜕𝑥 𝜕𝑦 𝜕𝑧
𝑦 + 𝑦𝑧 𝑥 + 3𝑧 3 + 𝑥𝑧 9𝑦𝑧 2 + 𝑥𝑦 + 1
= (9𝑧 2 + 𝑥 − (9𝑧 2 + 𝑥)) 𝐢 − (𝑦 − 𝑦) 𝐣 + (1 + 𝑧 − (1 + 𝑧)) 𝐤 = ⃑0⃑
∴ 𝐹⃑ is conservative field and the line integral is independent of path.
𝜕𝜙 𝜕𝜙 𝜕𝜙
𝐹⃑ = (𝑦 + 𝑦𝑧)𝐢 + (𝑥 + 3𝑧 3 + 𝑥𝑧)𝐣 + (9𝑦𝑧 2 + 𝑥𝑦 + 1)𝐤 = ∇𝜙 = 〈 , , 〉
𝜕𝑥 𝜕𝑦 𝜕𝑧
𝜕𝜙
= 𝑦 + 𝑦𝑧 ⇒ 𝜙 = ∫(𝑦 + 𝑦𝑧) 𝑑𝑥 = 𝑥𝑦 + 𝑥𝑦𝑧 … … … … … (1)
𝜕𝑥
𝜕𝜙
= 𝑥 + 3𝑧 3 + 𝑥𝑧 ⇒ 𝜙 = ∫(𝑥 + 3𝑧 3 + 𝑥𝑧) 𝑑𝑦 = 𝑥𝑦 + 3𝑦𝑧 3 + 𝑥𝑦𝑧 … … … … … (2)
𝜕𝑦
𝜕𝜙
= 9𝑦𝑧 2 + 𝑥𝑦 + 1 ⇒ 𝜙 = ∫(9𝑦𝑧 2 + 𝑥𝑦 + 1) 𝑑𝑧 = 3𝑦𝑧 3 + 𝑥𝑦𝑧 + 𝑧 … … … … … (3)
𝜕𝑧
To get 𝜙(𝑥, 𝑦, 𝑧), we add all equation but the repeated term is written once. The potential function is
𝜙 = 𝑥𝑦 + 𝑥𝑦𝑧 + 3𝑦𝑧 3 + 𝑧
∵ 𝐹⃑ conservative field, then the line integral can be evaluated as
𝐵 (2, 1, 4)
∫ 𝐹⃑ ⋅ 𝑑𝑟⃑ = 𝜙(𝐵) − 𝜙(𝐴) = 𝑥𝑦 + 𝑥𝑦𝑧 + 3𝑦𝑧 3 + 𝑧 | = 200
𝐴 (1, 1, 1)
𝑩(𝟏,−𝟏,𝟕)
Example 10 Show that the line integral ∫𝑨(𝟎,𝟏,𝟐) 𝟑𝒙𝟐 𝒅𝒙 + 𝟐𝒚𝒛𝒅𝒚 + 𝒚𝟐 𝒅𝒛, is independent of the path
then evaluate it.
Solution: 𝐹⃑ = (3𝑥 2 ) 𝐢 + (2𝑦𝑧) 𝐣 + (𝑦 2 ) 𝐤
𝐢 𝐣 𝐤
𝜕 𝜕 𝜕
∇ × 𝐹⃑ = || || = (2𝑦 − 2𝑦) 𝐢 − (0 − 0) 𝐣 + (0 − 0) 𝐤 = ⃑⃑
0
𝜕𝑥 𝜕𝑦 𝜕𝑧
3𝑥 2 2𝑦𝑧 𝑦 2
∴ 𝐹⃑ is conservative field and the line integral is independent of path.
𝜕𝜙 𝜕𝜙 𝜕𝜙
𝐹⃑ = 3𝑥 2 𝐢 + 2𝑦𝑧 𝐣 + 𝑦 2 𝐤 = ∇𝜙 = 〈 , , 〉
𝜕𝑥 𝜕𝑦 𝜕𝑧
𝜕𝜙
= 3𝑥 2 ⇒ 𝜙 = ∫ 3𝑥 2 𝑑𝑥 = 𝑥 3 … … … … … (1)
𝜕𝑥
𝜕𝜙
= 2𝑦𝑧 ⇒ 𝜙 = ∫ 2𝑦𝑧 𝑑𝑦 = 𝑦 2 𝑧 … … … … … (2)
𝜕𝑦
𝜕𝜙
= 𝑦 2 ⇒ 𝜙 = ∫ 𝑦 2 𝑑𝑧 = 𝑦 2 𝑧 … … … … … (3)
𝜕𝑧
To get 𝜙(𝑥, 𝑦, 𝑧), we add all equation but the repeated term is written once. The potential function is
𝜙 = 𝑦2𝑧 + 𝑥3

𝐵 (1, −1, 7)
⃑ 2 3
∫ 𝐹 ⋅ 𝑑𝑟⃑ = 𝜙(𝐵) − 𝜙(𝐴) = 𝑦 𝑧 + 𝑥 | = (7 + 1) − (2) = 6.
𝐴 (0, 1, 2)
- 39 -
Chapter 2 Line Integral

Sheet 4

1. Evaluate ∫𝑐 2𝑥 𝑑𝑠, where 𝐶 consists of the arc of the parabola 𝑦 = 𝑥 2 from (0, 0) to (1, 1) followed by
the vertical line segment from (1, 1) to (1,2).
2. Evaluate ∫𝑐 𝑦 sin 𝑧 𝑑𝑠 where 𝐶 is the circular helix given by the equation
𝑥 = cos 𝑡 , 𝑦 = sin 𝑡 , 𝑧 = 𝑡, 0 ≤ 𝑡 ≤ 2𝜋.
2𝑦

3. Evaluate ∫𝑐 (4𝑥 − 𝑒 𝑥 ) 𝑑𝑠 where 𝐶 has parametric equation
𝑥 = 𝑡, 𝑦 = 𝑡 2 , 𝑧 = 𝑒 −𝑡 , 0 ≤ 𝑡 ≤ 1.
4. Find the arc length of the curve traced out by the endpoint of the following vector-valued function
3
1
(i) 𝑟⃑(𝑡) = 〈𝑡 cos 𝑡 , 𝑡 sin 𝑡 , 3 (2𝑡)2 〉 ; 0 ≤ 𝑡 ≤ 2𝜋.
(ii) 𝑟⃑(𝑡) = 〈2𝑡, ln 𝑡 , 𝑡 2 〉; 1 ≤ 𝑡 ≤ 𝑒.
(iii) 𝑟⃑(𝑡) = 〈tan−1 𝑡 , ln(1 + 𝑡 2 ) , 2𝑡 − 2 tan−1 𝑡〉; 0 ≤ 𝑡 ≤ 𝜋.

5. Find the arc length of the portion of the curve determined by the intersection of the cone 𝑧 =
√𝑥 2 + 𝑦 2 and the plane 𝑦 + 𝑧 = 2 in the first octant.

6. A thin wire has the shape of the first-quadrant part of the circle with center the origin and radius 𝑎. If
the density function is 𝜌(𝑥, 𝑦) = 𝑥𝑦, find the mass of the wire.
7. Find the mass of a thin wire that lies along the curve 𝑟⃑(𝑡) = (𝑡 2 − 1) 𝐢 + 2𝑡 𝐤; 0 ≤ 𝑡 ≤ 1 if the density
3
function is 𝜌(𝑥, 𝑦, 𝑧) = 4 𝑧.
8. Evaluate the line integral ∫𝑐 𝑦 2 𝑑𝑥 + 𝑥 𝑑𝑦 along the following paths
(i) 𝐶 is the line segment from (−5, 3) to (0, 2).
(ii) 𝐶 is the arc of the parabola 𝑥 = 4 − 𝑦 2 from (−5, 3) to (0, 2).
9. Evaluate the line integral ∫𝑐 𝑦 𝑑𝑥 + 𝑧 𝑑𝑦 + 𝑥 𝑑𝑧 where 𝐶 consists of the line segment from (2, 0, 0) to
(3, 4, 5), followed by the vertical line segment from (3, 4, 5) to (3, 4, 0).
10. Find the work done by the force field 𝐹⃑ = 𝑥 2 𝐢 − 𝑥𝑦 𝐣 in moving a particle along the quarter-circle from
(1,0) to (0,1).
11. Compute the work done by the force field 𝐹(𝑥, 𝑦, 𝑧) = 〈4𝑦, 2𝑥𝑧, 3𝑦〉 acting on an object as it moves
along the helix defined parametrically by 𝑥 = 2 cos 𝑡 , 𝑦 = 2 sin 𝑡 and 𝑧 = 3𝑡, from the point (2, 0, 0)
to the point (−2, 0, 3𝜋).

12. Evaluate ∫𝑐 𝐹⃑ ∙ 𝑑𝑟⃑ where 𝐹(𝑥, 𝑦, 𝑧) = 𝑥𝑦 𝐢 + 𝑦𝑧 𝐣 + 𝑧𝑥 𝐤 and 𝐶 is the curve defined by 𝑥 = 𝑡, 𝑦 = 𝑡 2


and 𝑧 = 𝑡 3 for 0 ≤ 𝑡 ≤ 1.
13. Evaluate ∫𝑐 𝐹⃑ ∙ 𝑑𝑟⃑ where 𝐹(𝑥, 𝑦, 𝑧) = sin 𝑥 𝐢 + cos 𝑦 𝐣 + 𝑥𝑧 𝐤; 𝑟⃑(𝑡) = 𝑡 3 𝐢 − 𝑡 2 𝐣 + 𝑡 𝐤; 0 ≤ 𝑡 ≤ 1.
14. Show that the following integral is independent of the path. Evaluate the integral
𝐵(2,𝜋/2,1)
∫ (2𝑥 sin 𝑦 + 𝑒 3z ) 𝑑𝑥 + (𝑥 2 cos 𝑦) 𝑑𝑦 + (3𝑥 𝑒 3𝑧 ) 𝑑𝑧
𝐴(1,0,0)
15. Show that the line integral

∫(2𝑥𝑦 3 𝑧 4 ) 𝑑𝑥 + (3𝑥 2 𝑦 2 𝑧 4 ) 𝑑𝑦 + (4𝑥 2 𝑦 3 𝑧 3 ) 𝑑𝑧


𝑐
is independent of the path and evaluate the integral, where 𝐶 is a curve from (0, 1, 3) to (2, 1, −1).

- 40 -
Chapter 2 Line Integral

Homework 4:

1. Evaluate ∫𝑐 𝑦 sin 𝑧 𝑑𝑠 where 𝐶 is the helix given by the equations


𝑥 = cos 𝑡 , 𝑦 = sin 𝑡 , 𝑧 = 𝑡, 0 ≤ 𝑡 ≤ 2𝜋
2. Evaluate ∫𝑐 2𝑥 𝑑𝑠 where 𝐶 consists of the arc 𝐶1 of the parabola 𝑦 = 𝑥 2 from (0,0) to (1, 1) followed
by the vertical line segment 𝐶2 from (1,1) to (1, 2).
3. Evaluate ∫𝑐 (3𝑥 − 𝑦)𝑑𝑠 where 𝐶 that is the line segment from (1, 2) to (3,3), followed by the portion
of the circle 𝑥 2 + 𝑦 2 = 18 traversed from (3, 3) counterclockwise around to (−3, 3).
4. Evaluate ∫𝑐 (𝑥𝑦 + 𝑧 3 )𝑑𝑠 from (1, 0, 0) to (−1, 0, 𝜋) along the helix 𝐶 represented by the parametric
equations
𝑥 = cos 𝑡 , 𝑦 = sin 𝑡 , 𝑧 = 𝑡
5. Find the length of the part of the curve 𝑦 = 𝑥 3/2 from (0, 0) to (4, 8).
1
6. A thin wire has the shape of a plane curve 𝐶 and if the linear mass density is 𝜌(𝑥, 𝑦) = 3 (−16𝑥 +
16𝑦 + 20) and 𝐶: 𝑥 = 𝑡 2 , 𝑦 = 1 + 3𝑡 + 𝑡 2 , 0 ≤ 𝑡 ≤ 1.
7. Evaluate ∫𝑐 𝐹 ∙ 𝑑𝑟 where 𝐹(𝑥, 𝑦, 𝑧) = (𝑦 2 − 𝑧 2 ) 𝐢 + (2𝑦𝑧) 𝐣 − 𝑥 2 𝐤 and 𝐶 is the curve defined by 𝑥 =
𝑡 2 , 𝑦 = 2𝑡 and 𝑧 = 𝑡 for 0 ≤ 𝑡 ≤ 1.
8. Evaluate the line integral ∫𝑐 4𝑥 𝑑𝑦 + 2𝑦 𝑑𝑧 where 𝐶 is the line segment from (0, 1, 0) to (0, 1, 1)
followed by the line segment from (0, 1, 1) to (2, 1, 0) followed by the line segment from (2, 1, 1) to
(2, 4, 1).
9. Evaluate the line integral ∫𝑐 𝑥 2 𝑦 𝑑𝑥 + 𝑥 𝑑𝑦 in counterclockwise direction around the triangle whose
vertices are (0, 0), (1, 0) and (1,2).

10. Evaluate the work done by the force field 𝐹(𝑥, 𝑦, 𝑧) = 𝑦 𝐢 − 𝑥 𝐣 acting on an object as it moves along
the parabola 𝑦 = 𝑥 2 − 1 from (1, 0) to (−2, 3).
11. Evaluate the work done by the force field 𝐹(𝑥, 𝑦, 𝑧) = 4𝑦 𝐢 + (2𝑥𝑧) 𝐣 + 3𝑦 𝐤 acting on an object as it
moves along the path defined parametrically as
𝑥 = 2 cos 𝑡 , 𝑦 = 2 sin 𝑡 , 𝑧 = 3𝑡
From the point (2, 0, 0) to the point (−2, 0, 3𝜋).

12. Determine if the vector field is conservative for the following vector field
3𝑥 𝑦 2 𝑥 2𝑥 𝑦3 3
1. 𝐹⃑ (𝑥, 𝑦, 𝑧) = (4𝑦 2 + 2 ) i + (8𝑥𝑦 + 2 ) j + (11 − 3 ) k
𝑧 𝑧 𝑧
2. 𝐹⃑ (𝑥, 𝑦, 𝑧) = 6𝑥i + (2𝑦 − 𝑦 2)
j + (6𝑧 − 𝑥 3)
k

13. Show that the following line integrals are path independent, then evaluate it.
𝐵(1,1,1)
1. ∫𝐴(0,1,0) (𝑦𝑧 + 1) 𝑑𝑥 + (𝑥𝑧 + 1) 𝑑𝑦 + (𝑥𝑦 + 1) 𝑑𝑧.
𝐵(2,𝜋,1)
2. ∫𝐴(1,0,0) (2𝑥 cos 𝑦 − 2𝑧 3 ) 𝑑𝑥 + (3 + 2𝑦𝑒 𝑧 − 𝑥 2 sin 𝑦) 𝑑𝑦 + (𝑦 2 𝑒 𝑧 − 6𝑥𝑧 2 ) 𝑑𝑧.

Challenging Problems
𝑑𝑠
1. Evaluate ∫𝑐 , where 𝐶 is a segment of the straight line connecting the points 𝑂(0, 0) and
√𝑥 2 +𝑦2 +𝑧 2
𝐴(1, 2).

- 41 -
Chapter 2 Line Integral
𝑥2 𝑦2
2. Evaluate ∫𝑐 𝑥𝑦 𝑑𝑠 , where 𝐶 is a quarter of the ellipse + 𝑏2 = 1 lying in the first quadrant.
𝑎2
3. An object with mass 𝑚 moves with position function
𝜋
𝑟⃑(𝑡) = 𝑎 sin 𝑡 𝐢 + 𝑏 cos 𝑡 𝐣 + 𝑐𝑡 𝐤, 0 ≤ 𝑡 ≤ .
2
Find the work done on the object during this time period.
z
4. Show that the force field 𝐹⃑ = (𝑒 𝑦 𝑦
+ 𝑦𝑧)𝐢 + (𝑥𝑒 + 𝑥𝑧)𝐣 + 𝑥𝑦 𝐤 is A 1
a conservative field then use it to find the work done by 𝐹⃑ to move
a particle along the path 𝐴𝑂𝐵𝐶 (starting at 𝐴 and ending at 𝐶) as O 1
shown in the opposite figure. y
B
x 1 C

- 42 -
Chapter 3 Multiple Integrals

Chapter 3

Multiple Integrals
Double Integrals

Introduction

𝑏
In previous chapter, we gave the five steps leading to the definition of the definite integral ∫𝑎 𝑓(𝑥)𝑑𝑥 .
The analogous steps that lead to the concept of the two-dimensional definite integral, known simply as the
double integral of a function 𝑓 of two variables, will be given next.

1. Let the surface 𝑧 = 𝑓(𝑥, 𝑦) be defined in a closed and


bounded region 𝑅 of 2-space where 𝑓(𝑥, 𝑦) ≥ 0 and 𝑓 is
continuous on 𝑅.
2. By means of a grid of vertical and horizontal lines parallel to
the coordinate axes, form a partition 𝑃 of 𝑅 into 𝑛 rectangular
subregions 𝑅𝑘 of areas ∆𝐴𝑘 that lie entirely in 𝑅.
3. Let ‖𝑃‖ be the norm of the partition or the length of the longest
diagonal of the 𝑅𝑘 .
4. Choose a sample point (𝑥 ∗ , 𝑦 ∗ ) in each subregion 𝑅𝑘 .
5. Form the sum ∑𝑛𝑘=1 𝑓 (𝑥 ∗ , 𝑦 ∗ ) ∆𝐴𝑘 .

Definition: The double integrals


Let 𝑓 be a function of two variables defined on a closed region 𝑅 of 2-space. Then the double integral
of 𝑓 over 𝑅 is given by
𝑑 𝑛 𝑛

∬ 𝑓(𝑥, 𝑦)𝑑𝐴 = lim ∑ 𝑓 (𝑥 ∗ , 𝑦 ∗ ) ∆𝐴𝑘 = lim ∑ 𝑓 (𝑥 ∗ , 𝑦 ∗ ) ∆𝐴𝑘


||𝑝||→0 𝑛→∞
𝑅 𝑘=1 𝑘=1

- 43 -
Chapter 3 Multiple Integrals
Area
When 𝑓(𝑥, 𝑦) = 1 on 𝑅, then lim ∆𝐴𝑘 simply gives the area 𝐴 of the region; that is,
||𝑝||→0
𝑑

𝐴 = ∬ 𝑑𝐴
𝑅
Volume

If 𝑓(𝑥, 𝑦) ≥ 0 on 𝑅, then, as shown in figure, the


product𝑓(𝑥 ∗ , 𝑦 ∗ )∆𝐴𝑘 can be interpreted as the volume of a
rectangular prism of height 𝑓(𝑥 ∗ , 𝑦 ∗ ) and base of area ∆𝐴𝑘 . The
summation of volumes ∑𝑛𝑘=1 𝑓 (𝑥 ∗ , 𝑦 ∗ ) ∆𝐴𝑘 is an
approximation to the volume 𝑉 of the solid above the region 𝑅
and below the surface 𝑧 = 𝑓(𝑥, 𝑦).
The limit of this sum as ‖𝑃‖ → 0, if it exists, gives the exact
volume of this solid; that is, if 𝑓 is nonnegative on 𝑅, then

𝑉 = ∬ 𝑓(𝑥, 𝑦) 𝑑𝐴
𝑅

Properties of Double Integrals

Let 𝒇 and 𝒈 be functions of two variables that are integrable over a region 𝑹. Then

(i) ∬𝑅 𝑘𝑓(𝑥, 𝑦)𝑑𝐴 = 𝑘 ∬𝑅 𝑓(𝑥, 𝑦)𝑑𝐴, where 𝑘 is any constant.

(ii) ∬𝑅 [𝑓(𝑥, 𝑦) ± 𝑔(𝑥, 𝑦)]𝑑𝐴 = ∬𝑅 𝑓(𝑥, 𝑦)𝑑𝐴 ± ∬𝑅 𝑔(𝑥, 𝑦)𝑑𝐴

(iii)∬𝑅 𝑓(𝑥, 𝑦)𝑑𝐴 = ∬𝑅 𝑓(𝑥, 𝑦)𝑑𝐴 + ∬𝑅 𝑓(𝑥, 𝑦)𝑑𝐴, where 𝑅1 and 𝑅2 are
1 2
subregions of 𝑅 that do not overlap and 𝑅1 ∪ 𝑅2 = 𝑅

- 44 -
Chapter 3 Multiple Integrals
Evaluation of Double Integrals
Iterated integrals provide the means for evaluating a double integral ∬𝑅 𝑓(𝑥, 𝑦)𝑑𝐴 over a region 𝑅.
There are two types of regions that we need to look at. Here is a sketch of both of them.

Case I:

We first consider the case where the region 𝑅 lies between the
vertical lines 𝑥 = 𝑎 and 𝑥 = 𝑏, with 𝑎 < 𝑏, has a top defined by the
curve 𝑦 = 𝑔2 (𝑥) and a bottom defined by 𝑦 = 𝑔1 (𝑥), where 𝑔1 (𝑥) ≤
𝑔2 (𝑥) for all 𝑥 in (𝑎, 𝑏). That is, 𝑅 has the form
𝑅 = {(𝑥, 𝑦) | 𝑎 ≤ 𝑥 ≤ 𝑏 and 𝑔1 (𝑥) ≤ 𝑦 ≤ 𝑔2 (𝑥)}.
From pervious figure, observe that for each fixed 𝑥 ∈ [𝑎, 𝑏], the
double integral for both of this case are defined in terms of iterated
integrals as follows.
𝑏 𝑔2 (𝑥)
∬ 𝑓(𝑥, 𝑦)𝑑𝐴 = ∫ [∫ 𝑓(𝑥, 𝑦)𝑑𝑦] 𝑑𝑥
𝑎 𝑔1 (𝑥)
𝑅

Case II:

Suppose that the region 𝑅 has the form


𝑅 = {(𝑥, 𝑦) | 𝑐 ≤ 𝑦 ≤ 𝑑 and ℎ1 (𝑦) ≤ 𝑥 ≤ ℎ2 (𝑦)},
the double integral is given by
𝑑 ℎ2 (𝑦)
∬ 𝑓(𝑥, 𝑦)𝑑𝐴 = ∫ [∫ 𝑓(𝑥, 𝑦)𝑑𝑥 ] 𝑑𝑦
𝑐 ℎ1 (𝑦)
𝑅

Example 1 Let 𝑹 be the region bounded by the graph 𝒚 = √𝒙, 𝒙 = 𝟎 and 𝒚 = 𝟑. Evaluate
∬𝑹 (𝟐𝒙𝒚𝟐 + 𝟐𝒚𝐜𝐨𝐬 𝒙 )𝒅𝑨
Solution:
3 𝑦2
∬𝑅 (2𝑥𝑦 2 + 2𝑦 cos 𝑥)𝑑𝐴 = ∫0 ∫0 (2𝑥𝑦 2 + 2𝑦 cos 𝑥)𝑑𝑥 𝑑𝑦
3 𝑥=𝑦 2
= ∫0 [𝑥 2 𝑦 2 + 2𝑦 sin 𝑥]𝑥=0 𝑑𝑦
3
= ∫0 (𝑦 6 + 2𝑦 sin 𝑦 2 )𝑑𝑦
𝑦=3
𝑦7 37
= [ − cos 𝑦 2 ] = − cos 9 + 1
7 𝑦=0
7

Alternatively, integrating with respect to 𝑦 first, we get

- 45 -
Chapter 3 Multiple Integrals
9 3 9 2 𝑦=3
∬𝑅 (2𝑥𝑦 2 + 2𝑦 cos 𝑥)𝑑𝐴 = ∫0 ∫√𝑥(2𝑥𝑦 2 + 2𝑦 cos 𝑥)𝑑𝑦 𝑑𝑥 = ∫0 [3 𝑥𝑦 3 + 𝑦 2 cos 𝑥] 𝑑𝑥
𝑦=√𝑥

9 2 3
= ∫0 ( 𝑥 (27 − 𝑥 2 ) + (9 − 𝑥) cos 𝑥) 𝑑𝑥
3

9 5
2
= ∫0 (18𝑥 − 𝑥 2 + 9 cos 𝑥 − 𝑥 cos 𝑥) 𝑑𝑥
3

7 9
4 4
= [9𝑥 2 − 𝑥 2 + 9 sin 𝑥 − 𝑥 sin 𝑥 − cos 𝑥] = 93 − (37 ) − cos 9 + 1
21 0 21

37
= − cos 9 + 1
7
Example 2 Let 𝑹 be the region bounded by the graphs of 𝒚 = 𝒙, 𝒚 = 𝟎 and 𝒙 = 𝟒. Evaluate
𝟐
∬𝑹 (𝟒𝒆𝒙 − 𝟓 𝐬𝐢𝐧 𝒚)𝒅𝑨
Solution:
2 4 𝑥 2
∬𝑅 (4𝑒 𝑥 − 5 sin 𝑦)𝑑𝐴 = ∫0 ∫0 (4𝑒 𝑥 − 5 sin 𝑦)𝑑𝑦 𝑑𝑥
4 2 𝑦=𝑥
= ∫0 [4𝑒 𝑥 𝑦 + 5 cos 𝑦]𝑦=0 𝑑𝑥

4 2
= ∫0 (4𝑥𝑒 𝑥 + 5 cos 𝑥 − 5)𝑑𝑥

2 𝑥=4
= [2𝑒 𝑥 + 5 sin 𝑥 − 5𝑥]𝑥=0

= 2𝑒16 + 5 sin 4 − 22

Example 3 Evaluate ∬𝑹 (𝟒𝒙𝒚 − 𝒚𝟑 )𝒅𝑨, 𝑹 is the region bounded by 𝒚 = √𝒙 and 𝒚 = 𝒙𝟑


Solution:
1 √𝑥
∬𝑅 (4𝑥𝑦 − 𝑦 3 )𝑑𝐴 = ∫0 ∫𝑥 3 (4𝑥𝑦 − 𝑦 3 )𝑑𝑦 𝑑𝑥
1 1 𝑦=√𝑥
= ∫0 [2𝑥𝑦 2 − 𝑦 4 ] 𝑑𝑥
4 𝑦=𝑥 3

1 7 1
= ∫0 ( 𝑥 2 − 2𝑥 7 + 𝑥 12 ) 𝑑𝑥
4 4

7 1 1 𝑥=1 55
= [ 𝑥3 − 𝑥8 + 𝑥 13 ] =
12 4 52 𝑥=0 156

Example 4 Evaluate ∬𝑹 (𝟔𝒙𝟐 − 𝟒𝟎𝒚)𝒅𝑨, 𝑹 is the triangle has vertices (𝟎, 𝟑), (𝟏, 𝟏) and (𝟓, 𝟑)
Solution:

3 2𝑦−1 3
2 𝑥=2𝑦−1
∬(6𝑥 − 40𝑦)𝑑𝐴 = ∫ ∫ (6𝑥 2 − 40𝑦)𝑑𝑥 𝑑𝑦 = ∫ [2𝑥 3 − 40𝑥𝑦] 1 3 𝑑𝑦
1 3 𝑥=−2𝑦+2
1 −2𝑦+2 1
𝑅
3
1 3 3 2 3
= ∫ (100𝑦 − 100𝑦 + 2(2𝑦 − 1) − 2 (− 𝑦 + ) ) 𝑑𝑦
1 2 2

- 46 -
Chapter 3 Multiple Integrals

𝑥=3
100 1 1 3 4
= [50𝑦 − 𝑦 3 + (2𝑦 − 1)4 + (− 𝑦 + ) ]
3 4 2 2
𝑥=1

935
=−
3

Example 5 Find the volume of the prism whose base is the triangle in 𝒙𝒚 − plane bounded by the
𝒙 − axis and the lines 𝒚 = 𝒙 and 𝒙 = 𝟏 and whose top lies in the plane 𝒛 = 𝒇(𝒙, 𝒚) =
𝟑 − 𝒙 − 𝒚.
Solution:

𝑉 = ∬𝑅 𝑓(𝑥, 𝑦)𝑑𝐴 = ∬𝑅 (3 − 𝑥 − 𝑦)𝑑𝐴


𝑦=𝑥
1 𝑥 1 𝑦2
= ∫0 ∫0 (3 − 𝑥 − 𝑦)𝑑𝑦 𝑑𝑥 = ∫0 [3𝑦 − 𝑥𝑦 − ] 𝑑𝑥
2 𝑦=0

𝑥=1
1 3 3 𝑥3
= ∫0 (3𝑥 − 𝑥 2 ) 𝑑𝑥 = [ 𝑥 2 − ] = 1.
2 2 2 𝑥=0

When the order of integration is reversed, the integral for the volume is

𝑉 = ∬𝑅 𝑓(𝑥, 𝑦)𝑑𝐴 = ∬𝑅 (3 − 𝑥 − 𝑦)𝑑𝐴


𝑥=1
1 1 1 𝑥2
= ∫0 ∫𝑦 (3 − 𝑥 − 𝑦)𝑑𝑥 𝑑𝑦 = ∫0 [3𝑥 − − 𝑥𝑦] 𝑑𝑦
2 𝑥=𝑦
1 1 3 1 3 3
= ∫0 (3 − − 𝑦 − 3𝑦 + 𝑦 2 ) 𝑑𝑦 = ∫0 ( − 4𝑦 + 𝑦 2 ) 𝑑𝑦
2 2 2 2
𝑦=1
3 𝑦3
= [ 𝑦 − 2𝑦 + ] = 1.
2 2 𝑦=0

The two integrals are equal, as they should be.

Example 6 Find the area of the region 𝑹 bounded by 𝒚 = 𝒙 and 𝒚 = 𝒙𝟐 in the first quadrant.
Solution:
1 𝑥 1
𝑦=𝑥
𝐴 = ∬ 𝑑𝐴 = ∫ ∫ 𝑑𝑦 𝑑𝑥 = ∫ [𝑦]𝑦=𝑥 2 𝑑𝑥
0 𝑥2 0
𝑅

1 𝑥=1
2 )𝑑𝑥
𝑥2 𝑥3 1
= ∫ (𝑥 − 𝑥 =[ − ] =
0 2 3 𝑥=0 6

- 47 -
Chapter 3 Multiple Integrals

Example 7 Find the area of the region 𝑹 enclosed by the parabola 𝒚 = 𝒙𝟐 and the line 𝒚 = 𝒙 + 𝟐.
Solution:
2 𝑥+2 2
𝑦=𝑥+2
𝐴 = ∬ 𝑑𝐴 = ∫ ∫ 𝑑𝑦 𝑑𝑥 = ∫ [𝑦]𝑦=𝑥 2 𝑑𝑥
−1 𝑥 2 −1
𝑅
2 𝑥=2
2 )𝑑𝑥
𝑥2 𝑥3 9
= ∫ (𝑥 + 2 − 𝑥 = [ + 2𝑥 − ] = .
−1 2 3 𝑥=−1 2

𝟐
Example 8 Evaluate ∬𝑹 𝒙𝒆𝒚 𝒅𝑨 over the region 𝑹 in the first quadrant bounded by the graphs of
𝒚 = 𝒙𝟐 , 𝒙 = 𝟎 and 𝒚 = 𝟒.
Solution:

When the region is viewed as type II


𝑥= 𝑦
𝑦2
4
𝑥 2 𝑦2 √
√𝑦
𝑦2
4
∬ 𝑥𝑒 𝑑𝐴 = ∫ ∫ 𝑥𝑒 𝑑𝑥 𝑑𝑦 = ∫ [ 𝑒 ] 𝑑𝑦
0 0 0 2 𝑥=0
𝑅

4 𝑦=4
𝑦 2 1 2 1
= ∫ ( 𝑒 𝑦 ) 𝑑𝑦 = [𝑒 𝑦 ] = [𝑒16 − 1]
0 2 4 𝑦=0 4

𝐬𝐢𝐧 𝒙
Example 9 Calculate ∬𝑹 𝒅𝑨 where 𝑹 is the triangle on the 𝒙𝒚 − plane bounded by 𝒙 −axis, the
𝒙
𝟏
line 𝒚 = 𝒙, 𝒙 = and the line 𝒙 = 𝟏.
𝟐

Solution:

sin 𝑥 1 𝑥
sin 𝑥 1
sin 𝑥 𝑦=𝑥
∬ 𝑑𝐴 = ∫ ∫ 𝑑𝑦 𝑑𝑥 = ∫ [𝑦 ] 𝑑𝑥
𝑥 1
0 𝑥 1 𝑥 𝑦=0
𝑅 2 2
1
= ∫ (sin 𝑥)𝑑𝑥 = −[cos 𝑥]𝑥=11
1 𝑥=2
2

1
= − cos 1 + cos
2

- 48 -
Chapter 3 Multiple Integrals

Switching the Order of Integration (Reversing the integration order)

𝟏 𝟏 𝟐
Example 10 Evaluate ∫𝟎 ∫𝒚 𝒆𝒙 𝒅𝒙 𝒅𝒚.
Solution:
First, note that we cannot evaluate the integral the way it is presently
2
written, as we don’t know an antiderivative for 𝑒 𝑥 .
On the other hand, if we switch the order of integration, the integral
becomes quite simple, as follows. First, recognize that for each fixed 𝑦 on
the interval [0, 1], 𝑥 ranges from 𝑦 over to 1, giving us the triangular region
of integration shown in Figure.
If we switch the order of integration, notice that for each fixed x in the
interval [0, 1], 𝑦 ranges from 0 up to 𝑥 and we get the double integral:
0 ≤ 𝑦 ≤ 1, 𝑦≤𝑥≤1
1 1 1 𝑥 1
𝑦=𝑥
𝑥2 𝑥2 2
∫ ∫ 𝑒 𝑑𝑥 𝑑𝑦 = ∫ ∫ 𝑒 𝑑𝑦 𝑑𝑥 = ∫ [𝑦𝑒 𝑥 ]𝑦=0 𝑑𝑥
0 𝑦 0 0 0
1
2 1 2 𝑥=1 1
= ∫ 𝑥𝑒 𝑥 𝑑𝑥 = [𝑒 𝑥 ]𝑥=0 = (𝑒1 − 1)
0 2 2
𝟑 𝟗 𝟑
Example 11 Evaluate ∫𝟎 ∫𝒙𝟐 𝒙𝟑 𝒆𝒚 𝒅𝒚 𝒅𝒙.
Solution:
Let’s see how we reverse the order of integration. The best way to
reverse the order of integration is to first sketch the region given by the
original limits of integration. From the integral we see that the
inequalities that define this region are,
𝑥 2 ≤ 𝑦 ≤ 9, 0 ≤ 𝑥 ≤ 3.
The integral, with the order reversed, is now,
3 9 9 √𝑦
3 3
∫ ∫ 𝑥 3 𝑒 𝑦 𝑑𝑦 𝑑𝑥 = ∫ ∫ 𝑥 3 𝑒 𝑦 𝑑𝑥 𝑑𝑦
0 𝑥2 0 0
𝑥= 𝑦
𝑥 4 𝑦3 √9 9
𝑦 2 𝑦3 1 𝑦3 𝑦=9
=∫ [ 𝑒 ] 𝑑𝑦 = ∫ [ 𝑒 ] 𝑑𝑦 = [ 𝑒 ]
0 4 𝑥=0 0 4 12 𝑦=0

1 729
= (𝑒 − 1).
12
𝟖 𝟐
Example 12 Evaluate ∫𝟎 ∫𝟑√𝒚 √𝒙𝟒 + 𝟏𝒅𝒙 𝒅𝒚.
Solution:
We’ll hope that by reversing the order of integration we will get
something that we can integrate. Here are the limits for the
variables that we get from this integral.
3
0 ≤ 𝑦 ≤ 8, √𝑦 ≤ 𝑥 ≤ 2.
and here is a sketch of this region. So, if we reverse the order
of integration, we get the following limits.
0 ≤ 𝑥 ≤ 2, 0 ≤ 𝑦 ≤ 𝑥3.
- 49 -
Chapter 3 Multiple Integrals
8 2 2 𝑥3 2 𝑦=𝑥 3
∫ ∫ √𝑥 4 + 1𝑑𝑥 𝑑𝑦 = ∫ ∫ √𝑥 4 + 1𝑑𝑦 𝑑𝑥 = ∫ [𝑦√𝑥 4 + 1] 𝑑𝑥
0 3 𝑦=0
√𝑦 0 0 0
2 3 𝑥=2
1 1 3
= ∫ [𝑥 3 √𝑥 4 + 1] 𝑑𝑥 = [(𝑥 4 + 1)2 ] = (172 − 1)
0 6 𝑥=0 6

Double Integrals in Polar Coordinates:


Polar coordinates are particularly useful for dealing with certain double integrals, for several reasons.
Most importantly, if the region over which you are integrating is in some way circular, polar coordinates
may significantly simplify the integration.
Suppose the region 𝑅 can be written in the form

𝑅 = {(𝑟, 𝜃) | 𝛼 ≤ 𝜃 ≤ 𝛽 and 𝑔1 (𝜃) ≤ 𝑟 ≤ 𝑔2 (𝜃)},

where 0 ≤ 𝑔1 (𝜃) ≤ 𝑔2 (𝜃), for all 𝜃 in [𝛼, 𝛽], as pictured in the following
Figure. First, we partition 𝑅, but rather than use a rectangular grid, as we
have done with rectangular coordinates, we use a partition consisting of a
number of concentric circular arcs (of the form 𝑟 = constant) and rays (of
the form 𝜃 = constant).

We indicate such a partition of the region R in the following Figure.


Notice that rather than consisting of rectangles, the “grid” in this case is
made up of elementary polar regions, each bounded by two circular arcs
and two rays (as shown in the third Figure). In an inner partition, we
include only those elementary polar regions that lie completely inside R.

We pause now briefly to calculate the area ∆𝐴 of the elementary polar


region indicated in the following Figure. Recall that the area of a circular
1
sector is given by 𝐴 = 𝜃𝑟 2 , where 𝑟 = radius and 𝜃 is the central angle
2
of the sector. Consequently, we have the area of a circular sector can be
thought of as an ordinary rectangle with dimensions 𝑟 𝑑 𝜃 and 𝑑 𝑟 and
therefore has area

∆𝐴 = 𝑟∆𝑟∆𝜃 ⇒ 𝑑𝐴 = 𝑟𝑑𝑟𝑑𝜃.

THEOREM (Fubini’s Theorem)


Suppose that 𝑓 (𝑟, 𝜃) is continuous on the region 𝑅 = {(𝑟, 𝜃) | 𝜃1 ≤ 𝜃 ≤ 𝜃2 and 𝑟1 ≤ 𝑟 ≤ 𝑟2 },
where 0 ≤ 𝑟1 ≤ 𝑟2 for all 𝜃 in [𝜃1 , 𝜃2 ]. Then,

𝜃2 𝑟2
∬ 𝑓(𝑥, 𝑦)𝑑𝐴 = ∫ ∫ 𝑓(𝑟, 𝜃)𝑟𝑑𝑟𝑑𝜃
𝜃1 𝑟1
𝑅
and
𝑥 = 𝑟 cos 𝜃 , 𝑦 = 𝑟 sin 𝜃 and 𝑥2 + 𝑦2 = 𝑟2
- 50 -
Chapter 3 Multiple Integrals

The regions in the following Figure are special cases of a polar rectangle:
𝑅 = {(𝑟, 𝜃) | 𝜃1 ≤ 𝜃 ≤ 𝜃2 and 𝑟1 ≤ 𝑟 ≤ 𝑟2 }

y y
x2  y 2  9
x2  y 2  9

x x

x2  y 2  4

𝑅 = {(𝑟, 𝜃) | 0 ≤ 𝜃 ≤ 2𝜋 and 0 ≤ 𝑟 ≤ 3} 𝑅 = {(𝑟, 𝜃) | 0 ≤ 𝜃 ≤ 2𝜋 and 2 ≤ 𝑟 ≤ 3}

y y
x2  y 2  9 x2  y 2  9

x x

x2  y 2  4

𝜋 𝜋
𝑅 = {(𝑟, 𝜃) | 0 ≤ 𝜃 ≤ 𝜋 and 0 ≤ 𝑟 ≤ 3} 𝑅 = {(𝑟, 𝜃) | − ≤ 𝜃 ≤ and 2 ≤ 𝑟 ≤ 3}
2 2

y y
x2  y 2  9
x  y 9
2 2

x x

x2  y 2  4

𝜋 𝜋 3𝜋
𝑅 = {(𝑟, 𝜃) | 0 ≤ 𝜃 ≤ and 0 ≤ 𝑟 ≤ 3} 𝑅 = {(𝑟, 𝜃) | ≤ 𝜃 ≤ and 2 ≤ 𝑟 ≤ 3}
2 2 2

- 51 -
Chapter 3 Multiple Integrals
𝟐
Example 13 Evaluate ∬𝑹 (𝟑𝒙 + 𝟒𝒚 )𝒅𝑨, where 𝑹 is the region in the upper half-plane bounded by
the circles 𝒙𝟐 + 𝒚𝟐 = 𝟏 and 𝒙𝟐 + 𝒚𝟐 = 𝟒.
Solution:
The region 𝑅 can be described as
x2  y 2  1 y x2  y 2  4
𝑅 = {(𝑟, 𝜃) |0 ≤ 𝜃 ≤ 𝜋 and 1 ≤ 𝑟 ≤ 2}

Therefore,
𝜋 2
∬ (3𝑥 + 4𝑦 2 )𝑑𝐴 = ∫ ∫ (3𝑟 cos 𝜃 + 4𝑟 2 sin2 𝜃)𝑟𝑑𝑟𝑑𝜃
x
𝑅 0 1
𝜋 2
= ∫ ∫ (3𝑟 2 cos 𝜃 + 4𝑟 3 sin2 𝜃)𝑑𝑟𝑑𝜃
0 1
𝜋 𝜋
=∫ [𝑟 3 cos 𝜃 + 𝑟 4 sin2 𝜃]𝑟=2 2
𝑟=1 𝑑𝜃 = ∫ (7 cos 𝜃 + 15 sin 𝜃)𝑑𝜃
0 0
𝜋
15
= ∫ (7 cos 𝜃 + [1 − cos 2𝜃]) 𝑑𝜃
0 2
𝜋 𝜃=𝜋
15 1 15𝜋
= ∫ (7 sin 𝜃 + [𝜃 − sin 2𝜃]) 𝑑𝜃 =
0 2 2 𝜃=0 2

𝟐 𝟐
Example 14 Evaluate ∬𝑹 𝒆𝒙 +𝒚 𝒅𝑨, where 𝑹 = {(𝒙, 𝒚)| 𝒙 ≥ 𝟎, 𝒚 ≥ 𝟎 𝐚𝐧𝐝 𝒙𝟐 + 𝒚𝟐 ≤ 𝟏}
Solution:
The region 𝑅 can be described as y
x2  y 2  1
𝜋
𝑅 = {(𝑟, 𝜃) |0 ≤ 𝜃 ≤ and 0 ≤ 𝑟 ≤ 1}
2

Therefore,
𝜋
x
𝜋/2 1
2 +𝑦 2 2 1 2 2 𝑟=1
∬ 𝑒𝑥 𝑑𝐴 = ∫ ∫ (𝑒 𝑟 )𝑟𝑑𝑟𝑑𝜃 = ∫ [𝑒 𝑟 ]𝑟=0 𝑑𝜃
𝑅 0 0 2 0
𝜋
1 2 1 1 1 𝜃=𝜋/2 (𝑒1 − 1)𝜋
= ∫ (𝑒 − 1) 𝑑𝜃 = (𝑒 − 1)[𝜃]𝜃=0 =
2 0 2 4
𝟎 𝟎 𝟐
Example 15 Evaluate ∫−𝟏 ∫−√𝟏−𝒙𝟐 𝒅𝒚𝒅𝒙
𝟏+√𝒙𝟐 +𝒚𝟐
Solution: y
The region 𝑅 can be described as
−1 ≤ 𝑥 ≤ 0 and − √1 − 𝑥 2 ≤ 𝑦 ≤ 0
3𝜋 x
𝑅 = {(𝑟, 𝜃) |𝜋 ≤ 𝜃 ≤ and 0 ≤ 𝑟 ≤ 1}
2

Therefore,
x2  y 2  1
0 0
2
∫ ∫ 𝑑𝑦𝑑𝑥
−1 −√1−𝑥 2 1 + √𝑥 2 + 𝑦 2
3𝜋
3𝜋/2 1 3𝜋/2 1 1
2 1+𝑟−1 2 1
=∫ ∫ ( ) 𝑟𝑑𝑟𝑑𝜃 = 2 ∫ ∫ ( ) 𝑑𝑟𝑑𝜃 = 2 ∫ ∫ (1 − ) 𝑑𝑟𝑑𝜃
𝜋 0 1+𝑟 𝜋 0 1+𝑟 𝜋 0 1+𝑟
3𝜋
2 2(1 − ln 2)𝜋
= 2 ∫ (1 − ln(1 + 𝑟))𝑟=1
𝑟=0 𝑑𝜃 = .
𝜋 2
- 52 -
Chapter 3 Multiple Integrals
Green’s Theorem:
Introduction:
One of the most important theorems in vector integral calculus relates a line integral around a piecewise-
smooth simple closed curve 𝐶 to a double integral over the region 𝑅 bounded by the curve.

Line Integrals along Simple Closed Curves

We say the positive direction around a simple closed curve 𝐶 is that direction a point on the curve must
move, or the direction a person must walk on 𝐶, in order to keep the region 𝑅 bounded by 𝐶 to the left. The
positive and negative directions correspond to the counterclockwise and clockwise directions, respectively.
Line integrals on simple closed curves are written

∮ 𝑓1 (𝑥, 𝑦)𝑑𝑥 + 𝑓2 (𝑥, 𝑦)𝑑𝑦


𝑐

Theorem: Green's Theorem in the Plane

Suppose that 𝐶 is a piecewise-smooth simple closed curve bounding


𝜕𝑓1 (𝑥,𝑦) 𝜕𝑓2 (𝑥,𝑦)
a simply connected region 𝑅. If 𝑓1 (𝑥, 𝑦), 𝑓2 (𝑥, 𝑦), , and are
𝜕𝑦 𝜕𝑥
continuous on 𝑅, then
𝜕𝑓2 (𝑥, 𝑦) 𝜕𝑓1 (𝑥, 𝑦)
∮ 𝑓1 (𝑥, 𝑦)𝑑𝑥 + 𝑓2 (𝑥, 𝑦)𝑑𝑦 = ∬ ( − ) 𝑑𝐴
𝑐 𝜕𝑥 𝜕𝑦
𝑅

Example 11 Use Green’s Theorem to evaluate the line integral ∮𝒄 (𝒙𝟐 − 𝒚𝟐 )𝒅𝒙 + (𝟐𝒚 − 𝒙)𝒅𝒚, where
𝑪 is the boundary of the region in the first quadrant that is bounded bu the curves 𝒚 =
𝒙𝟐 and 𝒚 = 𝒙𝟑 in counterclockwise direction.
Solution:
If 𝑓1 (𝑥, 𝑦) = 𝑥 2 − 𝑦 2 and 𝑓2 (𝑥, 𝑦) = 2𝑦 − 𝑥, then 1
𝜕𝑓 (𝑥,𝑦)
= −2𝑦 y
𝜕𝑦 (1,1)
𝜕𝑓2 (𝑥,𝑦)
and = −1.
𝜕𝑥

∮ (𝑥 2 − 𝑦 2 )𝑑𝑥 + (2𝑦 − 𝑥)𝑑𝑦 = ∬(−1 + 2𝑦) 𝑑𝐴


𝑐
𝑅 y x2
1 𝑦=𝑥 2 1
𝑦=𝑥 2
= ∫ ∫ (−1 + 2𝑦) 𝑑𝑦𝑑𝑥 = ∫(−𝑦 + 𝑦 2 )𝑦=𝑥 3 𝑑𝑥 y x3
0 𝑦=𝑥 3 0
1 (0, 0)
11 x
= ∫(−𝑥 2 + 𝑥 4 + 𝑥 3 − 𝑥 6 )𝑑𝑥 = − .
420
0

Example 12 Use Green’s theorem to find the work done by the force 𝑭⃑ = (𝒆𝒙 − 𝒚𝟑 )𝐢 + (𝐜𝐨𝐬 𝒚 + 𝒙𝟑 ) 𝐣
on a particle that travels once around the unit circle 𝒙𝟐 + 𝒚𝟐 = 𝟏 in counterclockwise
direction.
Solution:
The work done by 𝐹 is given by

- 53 -
Chapter 3 Multiple Integrals

𝑊 = ∮ 𝐹 . 𝑑𝑟 = ∮ (𝑒 𝑥 − 𝑦 3 )𝑑𝑥 + (cos 𝑦 + 𝑥 3 )𝑑𝑦


𝑐 𝑐
and so by Green's theorem
𝜕𝑓2 (𝑥, 𝑦) 𝜕𝑓1 (𝑥, 𝑦)
𝑊 = ∬( − ) 𝑑𝐴 = ∬ (3𝑥 2 + 3𝑦 2 )𝑑𝐴
𝜕𝑥 𝜕𝑦 𝑅
𝑅
In view of the region 𝑅, the last integral is best handled in polar coordinates. Since 𝑅 is defined by
𝑅 = {(𝑟, 𝜃) | 0 ≤ 𝜃 ≤ 2𝜋 and 0 ≤ 𝑟 ≤ 1},
2𝜋 1 2𝜋 1 2𝜋 𝑟=1 2𝜋
𝑟4 3 3 3
𝑊 = 3 ∫ ∫(𝑟 2 )𝑟𝑑𝑟𝑑𝜃 = 3 ∫ ∫ 𝑟 3 𝑑𝑟𝑑𝜃 = 3 ∫ [ ] 𝑑𝜃 = ∫ 1 𝑑𝜃 = (2𝜋) = 𝜋.
4 𝑟=0 4 4 2
0 0 0 0 0 0

Example 13 Verify Green’s Theorem for ⃑𝑭 = (𝒙𝟐 + 𝒚)𝐢 + (𝒙𝒚𝟐 )𝐣, 𝑪 𝒚𝟐 = 𝒙, 𝒚 = −𝒙 .


Solution:
To verify Green Theorem, first, the line integral must calculate. So, the
line integral

∮ (𝑥 2 + 𝑦)𝑑𝑥 + (𝑥𝑦 2 )𝑑𝑦


𝑐
will calculate along two paths 𝐶1 and 𝐶2 .
Along 𝑪𝟏 : 𝒚𝟐 = 𝒙:
𝑟 = < 𝑡 2 , 𝑡 > , ⃑⃑⃑
𝑟 ′ = < 2𝑡, 1 > , 𝐹 (𝑟(𝑡)) = < 𝑡 4 + 𝑡, 𝑡 4 >

𝐼1 = ∫ (𝑥 2 + 𝑦)𝑑𝑥 + (𝑥𝑦 2 )𝑑𝑦 = ∫ 𝐹 (𝑟(𝑡)) ∙ ⃑⃑⃑


𝑟 ′ (𝑡)𝑑𝑡
𝑐1 𝑐1
−1

= ∫ (2𝑡 5 + 2𝑡 2 + 𝑡 4 ) 𝑑𝑡
0
6 3 5 𝒕=−𝟏
𝑡 𝑡 𝑡 1 2 1 8
=[
+2 + ] = − − =−
3 3 5 𝒕=𝟎 3 3 5 15
Along 𝑪𝟐 : 𝒚 = −𝒙:
𝑟 = < 𝑡, − 𝑡 > , ⃑⃑⃑
𝑟 ′ = < 1, −1 > , 𝐹 (𝑟(𝑡)) = < 𝑡 2 − 𝑡, 𝑡 3 >
0

𝐼2 = ∫ (𝑥 2 + 𝑦)𝑑𝑥 + (𝑥𝑦 2 )𝑑𝑦 = ∫ 𝐹 (𝑟(𝑡)) ∙ ⃑⃑⃑


𝑟 ′ (𝑡)𝑑𝑡 = ∫(𝑡 2 − 𝑡 − 𝑡 3 ) 𝑑𝑡
𝑐2 𝑐2 1
3 2 4 𝒕=𝟎
𝑡 𝑡 𝑡 1 1 1 5
=[ − − ] = −( − − ) = .
3 2 4 𝒕=𝟏 3 2 4 12
7
So, the value of the line integral along the two paths is 𝐼 = 𝐼1 + 𝐼2 = − .
60

The right hand side of Green Thoerm can calculate as follow

𝜕𝑓1 (𝑥,𝑦) 𝜕𝑓2 (𝑥,𝑦)


If 𝑓1 (𝑥, 𝑦) = 𝑥 2 + 𝑦 and 𝑓2 (𝑥, 𝑦) = 𝑥𝑦 2 , then = 1 and = 𝑦2.
𝜕𝑦 𝜕𝑥

𝑥=1 𝑦=−𝑥

∮ (𝑥 2 + 𝑦)𝑑𝑥 + (𝑥𝑦 2 )𝑑𝑦 = ∬(𝑦 2 − 1) 𝑑𝐴 = ∫ ∫ (𝑦 2 − 1) 𝑑𝑦𝑑𝑥


𝑐
𝑅 𝑥=0 𝑦=−√𝑥

- 54 -
Chapter 3 Multiple Integrals
𝑥=1 𝑥=1 3 5 𝑥=1
𝑦=−𝑥
𝑦3 𝑥3 1 𝑥4 𝑥
𝑥2 𝑥2 2 3
= ∫ ( − 𝑦) 𝑑𝑥 = ∫ (− + 𝑥 + − 𝑥 ) 𝑑𝑥 = (− + + 2 − 𝑥 2 )
2
3 𝑦=−
3 3 12 2 15 3
𝑥=0 √𝑥 𝑥=0 𝑥=0
1 1 2 2 7
=− + + − =− .
12 2 15 3 60

Example 13 Verify Green Theorem for ⃑𝑭 = (𝒙𝟐 + 𝒚)𝐢 + (𝒙𝒚𝟐 )𝐣 and 𝑪 is closed curve of the region
bounded by the triangle has vertices (𝟎, 𝟎), (𝟏, 𝟎) and (𝟏, 𝟏). 𝑪: 𝒚𝟐 = 𝒙, 𝒚 = −𝒙
Solution:
To verify Green Theorem, first, the line integral must calculate. So, the
y (1,1)
line integral C2 : y  x
C2 : x  1
∮ (1 − 𝑥𝑦)𝑑𝑥 + (𝑥 2 𝑦)𝑑𝑦
𝑐
will calculate along three paths 𝐶1 , 𝐶2 and 𝐶3 . (0, 0) (1, 0) x
C1 : y  0
Along 𝑪𝟏 : 𝒚 = 𝟎:
𝑟 = < 𝑡, 0 > , ⃑⃑⃑
𝑟 ′ = < 1, 0 > , 𝐹 (𝑟(𝑡)) = < 1, 0 >
1

𝐼1 = ∫ (1 − 𝑥𝑦 2 )𝑑𝑥 + (𝑥 2 𝑦)𝑑𝑦 = ∫ 𝐹 (𝑟(𝑡)) ∙ ⃑⃑⃑


𝑟 ′ (𝑡)𝑑𝑡 = ∫ 1 𝑑𝑡
𝑐1 𝑐1 0
= [𝑡]𝒕=𝟏
𝒕=𝟎 = 1.

Along 𝑪𝟐 : 𝒙 = 𝟏:
𝑟 = < 1, 𝑡 > , ⃑⃑⃑
𝑟 ′ = < 0, 1 > , 𝐹 (𝑟(𝑡)) = < 1 − 𝑡 2 , 𝑡 >
1 𝑡=1
𝑡2 1
𝐼2 = ∫ (1 − 𝑥𝑦 2 )𝑑𝑥 2 ⃑⃑⃑′
+ (𝑥 𝑦)𝑑𝑦 = ∫ 𝐹 (𝑟(𝑡)) ∙ 𝑟 (𝑡)𝑑𝑡 = ∫(𝑡) 𝑑𝑡 = [ ] = .
2 𝑡=0 2
𝑐2 𝑐2 0

Along 𝑪𝟑 : 𝒚 = 𝒙:
𝑟 = < 𝑡, 𝑡 > , ⃑⃑⃑
𝑟 ′ = < 1, 1 > , 𝐹 (𝑟(𝑡)) = < 1 − 𝑡 2 , 𝑡 3 >
0

𝐼2 = ∫ (1 − 𝑥𝑦 2 )𝑑𝑥 + (𝑥 2 𝑦)𝑑𝑦 = ∫ 𝐹 (𝑟(𝑡)) ∙ ⃑⃑⃑


𝑟 ′ (𝑡)𝑑𝑡 = ∫(1 − 𝑡 3 + 𝑡 3 ) 𝑑𝑡 = [𝑡]𝑡=0
𝑡=1 = −1
𝑐2 𝑐2 1
So, the value of the line integral along the two paths is
1
𝐼 = 𝐼1 + 𝐼2 + 𝐼3 = .
2

The right hand side of Green Thoerm can calculate as follow

𝜕𝑓1 (𝑥,𝑦) 𝜕𝑓2 (𝑥,𝑦)


If 𝑓1 (𝑥, 𝑦) = 1 − 𝑥𝑦 2 and 𝑓2 (𝑥, 𝑦) = 𝑥 2 𝑦, then = −2𝑥𝑦 and = 2𝑥𝑦.
𝜕𝑦 𝜕𝑥

𝑥=1 𝑦=𝑥 𝑥=1


𝑦=𝑥
∮ (1 − 𝑥𝑦 2 )𝑑𝑥 + (𝑥 2 𝑦)𝑑𝑦 = ∬(4𝑥𝑦) 𝑑𝐴 = ∫ ∫ (4𝑥𝑦) 𝑑𝑦𝑑𝑥 = ∫ (2𝑥𝑦 2 )𝑦=0 𝑑𝑥
𝑐
𝑅 𝑥=0 𝑦=0 𝑥=0

𝑥=1 𝑥=1
2𝑥 4 1
= ∫ (2𝑥 3 ) 𝑑𝑥 = ( ) = .
4 𝑥=0 2
𝑥=0
- 55 -
Chapter 3 Multiple Integrals

- 56 -
Chapter 3 Multiple Integrals

Sheet 5

1. Evaluate ∬𝑅 𝑥𝑦 𝑑𝐴; where 𝑅 is bounded by 𝑦 = 2 − 𝑥 and 𝑦 = √𝑥, 𝑦 = 0.

2. Evaluate ∬𝑅 12𝑦 𝑑𝐴; where 𝑅 is bounded by 𝑥 = 0, 𝑦 = 2𝑥 + 1 and 𝑦 = −2𝑥 + 5.

3. Evaluate ∬𝑅 (𝑥 + 𝑦) 𝑑𝐴, where 𝑅 is bounded by 𝑦 = |𝑥| and 𝑦 = 4.

4. Evaluate ∬𝑅 3𝑥 2 𝑑𝐴, where 𝑅 is bounded by 𝑦 = 0, 𝑦 = 2𝑥 + 4 and 𝑦 = 𝑥 3 .


1
5. Evaluate ∬𝑅 𝑑𝐴, where 𝑅 = {(𝑥, 𝑦): 𝑥 2 + 𝑦 2 ≤ 4, 𝑥 ≥ 0, 𝑦 ≥ 0}.
√16−𝑥 2 −𝑦2

6. Evaluate ∬𝑅 2𝑥𝑦 𝑑𝐴, where 𝑅 = {(𝑥, 𝑦): 𝑥 2 + 𝑦 2 ≤ 9, 𝑦 ≥ 0}


7. Find the area using the double integrals of the region 𝑅 lying in the first quadrant bounded by the 𝑦 −axis
and the parabolas 𝑦 = 𝑥 2 and 𝑦 = 1 − 𝑥 2 .

8. Find the area using the double integrals of the region 𝑅 bounded by 𝑥 + 𝑦 = 1, 𝑦 = 𝑒 𝑥 , 𝑦 = ln 𝑥 and
𝑥 = 2.
1
9. Find the volume of the solid below the surface 𝑓(𝑥, 𝑦) = 2 + and above the region 𝑅 in the 𝑥𝑦-plane
𝑦
bounded by the lines 𝑦 = 𝑥, 𝑦 = 8 − 𝑥, and 𝑦 = 1.

10. Find the volume of the solid region bounded by the paraboloids 𝑧 = 𝑥 2 + 𝑦 2 and 𝑧 = 8 − 𝑥 2 − 𝑦 2 .

11. Evaluate

√𝜋 √𝜋 2 4−𝑥 2 𝑥𝑒 2𝑦 1 1 2
(i) ∫0 ∫𝑦 sin(𝑥 2 ) 𝑑𝑥𝑑𝑦 (ii) ∫0 ∫0
4−𝑦
𝑑𝑦𝑑𝑥 (iii) ∫0 ∫𝑦 𝑒 𝑥 𝑑𝑥𝑑𝑦

4 2 𝑥 1 √1−𝑦 2 2 +𝑦 2 1 0
(iv) ∫0 ∫√𝑥 5 𝑑𝑦𝑑𝑥 (v) ∫0 ∫0 𝑒𝑥 𝑑𝑥𝑑𝑦 (vi) ∫−1 ∫−√1−𝑥 2(𝑥 2 + 𝑦 2 )3/2 𝑑𝑦𝑑𝑥
𝑦 +1
- 57 -
Chapter 3 Multiple Integrals
12. Use Green’s Theorem to evaluate the following line integrals
2 2
(i) ∮𝑐 (2𝑥 + 𝑒 𝑦 )𝑑𝑦 − (4𝑦 2 + 𝑒 𝑥 )𝑑𝑥, where 𝐶 is the boundary of the square with vertices
(0, 0), (1, 0), (1, 1) and (0, 1).

(ii) ∮𝑐 𝑥 2 𝑑𝑦 − 2𝑦 2 𝑑𝑥, where 𝐶 is the upper half of the unit circle from (1,0) to (−1, 0).

(iii)∮𝑐 (𝑥 2 + 𝑦 3 )𝑑𝑥 + 3𝑥𝑦 2 𝑑𝑦, where 𝐶 consists of the portion of 𝑦 = 𝑥 2 from (2, 4) to (0, 0),
followed by the line segments from (0, 0) to (2, 0) and from (2, 0) to (2, 4).

(iv) ∮𝑐 (𝑒 𝑥 + 6𝑥𝑦)𝑑𝑥 + (8𝑥 2 + sin 𝑦 2 )𝑑𝑦, where 𝐶 is positively-oriented boundary of the region
bounded by the circles of radii 1 and 3, centered at the origin and lying in the first quadrant.

13. Verify Green's Theorem for the vector 𝐹 along the path 𝐶, oriented counterclockwise
(i) 𝐹 = (𝑥𝑦 + 𝑦 2 ) 𝐢 + 𝑥 2 𝐣 and 𝐶 is closed curve of the region bounded by 𝑦 = 𝑥 and 𝑦 = 𝑥 2 .
(ii) 𝐹 = 2𝑥 2 𝑦 𝐢 + 2𝑥 3 𝐣 and 𝐶 is closed curve of the region bounded by 𝑥 = 2, 𝑦 = 0 and 𝑦 = 2𝑥.
(iii)𝐹 = 𝑦 3 𝐢 − 𝑥 3 𝐣 and 𝐶 is closed curve of the region bounded by 𝑦 = √1 − 𝑥 2 and 𝑦 = 0.

Homework 5:
𝑦
1. Evaluate ∬𝑅 𝑑𝐴, where 𝐷 = {(𝑥, 𝑦)| 0 ≤ 𝑥 ≤ 1, 0 ≤ 𝑦 ≤ 𝑥 2 }.
𝑥 5 +1

2. Evaluate ∬𝑅 𝑥 3 𝑑𝐴, where 𝐷 = {(𝑥, 𝑦)| 1 ≤ 𝑥 ≤ 𝑒, 0 ≤ 𝑦 ≤ ln 𝑥}.


3. Evaluate ∬𝑅 (𝑥 + 2𝑦)𝑑𝐴, where D is the region bounded by the parabolas 𝑦 = 2𝑥 2 and 𝑦 = 1 + 𝑥 2 .
4. Evaluate the following double integrals
1 1 1 1 4 2
2)
sin 𝑥
(i) ∫ ∫ sin(𝑦 𝑑𝑦𝑑𝑥 (ii) ∫ ∫ 𝑑𝑥𝑑𝑦 (iii) ∫ ∫ 𝑦 cos 𝑥 5 𝑑𝑥𝑑𝑦
0 𝑥 0 𝑦 𝑥 0 √𝑦
4 2 1 √1−𝑥 2 1 √1−𝑦 2
3
(iv) ∫ ∫ 𝑒 𝑥 𝑑𝑥𝑑𝑦 (v) ∫ ∫ √𝑥 2 + 𝑦 2 𝑑𝑦𝑑𝑥 (vi) ∫ ∫ cos(𝑥 2 + 𝑦 2 ) 𝑑𝑥𝑑𝑦
0 √𝑦 0 0 0 0
√2 √4−𝑦 2 2 √4−𝑦 2
1 2 +𝑦 2
(vii) ∫ ∫ 𝑑𝑥𝑑𝑦 (viii) ∫ ∫ 𝑒𝑥 𝑑𝑥𝑑𝑦
0 𝑦 √1 + 𝑥2 + 𝑦2 −2 −√4−𝑦 2

5. Evaluate ∬𝑅 (2 − 3𝑥 + 𝑥𝑦)𝑑𝐴, where 𝑅 is a triangular region with vertices (0, 0), (1, 0) and (1,3).

6. Sketch the region 𝑅 in the 𝑥𝑦-plane bounded by the given curves and find its area using the double
integrals:
(i) 𝑦 2 = 2𝑥 and 𝑦 = 𝑥.
1
(ii) 𝑦 = , 𝑦 = −𝑥 2 , 𝑥 = 1 and 𝑥 = 2.
𝑥
(iii ) 𝑦 = ln 𝑥 , 𝑥 = 0, 𝑦 = 0 and 𝑦 = 2.
𝜋
(iv) 𝑦 = sin 𝑥, 𝑦 = cos 𝑥, 𝑥 = 0 and 𝑥 = .
4
(v) 𝑦 = 𝑥, 𝑦 = 2𝑥 and 𝑥 + 𝑦 = 1.
(vi) 𝑥 = 𝑦 3 , 𝑥 + 𝑦 = 2 and 𝑦 = 0.
- 58 -
Chapter 3 Multiple Integrals

7. Find the volume of the solid that lies below the surface given by 𝑧 = 16𝑥𝑦 + 200 and lies above the
region in the 𝑥𝑦-plane bounded by 𝑦 = 𝑥 2 and 𝑦 = 8 − 𝑥 2 .
8. Find the volume of the solid bounded above by the plane 𝑧 = 4 − 𝑥 − 𝑦 and below by the rectangle
𝑅 = {(𝑥, 𝑦)| 0 ≤ 𝑥 ≤ 1, 0 ≤ 𝑦 ≤ 2}.
9. Use Green’s Theorem to find the work done by the force field 𝐹 on a particle that moves along the stated
path
1. 𝐹 (𝑥, 𝑦) = √𝑦 𝐢 + √𝑥 𝐣, the partical moves counterclockwise one time around the closed curve
𝑥3
given by the equations 𝑦 = 0, 𝑥 = 3 and 𝑦 = .
4
1
2. 𝐹 (𝑥, 𝑦) = 𝑥𝑦 𝐢 + ( 𝑥 2 + 𝑥𝑦) 𝐣, the partical starts at (5,0) traverses the upper semicircle 𝑥 2 +
2
𝑦 2 = 25 and returns to its starting point along 𝑥 −axis.

10. Verify Green's Theorem for the vector 𝐹 along the path 𝐶, oriented counterclockwise
1. 𝐹 (𝑥, 𝑦) = 14𝑦 2 𝐢 + 14𝑥 2 𝐣, 𝐶: the boundary of the region lying between the graphs of 𝑦 = 𝑥 and
𝑦 = 𝑥2.
2. 𝐹 (𝑥, 𝑦) = (2𝑥 3 − 𝑦 3 ) 𝐢 + (𝑥 3 + 𝑦 3 ) 𝐣,𝐶: the boundary of the unit circle.
3. 𝐹 = 𝑥𝑦 𝐢 + 𝑥𝑦 𝐣, 𝐶: the boundary of the region lying between the graphs of 𝑦 = √𝑥, 𝑦 = 1 and
𝑥 = 0.

Challenging Problems:

1. Explain how to calculate the area of region 𝑅 using Green’s Theorem?


2. Evaluate

1 √1−𝑥 2 1 1−𝑥

∫ ∫ √𝑥 2 + 𝑦 2 𝑑𝑦𝑑𝑥 + ∫ ∫ √𝑥 2 + 𝑦 2 𝑑𝑦𝑑𝑥
0 1−𝑥 0 0

3. Use the double integrals to find the shaded area in the following figures

- 59 -
Chapter 3 Multiple Integrals
Planes, Cylinders and Quadric Surfaces: z
(0, 0, c)
Planes:
A flat surface that extends into infinity in all directions is known as a
Plane. The general intercept form of the plane is given by:

𝑥 𝑦 𝑧 y
+ + =1 (0, b, 0)
𝑎 𝑏 𝑐
x (a, 0, 0)
Special Planes:
Parallel to 𝑥- axis Parallel to 𝑦- axis Parallel to 𝑧- axis

𝑦 𝑧 𝑥 𝑦
+ =1 + =1
𝑏 𝑐 𝑥 𝑧 𝑎 𝑏
+ =1
𝑎 𝑐
Parallel to 𝑥𝑦- plane Parallel to 𝑥𝑦- plane Parallel to 𝑦𝑧- plane

𝑧
=1 ⇒ 𝑧=𝑐
𝑐 𝑦 𝑥
=1 ⇒ 𝑦=𝑏 =1 ⇒ 𝑥=𝑎
𝑏 𝑎

Cylinders:
The cylinders, for the algebraic point of view, are the quadratic
equations involving 2 variables. Since 3-d quadratic equations,
generally, should contain 3 variables (𝑥, 𝑦, 𝑧), if there are only 2
variables appearing in the equations, it means along the missing
variable, the surface will repeat itself infinitely many times.
Circular Cylinder:
In 2-d space, 𝑥 2 + 𝑦 2 = 9 stands for a circle. But when we extend the
dimension into 3-d, it is called circular cylinder The fact that 𝑧 is
missing in the equation implies that if we cut 𝑥 2 + 𝑦 2 = 9 by 𝑧 = 𝐶,
the intersection (horizontal trace) will be always a circle. And the
vertical traces are lines. The figure is a 3-d circular cylinder.
- 60 -
Chapter 3 Multiple Integrals
Parabolic Cylinder:
The graph of the surface 𝑧 = 𝑦 2 is called parabolic cylinder. Notice that since there are no 𝑥' s in the
equation, the trace of the graph in the plane 𝑥 = 𝑘 is the same for every 𝑘. This is then a cylinder whose

trace in every plane parallel to the 𝑦𝑧 - plane is the parabola 𝑧 = 𝑦 2 . To draw this, we first draw the trace
in the 𝑦𝑧 - plane (see the left Figure) and then make several copies of the trace, locating the vertices at
various points along the 𝑥 - axis and finally, connect the traces with lines parallel to the 𝑥 - axis to give the
drawing its three-dimensional look (see the right Figure).

Quadric Surfaces:

The graph of the equation


𝑎𝑥 2 + 𝑏𝑦 2 + 𝑐𝑧 2 + 𝑑𝑥𝑦 + 𝑒𝑦𝑧 + 𝑓𝑥𝑧 + 𝑔𝑥 + ℎ𝑦 + 𝑗𝑧 + 𝑘 = 0
in three-dimensional space (where 𝑎, 𝑏, 𝑐, 𝑑, 𝑒, 𝑓, 𝑔, ℎ, 𝑗 and 𝑘 are all constants and at least one
of 𝑎, 𝑏, 𝑐, 𝑑, 𝑒 or 𝑓 is nonzero) is referred to as a quadric surface.
Sphere:
The most familiar quadric surface is the sphere:
𝑥2 + 𝑦2 + 𝑧2 = 𝑟2
of radius 𝑟 centered at the point (0, 0, 0). To draw
the sphere centered at (0, 0, 0), first draw a circle of
radius 𝑟, centered at the origin in the 𝑦𝑧 - plane. Then, to give the
surface its three-dimensional look, draw circles of radius 𝑟 centered
at the origin, in both the 𝑥𝑧 - and 𝑥𝑦 – planes
Paraboloid:
The equation of the paraboloid is given by

𝑧 = 𝑥2 + 𝑦2

To get an idea of what the graph looks like, first draw its traces in the three
coordinate planes. In the 𝑦𝑧 - plane, we have 𝑥 = 0 and so 𝑦 2 = 𝑧 (a
parabola). In the 𝑥𝑧 - plane, we have 𝑦 = 0 and so, 𝑥 2 = 𝑧 (a parabola). In
the 𝑥𝑦 - plane, we have 𝑧 = 0 and so, 𝑥 2 + 𝑦 2 = 0 (a point of the origin).
We sketch the traces in first Figure. Finally, since the trace in the 𝑥𝑦 - plane
is just a point, we consider the traces in the planes 𝑧 = 𝑘 (for 𝑘 > 0 ). Notice

- 61 -
Chapter 3 Multiple Integrals
that these are the circles 𝑥 2 + 𝑦 2 = 𝑘 , where for larger values of 𝑧 (i.e., larger
values of 𝑘 ), we get circles of larger radius. We sketch the surface in the second
Figure. Such surfaces are called paraboloids and since the traces in
planes parallel to the 𝑥𝑦 - plane are circles, this is called a circular paraboloid.

Cone:
The equation of the cone is given by

𝑧2 = 𝑥2 + 𝑦2

we start by looking at the traces in the coordinate planes. For the 𝑦𝑧 -


plane, we have 𝑥 = 0, so that 𝑦 = ± 𝑧. That is, the trace is a pair
of lines: 𝑦 = 𝑧 and 𝑦 = −𝑧. We show these in the first Figure.
Likewise, the trace in the 𝑥𝑧 - plane is a pair of lines: 𝑥 = ±𝑧. The trace
in the 𝑥𝑦 - plane is simply the origin. Finally, the traces in the
planes 𝑧 = 𝑘 ( 𝑘 ≠ 0 ), parallel to the 𝑥𝑦 - plane are the
circles: 𝑥 2 + 𝑦 2 = 𝑘. Adding these to the drawing gives us the
double-cone seen in the second Figure.

Since the traces in planes parallel to the 𝑥𝑦 - plane are ellipses, we


refer to this as a circular cone.

- 62 -
Chapter 3 Multiple Integrals

Triple Integrals

Introduction

Just as double integrals allow us to deal with more general situations than could be handled by single
integrals, triple integrals enable us to solve still more general problems. We use triple integrals to calculate
the volumes of three-dimensional shapes and the average value of a function over a three-dimensional
region. Triple integrals also arise in the study of vector fields and fluid flow in three dimensions.

The steps leading to the definition of the three-dimensional definite integral or triple integral are quite
similar to the steps leading to the definition of the double integral. Obvious differences: instead of a function
of two variables we are integrating a function 𝐹 of three variables, not over a region 𝑅 in a coordinate plane,
but over a region 𝐷 of 3-space.

1. Let 𝑤 = 𝐹(𝑥, 𝑦, 𝑧) be defined in a closed and bounded


region 𝐷 of space.
2. By means of a three dimensions grid of vertical and
horizontal lines parallel to the coordinate planes, form a
partition 𝑃 of 𝐷 into 𝑛 subregions (boxes) 𝐷𝑘 of volumes
∆𝑉𝑘 that lie entirely in 𝐷.
3. Let ||𝑃|| be the norm of the partition or the length of the
longest diagonal of the 𝐷𝑘 .
4. Choose a sample point (𝑥 ∗ , 𝑦 ∗ , 𝑧 ∗ ) in each subregion 𝐷𝑘 .
5. Form the sum ∑𝑛𝑘=1 𝐹 (𝑥 ∗ , 𝑦 ∗ , 𝑧 ∗ ) ∆𝑉𝑘 .

Definition: The Triple integrals


Let 𝑓 be a function of three variables defined on a closed region 𝐷 of 3-space. Then the triple integral
of 𝑓 over 𝐷 is given by

𝑛 𝑛

∬ 𝐹(𝑥, 𝑦, 𝑧)𝑑𝑉 = lim ∑ 𝐹 (𝑥 ∗ , 𝑦 ∗ , 𝑧 ∗ ) ∆𝑉𝑘 = lim ∑ 𝐹 (𝑥 ∗ , 𝑦 ∗ , 𝑧 ∗ ) ∆𝑉𝑘


||𝑝||→0 𝑛→∞
𝑅 𝑘=1 𝑘=1

- 63 -
Chapter 3 Multiple Integrals
Applications: A list of some of the standard applications of the triple integral follows:

Volume
If 𝐹(𝑥, 𝑦, 𝑧) = 1, then the volume of the solid 𝐷 is given by

𝑉 = ∭ 𝑑𝑉
𝐷
Mass
If 𝐹(𝑥, 𝑦, 𝑧) = 𝜌(𝑥, 𝑦, 𝑧) is the density, then the mass of the solid 𝐷 is given by

𝑚 = ∭ 𝜌(𝑥, 𝑦, 𝑧) 𝑑𝑉
𝐷

Evaluation of Triple Integrals

We evaluate a triple integral by applying a three-dimensional version of Fubini's Theorem to evaluate


it by three repeated single integrations. As with double integrals, there is a geometric procedure for finding
the limits of integration for these single integrals.
To evaluate

∭ 𝐹(𝑥, 𝑦, 𝑧)𝑑𝑉
𝐷

over a region D, integrate first with respect to z, then with respect to y, and finally with respect to x. (You
might choose a different order of integration, but the procedure is similar)

1. Sketch and find the z-limits of integration.

Sketch the region 𝐷 along with its "shadow" 𝑅


(vertical projection) in the 𝑥𝑦-plane. Label the upper and
lower bounding surfaces of 𝐷 and the upper and lower
bounding curves of R.

Draw a line 𝑀 passing through a typical point (𝑥, 𝑦)


in 𝑅 parallel to the 𝑧-axis. As 𝑧 increases, 𝑀 enters 𝐷 at
𝑧 = 𝐺(𝑥, 𝑦) and leaves at 𝑧 = 𝐻(𝑥, 𝑦). These are the
𝑧-limits of integration.

- 64 -
Chapter 3 Multiple Integrals
2. Find the 𝒚 and 𝒙 limits of integration.
Draw a line 𝐿 through (𝑥, 𝑦) parallel to the 𝑦-axis. As 𝑦 increases, 𝐿 enters 𝑅 at 𝑦 = 𝑔(𝑥) and leaves
at 𝑦 = ℎ(𝑥). These are the 𝑦-limits of integration. Choose 𝑥-limits that include all lines through 𝑅 parallel
to the 𝑦-axis (𝑥 = 𝑎 and 𝑥 = 𝑏 in the following figure). These are the 𝑥-limits of integration.

The integral is

𝑥=𝑏 𝑦=ℎ(𝑥) 𝑧=𝐻(𝑥,𝑦)

∫ ∫ ∫ 𝐹(𝑥, 𝑦, 𝑧) 𝑑𝑧𝑑𝑦𝑑𝑥.
𝑥=𝑎 𝑦=𝑔(𝑥) 𝑧=𝐺(𝑥,𝑦)

Follow similar procedures if you change the order of integration. The "shadow" of region D lies in the
plane of the last two variables with respect to which the iterated integration takes place.

Integration in Other Orders

In previous formula for integrating over a simple 𝑥𝑦-solid, the 𝑧-integration was performed first.
However, there are situations in which it is preferable to integrate in a different order. For example, the
following figures show a simple 𝒙𝒛-solid and a simple 𝒚𝒛-solid. For a simple 𝑥𝑧-solid it is usually best to
integrate with respect to 𝑦 first, and for a simple 𝑦𝑧-solid it is usually best to integrate with respect to 𝑥
first:
𝑦=𝑔2 (𝑥,𝑧)

(𝑎) ∭ 𝐹(𝑥, 𝑦, 𝑧) 𝑑𝑧𝑑𝑦𝑑𝑥 = ∬ [ ∫ 𝐹(𝑥, 𝑦, 𝑧)𝑑𝑦] 𝑑𝐴.


𝐺 𝑅 𝑦=𝑔1 (𝑥,𝑧)

𝑥=𝑔2 (𝑦,𝑧)

(𝑏) ∭ 𝐹(𝑥, 𝑦, 𝑧) 𝑑𝑧𝑑𝑦𝑑𝑥 = ∬ [ ∫ 𝐹(𝑥, 𝑦, 𝑧)𝑑𝑥 ] 𝑑𝐴.


𝐺 𝑅 𝑥=𝑔1 (𝑦,𝑧)

- 65 -
Chapter 3 Multiple Integrals

Example 14 Evaluate ∭𝑫 𝟔𝒙𝒚 𝒅𝑽, where 𝑫 is the tetrahedron bounded by the planes 𝒙 = 𝟎, 𝒚 =
𝟎, 𝒛 = 𝟎 and 𝟐𝒙 + 𝒚 + 𝒛 = 𝟒
Solution:
Notice that each point in the solid lies above the triangular region
𝑅 in the 𝑥𝑦-plane indicated in the two Figures. You can think of 𝑅
as forming the base of the solid. Notice that for each fixed point
(𝑥, 𝑦) ∈ 𝑅, 𝑧 ranges from 𝑧 = 0 up to 𝑧 = 4 − 2𝑥 − 𝑦.
It helps to draw a vertical line from the base and through the top
surface of the solid. The line first enters the solid on the 𝑥𝑦-plane
(𝑧 = 0) and exits the solid on the plane 𝑧 = 4 − 2𝑥 − 𝑦. This
tells you that the innermost limits of integration (given that the first
integration is with respect to 𝑧) are 𝑧 = 0 and 𝑧 = 4 − 2𝑥 − 𝑦.
We now have

𝑧=4−2𝑥−𝑦

∭ 6𝑥𝑦𝑑𝑉 = ∬ ∫ 6𝑥𝑦 𝑑𝑧 𝑑𝐴
𝐷 𝑅 𝑧=0

𝑧=4−2𝑥−𝑦
= ∬(6𝑥𝑦𝑧)𝑧=0 𝑑𝐴 = ∬ 6𝑥𝑦(4 − 2𝑥 − 𝑦) 𝑑𝐴
𝑅 𝑅

= ∬(24𝑥𝑦 − 12𝑥 2 𝑦 − 6𝑥𝑦 2 ) 𝑑𝐴


𝑅
𝑥=2 𝑦=4−2𝑥

= ∫ ∫ (24𝑥𝑦 − 12𝑥 2 𝑦 − 6𝑥𝑦 2 ) 𝑑𝑦𝑑𝑥


𝑥=0 𝑦=0
𝑥=2
𝑦=4−2𝑥
= ∫ (12𝑥𝑦 2 − 6𝑥 2 𝑦 2 − 2𝑥𝑦 3 )𝑦=0 𝑑𝑥
𝑥=0
𝑥=2
64
= ∫ (12𝑥(4 − 2𝑥)2 − 6𝑥 2 (4 − 2𝑥)2 − 2𝑥(4 − 2𝑥)3 )𝑑𝑥 =
5
𝑥=0

- 66 -
Chapter 3 Multiple Integrals
Example 15 Find the mass of the solid of constant mass density 𝝆 bounded by the graphs of the right
circular cone 𝒛 = √𝒙𝟐 + 𝒚𝟐 and the plane 𝒛 = 𝟒.
Solution:
Notice that the projection 𝑅 of the solid onto the 𝑥𝑦-plane is the
disk of radius 4 centered at the origin. Further, for each (𝑥, 𝑦) ∈
𝑅, 𝑧 ranges from the cone (𝑧 = √𝑥 2 + 𝑦 2 ) up to the plane 𝑧 =
4. The total mass of the solid is given by

𝑧=4

𝑚 = ∭ 𝜌 𝑑𝑉 = 𝜌 ∬ ∫ 𝑑𝑧 𝑑𝐴
𝐷 𝑅 𝑧=√𝑥 2 +𝑦 2

𝑧=4 2 2
= 𝜌 ∬(𝑧)𝑧=√𝑥 2 +𝑦 2 𝑑𝐴 = 𝜌 ∬(4 − √𝑥 + 𝑦 ) 𝑑𝐴
𝑅 𝑅
𝜃=2𝜋 𝑟=4 𝜃=2𝜋 𝑟=4
𝑟3
2
= 𝜌∫ ∫ (4 − 𝑟)𝑟𝑑𝑟𝑑𝜃 = 𝜌 ∫ (2𝑟 − ) 𝑑𝜃
𝜃=0 𝑟=0 𝜃=0 3 𝑟=0
64 64
= 𝜌 (32 − ) (2𝜋) = 𝜌𝜋.
3 3
Example 16 Find the volume of the region cut from the cylinder 𝒙𝟐 + 𝒚𝟐 = 𝟒 by the plane 𝒛 = 𝟎
and the plane 𝒙 + 𝒛 = 𝟑.
Solution:
Notice that the projection 𝑅 of the solid onto the 𝑥𝑦-plane is the disk of
radius 2 centered at the origin. Further, for each (𝑥, 𝑦) ∈ 𝑅, 𝑧 ranges
from the plane 𝑧 = 0 up to the plane 𝑧 = 3 − 𝑥. The volume of the solid
is given by
𝑧=3−𝑥
𝑉 = ∭𝐷 1 𝑑𝑉 = ∬𝑅 ∫𝑧=0 𝑑𝑧 𝑑𝐴

𝑧=3−𝑥
= ∬(𝑧)𝑧=0 𝑑𝐴 = ∬(3 − 𝑥) 𝑑𝐴
𝑅 𝑅
𝜃=2𝜋 𝑟=2
=∫ ∫ (3 − 𝑟 cos 𝜃) 𝑟𝑑𝑟𝑑𝜃
𝜃=0 𝑟=0
𝜃=2𝜋 𝑟=2
3 2 𝑟3
=∫ ( 𝑟 − cos 𝜃) 𝑑𝜃
𝜃=0 2 3 𝑟=0
𝜃=2𝜋 𝜃=2𝜋
8 8
=∫ (6 − cos 𝜃) 𝑑𝜃 = (6𝜃 − sin 𝜃) = 12𝜋.
𝜃=0 3 3 𝜃=0

Example 17 Find the volume of the solid enclosed between the paraboloids 𝒛 = 𝒙𝟐 + 𝒚𝟐 and 𝒛 = 𝟖 −
𝒙𝟐 − 𝒚𝟐 .
Solution:
Notice that the projection 𝑅 of the solid onto the 𝑥𝑦-plane is the disk of radius 2 centered at the origin.
Further, for each (𝑥, 𝑦) ∈ 𝑅, 𝑧 ranges from the paraboloid 𝑧 = 𝑥 2 + 𝑦 2 up to the paraboloid𝑧 = 8 − 𝑥 2 −
𝑦 2 . The volume of the solid is given by
𝑧=8−(𝑥 2 +𝑦 2 )
𝑧=8−(𝑥 2 +𝑦 2 )
𝑉 = ∭ 1 𝑑𝑉 = ∬ ∫ 𝑑𝑧 𝑑𝐴 = ∬(𝑧)𝑧=𝑥 2 +𝑦2 𝑑𝐴
𝐷 𝑅 𝑧=𝑥 2 +𝑦 2 𝑅
- 67 -
Chapter 3 Multiple Integrals

= ∬(8 − 2(𝑥 2 + 𝑦 2 )) 𝑑𝐴
𝑅
𝜃=2𝜋 𝑟=2
=∫ ∫ (8 − 2𝑟 2 ) 𝑟𝑑𝑟𝑑𝜃
𝜃=0 𝑟=0
𝜃=2𝜋 𝑟=2
=∫ ∫ (8𝑟 − 2𝑟 3 ) 𝑑𝑟𝑑𝜃
𝜃=0 𝑟=0
𝜃=2𝜋
2
1 4 𝑟=2
=∫ (4𝑟 − 𝑟 ) 𝑑𝜃
𝜃=0 2 𝑟=0
𝜃=2𝜋
=∫ 8 𝑑𝜃 = (8𝜃)𝜃=2𝜋
𝜃=0 = 16𝜋.
𝜃=0

Example 18 Find the volume of the solid bounded by the


parabolic cylinder 𝒚 = 𝒙𝟐 and the planes 𝒚 + 𝒛 =
𝟏, 𝒛 = 𝟎 shown in the following figure.
Solution:
𝑧=1−𝑦
𝑧=1−𝑦
𝑉 = ∭ 1 𝑑𝑉 = ∬ ∫ 𝑑𝑧 𝑑𝐴 = ∬[𝑧]𝑧=0 𝑑𝐴
𝐷 𝑅 𝑧=0 𝑅

𝑥=1 𝑦=1
= ∬(1 − 𝑦) 𝑑𝐴 = ∫ ∫ (1 − 𝑦) 𝑑𝑦𝑑𝑥
𝑥=−1 𝑦=𝑥 2
𝑅
𝑥=1 𝑦=1
𝑥=1 𝑥=1
𝑦2 1 𝑥4 1 𝑥4
=∫ [𝑦 − ] 𝑑𝑥 = ∫ [1 − − 𝑥 2 + ] 𝑑𝑥 = ∫ [ − 𝑥 2 + ] 𝑑𝑥
𝑥=−1 2 𝑦=𝑥 2 𝑥=−1 2 2 𝑥=−1 2 2
1
1 𝑥3 𝑥5 2 1 8
=[ 𝑥− + ] =1− + = .
2 3 10 −1 3 5 15

Triple Integrals in Cylindrical Coordinates

As with two-dimensional space the standard (𝑥, 𝑦, 𝑧) coordinate system is called the Cartesian
coordinate system. We’ll start off with the cylindrical coordinate system. This one is fairly simple as it is
nothing more than an extension of polar coordinates into three dimensions. Not only is it an extension of
polar coordinates, but we extend it into the third dimension just as we extend Cartesian coordinates into the
third dimension. All that we do is add a 𝑧 on as the third coordinate. The 𝑟 and 𝜃 are the same as with
polar coordinates.

- 68 -
Chapter 3 Multiple Integrals
z z

P (x , y , z ) P (r , , z )

z z

y r y
x 
y
x x

Cartesian coordinate system Cylindrical coordinate system

The conversions for 𝑥 and 𝑦 are the same conversions that we used back when we were looking at polar
coordinates. So, if we have a point in cylindrical coordinates the Cartesian coordinates can be found by
using the following conversions.

𝑥 = 𝑟 cos 𝜃 , 𝑦 = 𝑟 sin 𝜃 , 𝑧=𝑧


Likewise, if we have a point in Cartesian coordinates the
cylindrical coordinates can be found by using the following
conversions.
𝑦
𝑟 = √𝑥 2 + 𝑦 2 , 𝜃 = tan−1 , 𝑧 = 𝑧
𝑥

Form the opposite figure, we can write 𝑑𝑉 = 𝑑𝑥𝑑𝑦𝑑𝑧 in terms


of 𝑟, 𝜃 and 𝑧 as

𝑑𝑉 = 𝑑𝑥𝑑𝑦𝑑𝑧 = 𝑑𝑧𝑑𝐴 = 𝑟𝑑𝑧𝑑𝑟𝑑𝜃

Let 𝐺 be a solid region whose upper surface has the equation


𝑧 = 𝑔2 (𝑟, 𝜃) and whose lower surface has the equation 𝑧 =
𝑔1 (𝑟, 𝜃) in cylindrical coordinates. If the projection of the solid on
the 𝑥𝑦 −plane is a simple polar region 𝑅, and if 𝐹(𝑟, 𝜃, 𝑧) is
continuous on 𝐺, then

∭ 𝐹(𝑥, 𝑦, 𝑧) 𝑑𝑧𝑑𝑦𝑑𝑥
𝐺
𝜃=𝜃2 𝑟=𝑟2 (𝜃) 𝑧=𝑔2 (𝑟,𝜃)

= ∫ ∫ ∫ 𝐹(𝑟, 𝜃, 𝑧) 𝑟𝑑𝑧𝑑𝑟𝑑𝜃.
𝜃=𝜃1 𝑟=𝑟1 (𝜃) 𝑧=𝑔1 (𝑟,𝜃)

- 69 -
Chapter 3 Multiple Integrals
Determining Limits of Integration: Cylindrical Coordinates
Step 1. Identify the upper surface 𝑧 = 𝑔2 (𝑟, 𝜃) and the lower surface 𝑧 = 𝑔1 (𝑟, 𝜃) of the solid. The
functions 𝑔1 (𝑟, 𝜃) and 𝑔2 (𝑟, 𝜃) determine the 𝑧-limits of integration. (If the upper and lower
surfaces are given in rectangular coordinates, convert them to cylindrical coordinates.)
Step 2. Make a two-dimensional sketch of the projection 𝑅 of the solid on the 𝑥𝑦-plane. From this sketch
the 𝑟- and 𝜃-limits of integration may be obtained exactly as with double integrals in polar
coordinates.
Example 19 Use triple integration in cylindrical coordinates to find the volume of the solid 𝑮 that is
bounded above by the hemisphere 𝒛 = √𝟐𝟓 − 𝒙𝟐 − 𝒚𝟐 , below by the 𝒙𝒚-plane, and
laterally by the cylinder 𝒙𝟐 + 𝒚𝟐 = 𝟗.
Solution:
The solid 𝐺 and its projection 𝑅 on the 𝑥𝑦-plane are shown in following figure. In cylindrical coordinates,
the upper surface of 𝐺 is the hemisphere 𝑧 = √25 − 𝑟 2 and lower surface is the plane 𝑧 = 0. Thus, the
volume of 𝐺 is
𝑧=√25−𝑟 2

𝑉 = ∭ 1 𝑑𝑧𝑑𝑦𝑑𝑥 = ∬[ ∫ 1 𝑑𝑧] 𝑑𝐴
𝐺 𝑅 𝑧=0

𝑧=√25−𝑟 2
= ∬[𝑧]𝑧=0 𝑑𝐴 = ∬ √25 − 𝑟 2 𝑑𝐴
𝑅 𝑅
𝜃=2𝜋 𝑟=3

= ∫ ∫ √25 − 𝑟 2 𝑟𝑑𝑟𝑑𝜃
𝜃=0 𝑟=0
𝜃=2𝜋 3 𝑟=3
[25 − 𝑟 2 ]2
= ∫ [− ] 𝑑𝜃
3
𝜃=0 𝑟=0
𝜃=2𝜋
61 61 𝜃=2𝜋 61 122
= ∫ 𝑑𝜃 = [𝜃]𝜃=0 = (2𝜋) = 𝜋.
3 3 3 3
𝜃=0
𝟑 √𝟗−𝒙𝟐 𝟗−𝒙𝟐 −𝒚𝟐
Example 20 Use the cylindrical coordinates to evaluate ∫−𝟑 ∫−√𝟗−𝒙𝟐 ∫𝟎 𝒙𝟐 𝒅𝒛𝒅𝒚𝒅𝒙
Solution:
In problems of this type, it is helpful to sketch the region of integration 𝐺
and its projection 𝑅 on the 𝑥𝑦-plane. From the 𝑧-limits of integration, the
upper surface of 𝐺 is the paraboloid 𝑧 = 9 − 𝑥 2 − 𝑦 2 and the lower
surface is the 𝑥𝑦 −plane 𝑧 = 0. From the 𝑥 and 𝑦 −limits of integration,
the projection 𝑅 is the region in the 𝑥𝑦-plane enclosed by the circle 𝑥 2 +
𝑦2 = 9
2 2
3 √9−𝑥 2 9−𝑥 −𝑦

∫ ∫ ∫ 𝑥 2 𝑑𝑧𝑑𝑦𝑑𝑥 = ∭ 𝑥 2 𝑑𝑉
−3 −√9−𝑥 2 0 𝐺

2𝜋 3 9−𝑟 2 2𝜋 3 9−𝑟 2

= ∫ ∫ ∫ 𝑟 2 cos2 𝜃 𝑟𝑑𝑧 𝑑𝑟𝑑𝜃 = ∫ ∫ ∫ 𝑟 3 cos 2 𝜃 𝑑𝑧 𝑑𝑟𝑑𝜃


0 0 0 0 0 0
243
= 𝜋
4
- 70 -
Chapter 3 Multiple Integrals

Triple Integrals in Spherical Coordinates

In the spherical coordinate system, a point 𝑃 in space is represented


by the ordered triple (𝜌, 𝜃, 𝜙), where 𝜌 > 0 is the distance from the origin
to 𝑃, 𝜃 is the same angle as in cylindrical coordinates, and 0 ≤ 𝜙 ≤ 𝜋 is the
angle between the positive 𝑧-axis and the line segment 𝑂𝑃.

The connection between rectangular and


spherical coordinates can be seen in the opposite
figures. If the point 𝑃 has rectangular
coordinates (𝑥, 𝑦, 𝑧) and spherical coordinates
(𝜌, 𝜃, 𝜙), then

𝑥 = 𝜌 sin 𝜙 cos 𝜃,
𝑦 = 𝜌 sin 𝜙 sin 𝜃,
𝑧 = 𝜌 cos 𝜙,

and
𝜌 = √𝑥 2 + 𝑦 2 + 𝑧 2 , 0 ≤ 𝜃 ≤ 2𝜋, 0≤𝜙≤𝜋

Form the opposite figure, we can write 𝑑𝑉 = 𝑑𝑥𝑑𝑦𝑑𝑧 in terms of 𝜌, 𝜃


and 𝜙 as

𝑑𝑉 = 𝑑𝑥𝑑𝑦𝑑𝑧 = 𝜌2 sin 𝜙 𝑑𝜌𝑑𝜙𝑑𝜃

So,

∭ 𝐹(𝑥, 𝑦, 𝑧) 𝑑𝑧𝑑𝑦𝑑𝑥
𝐺
𝜃=𝜃2 𝜙=𝜙1 𝜌=𝜌2

= ∫ ∫ ∫ 𝐹(𝜌 sin 𝜙 cos 𝜃 , 𝜌 sin 𝜙 sin 𝜃 , 𝜌 cos 𝜙) 𝜌2 sin 𝜙 𝑑𝜌𝑑𝜙𝑑𝜃 .


𝜃=𝜃1 𝜙=𝜙1 𝜌=𝜌1

- 71 -
Chapter 3 Multiple Integrals
How the limits of integration in spherical coordinates can be obtained for some other
common solids

- 72 -
Chapter 3 Multiple Integrals

Example 21 Use the spherical coordinates to find the volume of the solid 𝑮 bounded above by the
sphere 𝒙𝟐 + 𝒚𝟐 + 𝒛𝟐 = 𝟏𝟔 and below by the cone 𝒛 = √𝒙𝟐 + 𝒚𝟐 .
Solution:
The solid 𝐺 is sketched in the opposite figure. In spherical coordinates, the
equation of the sphere 𝑥 2 + 𝑦 2 + 𝑧 2 = 16 is 𝜌 = 4 and the equation of the
cone 𝑧 = √𝑥 2 + 𝑦 2 is
𝜌 cos 𝜙 = √(𝜌 sin 𝜙 cos 𝜃)2 + (𝜌 sin 𝜙 sin 𝜃)2
𝜌 cos 𝜙 = 𝜌 sin 𝜙 √cos2 𝜃 + sin2 𝜃 ⇒ 𝜌 cos 𝜙 = 𝜌 sin 𝜙
or
𝜋
cos 𝜙 = sin 𝜙 ⇒ tan 𝜙 = 1 ⇒ 𝜙 =
4
Thus, it follows from the second entry in the previous table that the volume
of 𝐺 is

- 73 -
Chapter 3 Multiple Integrals
𝜋 𝜋
2𝜋 4 4 2𝜋 4 4
𝜌3 2
𝑉 = ∭ 1 𝑑𝑧𝑑𝑦𝑑𝑥 = ∫ ∫ ∫ 𝜌 sin 𝜙 𝑑𝜌𝑑𝜙𝑑𝜃 = ∫ ∫ [ sin 𝜙] 𝑑𝜙𝑑𝜃
3 0
𝐺 0 0 0 0 0
𝜋
2𝜋 4 2𝜋 𝜋 2𝜋
64 64 4 64 1 128𝜋 1
= ∫ ∫ [ sin 𝜙] 𝑑𝜙𝑑𝜃 = ∫ [− cos 𝜙] 𝑑𝜃 = ∫ [1 − ] 𝑑𝜃 = [1 − ].
3 3 0 3 √2 3 √2
0 0 0 0
Example 22 Use the spherical coordinates to evaluate
𝟐 √𝟒−𝒙𝟐 √𝟒−𝒙𝟐 −𝒚𝟐 𝟐
∫−𝟐 ∫−√𝟒−𝒙𝟐 ∫𝟎 𝒛 √𝒙𝟐 + 𝒚𝟐 + 𝒛𝟐 𝒅𝒛𝒅𝒚𝒅𝒙
Solution:
In problems like this, it is helpful to begin (when possible) with a sketch
of the region 𝐺 of integration. From the 𝑧-limits of integration, the upper
surface of 𝐺 is the hemisphere 𝑧 = 4 − 𝑥 2 − 𝑦 2 and the lower surface
is the 𝑥𝑦-plane 𝑧 = 0. From the 𝑥 − and 𝑦 −limits of integration, the
projection of the solid 𝐺 on the 𝑥𝑦-plane is the region enclosed by the
circle 𝑥 2 + 𝑦 2 = 4. From this information we obtain the sketch of 𝐺 in
the opposite figure. Thus,
2 2
2 √4−𝑥 2 √4−𝑥 −𝑦

∫ ∫ ∫ 𝑧 2 √𝑥 2 + 𝑦 2 + 𝑧 2 𝑑𝑧𝑑𝑦𝑑𝑥
−2 −√4−𝑥 2 0
2𝜋 𝜋/2 2

= ∭ 𝑧 2 √𝑥 2 + 𝑦 2 + 𝑧 2 𝑑𝑉 = ∫ ∫ ∫ 𝜌5 cos2 𝜙 sin 𝜙 𝑑𝜌𝑑𝜙𝑑𝜃


𝐺 0 0 0
2𝜋 𝜋/2 2 2𝜋 𝜋/2
𝜌6 32
= ∫ ∫ [ cos 2 𝜙 sin 𝜙] 𝑑𝜌𝑑𝜙𝑑𝜃 = ∫ ∫ cos 2 𝜙 sin 𝜙 𝑑𝜙𝑑𝜃
6 0
3
0 0 0 0
2𝜋 𝜋/2 2𝜋
3
32 −cos 𝜙 32 64
= ∫[ ] 𝑑𝜃 = ∫ 1 𝑑𝜃 = 𝜋.
3 3 0
9 9
0 0

Sheet 6
1. Evaluate the integral ∭𝐸 (𝑥𝑧 − 𝑦 3 ) 𝑑𝑉 , where 𝐸 = {(𝑥, 𝑦, 𝑧)| − 1 ≤ 𝑥 ≤ 1, 0 ≤ 𝑦 ≤ 2, 0 ≤ 𝑧 ≤ 1}
using three different orders of integration.
1
2. Evaluate the integral ∭𝐸 𝑒 −𝑦 𝑑𝑉 , where 𝐸 = {(𝑥, 𝑦, 𝑧)| 0 ≤ 𝑥 ≤ , 0 ≤ 𝑦 ≤ 𝑥𝑧, 1 ≤ 𝑧 ≤ 2} .
𝑧

3. The opposite figure shows the region of integration for the integral
1 1−𝑥 2 1−𝑥

∫ ∫ ∫ 𝑓(𝑥, 𝑦, 𝑧) 𝑑𝑦𝑑𝑧𝑑𝑥 .
0 0 0
Rewrite this integral as an equivalent iterated integral in the
five other orders.
4. Rewrite the following triple integral using the order of integration
𝑑𝑥𝑑𝑧𝑑𝑦, then evaluate it.
1 1−𝑥 1−𝑥−𝑦

∫ ∫ ∫ 1 − 𝑦 − 𝑧 𝑑𝑧𝑑𝑦𝑑𝑥
0 0 0

- 74 -
Chapter 3 Multiple Integrals
2 2 2
5. Evaluate∭𝐸 𝑧 𝑑𝑉 where 𝐸 is the portion of the solid sphere 𝑥 + 𝑦 + 𝑧 ≤ 9 that is inside the cylinder
𝑥 2 + 𝑦 2 = 1 and above the cone 𝑧 = √𝑥 2 + 𝑦 2 .

6. Find the volume of the solid region above the cone 𝑧 = √𝑥 2 + 𝑦 2 and below the sphere 𝑥 2 + 𝑦 2 +
𝑧 2 = 8.
7. Consider the region 𝑅 below the paraboloid 𝑧 = 1 − 𝑥 2 − 𝑦 2 and above the 𝑥𝑦-plane. Compute the
volume of 𝑅.
8. Find the mass of the solid above the paraboloid 𝑧 = 𝑥 2 + 𝑦 2 and below the plane 𝑧 = 4 if the density
function of this solid is 𝜌(𝑥, 𝑦, 𝑧) = 𝑥 2 + 𝑦 2 .

9. Let 𝐸 be the tetrahedron with vertices (0, 0, 0), (1, 0, 0), (0, 1,0) and (0, 0, 2). Setup but do not evaluate
a triple integral that computes the mass of 𝐸 if the density function is 𝜌(𝑥, 𝑦, 𝑧) = 10.

10. Let 𝐸 be the solid above the cone 𝑧 = √𝑥 2 + 𝑦 2 and below the plane 𝑧 = 2. Use spherical coordinates
to computes the volume of 𝐸.

11. Let 𝐸 be the portion of the positive octant which is inside the unit sphere. Use spherical coordinates to
evaluate the integral ∭𝐸 𝑥 + 𝑧 𝑑𝑉 .

12. Let 𝐸 be the solid in the first octant bounded by the sphere 𝑥 2 + 𝑦 2 + 𝑧 2 = 4 and the coordinate
planes. Evaluate

∭ 𝑥𝑦𝑧 𝑑𝑉
𝐸
(i) using rectangular coordinates
(ii) using cylindrical coordinates
(iii)using spherical coordinates.
13. Evaluate

2 √4−𝑥 2 2 𝑎 √𝑎2 −𝑦2 √𝑎2 −𝑥 2 +𝑦 2

(i) ∫ ∫ ∫ 𝑥 2 + 𝑦 2 𝑑𝑧𝑑𝑦𝑑𝑥 (ii) ∫ ∫ ∫ (𝑥 2 𝑧 + 𝑦 2 𝑧 + 𝑧 3 ) 𝑑𝑧𝑑𝑥𝑑𝑦


−2 −√4−𝑥 2 √𝑥 2 +𝑦 2 −𝑎 −√𝑎2 −𝑦 2 −√𝑎2 −𝑥 2 +𝑦2

2 2 2 2
3 √9−𝑥 2 √9−𝑥 −𝑦 2 √4−𝑥 2 √8−𝑥 −𝑦

(iii) ∫ ∫ ∫ √𝑥 2 + 𝑦 2 𝑑𝑧𝑑𝑦𝑑𝑥 (iv) ∫ ∫ ∫ √(𝑥 2 + 𝑦 2 + 𝑧 2 )3 𝑑𝑧𝑑𝑦𝑑𝑥


−3 0 0 −2 0 √𝑥 2 +𝑦 2
2 √4−𝑦 2 2 1 √1−𝑥 2 √1−𝑥 2 −𝑦 2

(v) ∫ ∫ ∫ 𝑥𝑦 𝑑𝑧𝑑𝑥𝑑𝑦 (vi) ∫ ∫ ∫ cos(𝑥 2 + 𝑦 2 + 𝑧 2 )3/2 𝑑𝑧𝑑𝑦𝑑𝑥


−2 −√4−𝑦 2 √𝑥 2 +𝑦2 −1 −√1−𝑥 2 −√1−𝑥 2 −𝑦 2

Homework 6:
𝑧
1. Evaluate ∭𝐺 cos ( ) 𝑑𝑉 , where 𝐺 is the solid defined by the inequalities
𝑦
𝜋 𝜋 𝜋
2. ≤𝑦≤ , 𝑦≤𝑥≤ , 0 ≤ 𝑧 ≤ 𝑥𝑦.
6 2 6

- 75 -
Chapter 3 Multiple Integrals
3. Evaluate ∭𝐺 𝑧 𝑑𝑉 , where 𝐺 is the tetrahedron bounded by the four planes 𝑥 = 0, 𝑦 = 0, 𝑧 = 0 and 𝑥 +
𝑦 + 𝑧 = 1.
4. Use the triple integral to find the volume of the solid within the cylinder 𝑥 2 + 𝑦 2 = 9 and between the
planes 𝑧 = 1 and 𝑥 + 𝑧 = 6.
5. Use the triple integral to find the volume of the tetrahedron bounded by the planes 𝑥 = 2𝑦, 𝑥 = 0, 𝑥 +
2𝑦 + 𝑧 = 2 and 𝑧 = 0.
6. Use the triple integral to find the volume of the solid bounded by the cylinder 𝑦 = 𝑥 2 and the planes
𝑧 = 0 and 𝑦 + 𝑧 = 4.
7. Use cylindrical or spherical coordinates to evaluate the following integrals
2 2
1 √1−𝑥 2 1−𝑥 −𝑦 2 √4−𝑥 2 2
2
(i) ∫ ∫ ∫ 𝑥 𝑑𝑧𝑑𝑦𝑑𝑥 (ii) ∫ ∫ ∫ (𝑥 2 + 𝑦 2 ) 𝑑𝑧𝑑𝑦𝑑𝑥
0 0 0 −2 −√4−𝑥 2 √𝑥 2 +𝑦2

2 2 2 2 2
1 √1−𝑥 2 √1−𝑥 −𝑦 2 √4−𝑦 √8−𝑥 −𝑦
3
−(𝑥 2 +𝑦 2 +𝑧 2 )2
(iii) ∫ ∫ ∫ 𝑒 𝑑𝑧𝑑𝑦𝑑𝑥 (v) ∫ ∫ ∫ 𝑧 2 𝑑𝑧𝑑𝑥𝑑𝑦
−1 0 0 0 0 √𝑥 2 +𝑦 2

3 √9−𝑦 2 √9−𝑥 2 −𝑦 2 2
2 √4−𝑥 2 √8−𝑥 −𝑦
2

(vi) ∫ ∫ ∫ √𝑥 2 + 𝑦 2 + 𝑧 2 𝑑𝑧𝑑𝑥𝑑𝑦 (vii) ∫ ∫ ∫ 𝑧 2 𝑑𝑧𝑑𝑦𝑑𝑥


−3 −√9−𝑦 2 −√9−𝑥 2 −𝑦2 0 0 √𝑥 2 +𝑦2

8. Find the volume enclosed by 𝑥 2 + 𝑦 2 + 𝑧 2 = 𝑎2 using

(a) cylindrical coordinates


(b) spherical coordinates.
9. Find the volume of the solid in the first octant bounded by the sphere 𝜌 = 2, the coordinate planes, and
𝜋 𝜋
the cones 𝜙 = and 𝜙 = .
6 3

10. Use cylindrical coordinates to find the mass of the solid with density 𝜌(𝑥, 𝑦, 𝑧) = 3 − 𝑧 that is bounded
by the cone 𝑧 = √𝑥 2 + 𝑦 2 and the plane 𝑧 = 3.

Challenging Problems:
1. Find the volume and mass of the solid region 𝐸 in the first
octant shown at right if the mass density is 𝜌(𝑥, 𝑦, 𝑧) = 𝑧.

2. Evaluate
2 1 √1−𝑧 2
1
∫ ∫ ∫ 𝑑𝑥𝑑𝑧𝑑𝑦
(1 + 𝑥2 + 𝑧 2 )2
−2 −1 0

3. Evaluate ∭𝐺 𝑧 2 𝑑𝑉 , where 𝐺 is inside 𝑥 2 + 𝑦 2 + 𝑧 2 = 2 and outside 𝑥 2 + 𝑦 2 = 1.

- 76 -
Chapter 3 Multiple Integrals

Surface Integrals

Oriented Surfaces
In order to evaluate a surface integral of a
vector field, we define a smooth surface 𝑆
orientable or two-sided.
The Möbius band in opposite figure is not orientable. No matter
where you start to construct a continuous unit normal field, moving the
vector continuously around the surface in the manner shown will return it
to the starting point with a direction opposite to the one it had when it
started out. The vector at that point cannot point both ways and yet it must
if the field is to be continuous. We conclude that no such field exists.

From now on, we consider only orientable (two-sided). It is possible


to define a field 𝑛̂ of unit normal vectors on 𝑆 that varies continuously
with position. Any patch or sub-portion of an orientable surface is
orientable. Spheres and other smooth closed surfaces in space (smooth
surfaces that enclose solids) are orientable. By convention, we choose 𝑛̂
on a closed surface to point outward.

Once 𝑛̂ has been chosen, as in the opposite Figure, we say that we


have oriented the surface, and we call the surface together with its normal
field an oriented surface. The vector 𝑛̂ at any point is called the positive
direction at that point.

A surface 𝑆 defined by 𝑧 = 𝑓(𝑥, 𝑦) has an upward orientation when the unit normals are directed
upward-that is, have positive 𝑘 components, and it has a downward orientation when the unit normals are
directed downward-that is, have negative 𝑘 components.

Upward orientation Downward orientation

- 77 -
Chapter 3 Multiple Integrals
If a smooth surface 𝑆 is defined by 𝑔(𝑥, 𝑦, 𝑧) = 0, then recall that a unit normal is
1
𝑛̂ = ∇𝑔
||∇𝑔||
𝜕𝑔 𝜕𝑔 𝜕𝑔
where ∇𝑔 = 𝐢+ 𝐣+ 𝐤, the gradient of 𝑔. If 𝑆 is defined by 𝑧 = 𝑓(𝑥, 𝑦), then we can use
𝜕𝑥 𝜕𝑦 𝜕𝑧
𝑔(𝑥, 𝑦 , 𝑧 ) = 𝑧 − 𝑓(𝑥, 𝑦) = 0 or 𝑔(𝑥, 𝑦 , 𝑧 ) = 𝑓(𝑥, 𝑦 ) − 𝑧 = 0 depending on the orientation of 𝑆.

1 < −𝑓𝑥 , −𝑓𝑦 , 1 >


Upward orientation: 𝑛̂1 = ∇𝑔 = ,
||∇𝑔|| 2
√(𝑓𝑥 )2 + (𝑓𝑦 ) + 1

1 < 𝑓𝑥 , 𝑓𝑦 , −1 >
Downward orientation: 𝑛̂2 = ∇𝑔 =
||∇𝑔|| 2
√(𝑓𝑥 )2 + (𝑓𝑦 ) + 1
As we shall see in the next example, the two orientations of an orientable closed surface are outward and
inward.

Example 23 For the paraboloid 𝒛 = 𝒙𝟐 + 𝒚𝟐 , find the two orientation.


Solution:

If we define 𝑔(𝑥, 𝑦, 𝑧) = 𝑧 − 𝑥 2 − 𝑦 2 = 0, then


𝛻 𝑔 = −2𝑥 𝐢 − 2𝑦 𝐣 + 1 𝐤 and ‖𝛻 𝑔‖ = √4𝑥 2 + 4𝑦 2 + 1

Then the two orientations of the surface are


2𝑥 2𝑦 1
𝑛̂ = − 𝐢 − 𝐣 + 𝐤
√4𝑥 2 + 4𝑦 2 + 1 √4𝑥 2 + 4𝑦 2 + 1 √4𝑥 2 + 4𝑦 2 + 1
and
2𝑥 2𝑦 1
−𝑛̂ = 𝐢+ 𝐣− 𝐤
√4𝑥 + 4𝑦 + 1
2 2
√4𝑥 + 4𝑦 + 1
2 2
√4𝑥 + 4𝑦 2 + 1
2

The vector field 𝑛 defines an outward orientation, whereas − 𝑛 defines an inward orientation.

Another solution:
If we define 𝑧 = 𝑓(𝑥, 𝑦) = 𝑥 2 + 𝑦 2 , then the two orientations of the surface are

< −𝑓𝑥 , −𝑓𝑦 , 1 > < −2𝑥, −2𝑦, 1 >


𝑛̂1 = =
2
√(𝑓𝑥 )2 + (𝑓𝑦 ) + 1 √4𝑥 2 + 4𝑦 2 + 1

and
< 𝑓𝑥 , 𝑓𝑦 , −1 > < 2𝑥, 2𝑦, −1 >
𝑛̂2 = =
2
√(𝑓𝑥 )2 + (𝑓𝑦 ) + 1 √4𝑥 2 + 4𝑦 2 + 1

- 78 -
Chapter 3 Multiple Integrals

The relation between 𝒅𝑺 and 𝒅𝑨


Let the normal vector 𝑁 ⃑ to 𝑆 have the angles 𝛼, 𝛽 and 𝛾
with the positive directions of the coordinate axes. From the
opposite Figure, we get
𝑑𝐴 = 𝑑𝑠 cos 𝛾

𝑁
The direction cosine of the unit normal vector 𝑛̂ = ⃑ ||
is
||𝑁
obtained using the
1 〈−𝑓𝑥 , −𝑓𝑦 , 1〉
𝑛̂ = ∇𝑔 =
‖∇𝑔‖ 2
√(𝑓𝑥 )2 + (𝑓𝑦 ) + 1

and
〈−𝑓𝑥 , −𝑓𝑦 , 1〉 1 1
cos 𝛾 = 𝑛̂ ∙ 𝐤 = ∙ 〈0, 0, 1〉 = =
2 2 ⃑‖
‖𝑁
√(𝑓𝑥 )2 + (𝑓𝑦 ) + 1 √(𝑓𝑥 )2 + (𝑓𝑦 ) + 1

So,
⃑ ‖𝑑𝐴
𝑑𝑆 = ‖𝑁

Surface Integrals of a Scalar Field

The surface integral of a function 𝑔(𝑥, 𝑦, 𝑧) over a surface 𝑆 of the form 𝑧 = 𝑓(𝑥, 𝑦) written
∬𝑆 𝑔(𝑥, 𝑦, 𝑧)𝑑𝑆, is given by

⃑ ‖ 𝑑𝐴 = ∬ 𝑔(𝑥, 𝑦, 𝑓(𝑥, 𝑦))√(𝑓𝑥 )2 + (𝑓𝑦 )2 + 1 𝑑𝐴


∬ 𝑔(𝑥, 𝑦, 𝑧)𝑑𝑆 = ∬ 𝑔(𝑥, 𝑦, 𝑓(𝑥, 𝑦))‖𝑁
𝑆 𝑅 𝑅

Area for Surfaces of the Form 𝒛 = 𝒇 (𝒙, 𝒚)


The surface area of a part of the surface in the form 𝑧 = 𝑓 (𝑥, 𝑦) is given by

⃑ ‖ 𝑑𝐴 = ∬ √(𝑓𝑥 )2 + (𝑓𝑦 )2 + 1 𝑑𝐴
∬ 1 𝑑𝑆 = ∬‖𝑁
𝑆 𝑅 𝑅

Mass for Surfaces of the Form 𝒛 = 𝒇 (𝒙, 𝒚)

Let 𝑧 = 𝑓(𝑥, 𝑦) define a surface in 𝑥𝑦𝑧 space above a region 𝑅 in the 𝑥𝑦 plane. Suppose that the
density per unit area of the surface is given by the function 𝜌(𝑥, 𝑦, 𝑧). Then, the mass of the surface is given
by

2
⃑ ‖ 𝑑𝐴 = ∬ 𝜌(𝑥, 𝑦, 𝑓(𝑥, 𝑦)) √(𝑓𝑥 )2 + (𝑓𝑦 ) + 1 𝑑𝐴.
∬ 𝜌(𝑥, 𝑦, 𝑧) 𝑑𝑆 = ∬ 𝜌(𝑥, 𝑦, 𝑓(𝑥, 𝑦)) ‖𝑁
𝑆 𝑅 𝑅

- 79 -
Chapter 3 Multiple Integrals

Example 24 Compute the surface integral ∬𝑺 (𝒙𝒚 + 𝒛) 𝒅𝑺 where S is that part of the plane 𝒙 +
𝒚 + 𝒛 = 𝟐 in the first octant.
Solution:
The surface can be written in the form 𝑧 = 2 − 𝑥 − 𝑦. So, the normal vector is
⃑ = 〈−𝑓𝑥 , −𝑓𝑦 , 1〉 = 〈1, 1, 1〉
𝑁
⃑ ‖ = √3. We can write 𝑑𝑆 as
The magnitude of the normal vector is ‖𝑁
⃑ ‖𝑑𝐴 = √3𝑑𝐴
𝑑𝑆 = ‖𝑁

∬(𝑥𝑦 + 𝑧)𝑑𝑆 = ∬(𝑥𝑦 + 2 − 𝑥 − 𝑦)√3 𝑑𝐴


𝑆 𝑅
𝑥=2 𝑦=2−𝑥

= √3 ∫ ∫ (𝑥𝑦 + 2 − 𝑥 − 𝑦)𝑑𝑦𝑑𝑥
𝑥=0 𝑦=0
𝑥=2
1 2 1 2 𝑦=2−𝑥
= √3 ∫ [ 𝑥𝑦 + 2𝑦 − 𝑥𝑦 − 𝑦 ] 𝑑𝑥
2 2 𝑦=0
𝑥=0
𝑥=2
𝑦=2−𝑥
1 2
1 2
= √3 ∫ [ 𝑥(2 − 𝑥) + 2(2 − 𝑥) − 𝑥(2 − 𝑥) − (2 − 𝑥) ] 𝑑𝑥
2 2 𝑦=0
𝑥=0
𝑥=2
2
3𝑥 2 𝑥 3 1 1
= √3 ∫ [2 − + ] 𝑑𝑥 = √3 [2𝑥 − 𝑥 3 + 𝑥 4 ] = 2√3
2 2 2 8 0
𝑥=0

Example 25 Compute the surface area of the portion of the paraboloid 𝒛 = 𝒙𝟐 + 𝒚𝟐 below the plane
𝒛 = 𝟏.
Solution:
The surface can be written in the form 𝑧 = 𝑥 2 + 𝑦 2 . So, the normal
vector is
⃑ = 〈−𝑓𝑥 , −𝑓𝑦 , 1〉 = 〈−2𝑥, −2𝑦, 1〉
𝑁
⃑ ‖ = √4𝑥 2 + 4𝑦 2 + 1.
The magnitude of the normal vector is ‖𝑁
We can write 𝑑𝑆 as

⃑ ‖𝑑𝐴 = √4𝑥 2 + 4𝑦 2 + 1𝑑𝐴


𝑑𝑆 = ‖𝑁

Surface Area = ∬ 1𝑑𝑆 = ∬ √4𝑥 2 + 4𝑦 2 + 1 𝑑𝐴


𝑆 𝑅
𝜃=2𝜋 𝑟=1 𝜃=2𝜋 𝜃=2𝜋
1 3 𝑟=1 1
2
= ∫ ∫ √4𝑟 + 1 𝑟𝑑𝑟𝑑𝜃 = ∫ [ (4𝑟 + 1)2 ]
2 𝑑𝜃 = (5√5 − 1) ∫ 𝑑𝜃
12 𝑟=0 12
𝜃=0 𝑟=0 𝜃=0 𝜃=0
𝜋
= (5√5 − 1).
6

- 80 -
Chapter 3 Multiple Integrals
𝟐 𝟐
Example 26 Compute the mass of the portion of the paraboloid 𝒛 = 𝟒 − 𝒙 − 𝒚 laying above the 𝒙𝒚
plane if the density per unit area is 𝝆(𝒙, 𝒚, 𝒛) = 𝟑
Solution:
The surface can be written in the form 𝑧 = 4 − 𝑥 2 − 𝑦 2 . So, the
normal vector is
⃑ = 〈−𝑓𝑥 , −𝑓𝑦 , 1〉 = 〈2𝑥, 2𝑦, 1〉
𝑁
⃑ ‖ = √4𝑥 2 + 4𝑦 2 + 1.
The magnitude of the normal vector is ‖𝑁
We can write 𝑑𝑆 as
𝑑𝑆 = ‖𝑁⃑ ‖𝑑𝐴 = √4𝑥 2 + 4𝑦 2 + 1𝑑𝐴

Mass = ∬ 𝜌(𝑥, 𝑦, 𝑧)𝑑𝑆 = ∬ 3 √4𝑥 2 + 4𝑦 2 + 1 𝑑𝐴


𝑆 𝑅
𝜃=2𝜋 𝑟=2 𝜃=2𝜋 𝜃=2𝜋
1 3 𝑟=2 1
2
=3 ∫ ∫ √4𝑟 + 1 𝑟𝑑𝑟𝑑𝜃 = 3 ∫ [ (4𝑟 + 1)2 ]
2 𝑑𝜃 = (−1 + 17√17) ∫ 𝑑𝜃
12 𝑟=0 4
𝜃=0 𝑟=0 𝜃=0 𝜃=0
𝜋
= (−1 + 17√17).
2

Surface Integrals of a vector field


Flux
If 𝐹 (𝑥, 𝑦, 𝑧) = 𝑃(𝑥, 𝑦, 𝑧) 𝐢 + 𝑄(𝑥, 𝑦, 𝑧) 𝐣 + 𝑅(𝑥, 𝑦, 𝑧)𝐤
is the velocity field of a fluid, then, as we saw in opposite
figure, the volume of the fluid flowing through an element of
surface area ∆𝑆 per unit time is approximated by

(height)×(area of base) = (𝐹 ∙ 𝑛̂) ∆𝑆,

where 𝑛̂ is a unit normal to the surface. The total volume of a


fluid passing through 𝑆 per unit time is called the flux of 𝐹
through 𝑆 and is given by

flux = ∬(𝐹 ∙ 𝑛̂) 𝑑𝑆


𝑆

The flux is given by


𝑁
⃑ ‖𝑑𝐴 = ∬ (𝐹 ∙
flux = ∬(𝐹 ∙ 𝑛̂) 𝑑𝑆 = ∬(𝐹 ∙ 𝑛̂) ‖𝑁 ⃑ ‖ 𝑑𝐴 = ∬(𝐹 ∙ 𝑁
) ‖𝑁 ⃑ ) 𝑑𝐴
⃑‖
‖𝑁
𝑆 𝑅 𝑅 𝑅

and
⃑ = 〈−𝑓𝑥 , −𝑓𝑦 , 1〉
𝑁 (Surface oriented upward)
⃑ = 〈𝑓𝑥 , 𝑓𝑦 , −1〉
𝑁 (Surface oriented downward)

- 81 -
Chapter 3 Multiple Integrals
Example 27 Compute the flux of the vector field ⃑𝑭(𝒙, 𝒚, 𝒛) = 〈𝒙, 𝒚, 𝟎〉 over the portion of the
paraboloid 𝒛 = 𝒙𝟐 + 𝒚𝟐 below 𝒛 = 𝟒 (oriented with upward-pointing normal
vectors).
Solution:
First, observe that at any given point, the normal vectors for the
paraboloid 𝑧 = 𝑥 2 + 𝑦 2 are ± 〈2𝑥, 2𝑦, −1〉. For the normal
vector to point upward, we need a positive 𝑧-component. In this
case,
⃑ = 〈−𝑓𝑥 , −𝑓𝑦 , 1〉 = 〈−2𝑥, −2𝑦, 1〉
𝑁
is such a normal vector.

⃑ 𝑑𝐴 = ∬〈𝑥, 𝑦, 0〉 ∙ 〈−2𝑥, −2𝑦, 1〉 𝑑𝐴


flux = ∬ 𝐹 ∙ 𝑁
𝑅 𝑅
𝜃=2𝜋 𝑟=2
2 2 )𝑑𝐴
= ∬(−2𝑥 − 2𝑦 = −2 ∫ ∫ (𝑟 2 cos2 𝜃 + 𝑟 2 sin2 𝜃)𝑟𝑑𝑟𝑑𝜃
𝑅 𝜃=0 𝑟=0
𝜃=2𝜋 𝑟=2

= −2 ∫ ∫ 𝑟 3 𝑑𝑟𝑑𝜃 = −16𝜋
𝜃=0 𝑟=0

Example 28 Determine the flux of the vector field ⃑𝑭(𝒙, 𝒚, 𝒛) = 〈𝟎, −𝟏, −𝟐〉 over the portion of the
plane 𝒛 = 𝟔 − 𝒙 − 𝒚 in the first octant (downward orientation).
Solution: z
First, observe that at any given point, the normal vectors for the plane (0, 0, 6)
𝑧 = 6 − 𝑥 − 𝑦 are ± 〈−1, −1, −1〉. For the normal vector to point
downward, we need a negative 𝑧-component. In this case,
⃑ = 〈𝑓𝑥 , 𝑓𝑦 , −1〉 = 〈−1, −1, −1〉
𝑁
y
is such a normal vector.
(0, 6, 0)
(0, 0, 6)
⃑ 𝑑𝐴 = ∬〈0, −1, −2〉 ∙ 〈−1, −1, −1〉𝑑𝐴
flux = ∬ 𝐹 ∙ 𝑁
𝑅 𝑅
x

1 3
= ∬(3) 𝑑𝐴 = 3 × × (base) × (height) = × 6 × 6 = 54.
2 2
𝑅

Example 29 Determine the flux of the vector field ⃑𝑭(𝒙, 𝒚, 𝒛) = 〈𝒙, 𝒚, 𝟑〉 over the portion of the plane
𝒛 = 𝟔 − 𝒙 cut from the cylinder 𝒙𝟐 + 𝒚𝟐 = 𝟒 (upward orientation).
Solution:
Notice that the projection 𝑅 of the solid onto the 𝑥𝑦-plane is the disk of radius 2 centered at the
origin. Second, observe that at any given point, the normal vectors for the paraboloid 𝑧 = 6 − 𝑥
are ± < −1, 0, −1 >. For the normal vector to point upward, we need a positive 𝑧-component. In
this case,
⃑ = 〈−𝑓𝑥 , −𝑓𝑦 , 1〉 = 〈1, 0, 1〉
𝑁
is such a normal vector.

⃑ 𝑑𝐴 = ∬〈𝑥, 𝑦, 3〉 ∙ 〈1, 0, 1〉 𝑑𝐴
flux = ∬ 𝐹 ∙ 𝑁
𝑅 𝑅

- 82 -
Chapter 3 Multiple Integrals
𝜃=2𝜋 𝑟=2 𝜃=2𝜋 𝑟=2

= ∬(𝑥 + 3) 𝑑𝐴 = ∫ ∫ (𝑟 cos 𝜃 + 3)𝑟𝑑𝑟𝑑𝜃 = ∫ ∫ (𝑟 2 cos 𝜃 + 3𝑟)𝑑𝑟𝑑𝜃


𝑅 𝜃=0 𝑟=0 𝜃=0 𝑟=0
𝜃=2𝜋 𝑟=2 𝜃=2𝜋
𝑟3 3 8
= ∫ ( cos 𝜃 + 𝑟 2 ) 𝑑𝜃 = ∫ ( cos 𝜃 + 6) 𝑑𝜃 = 12𝜋
3 2 𝑟=0
3
𝜃=0 𝜃=0

Remarks
If the surface 𝑆 is piecewise defined, we express a surface integral
over 𝑆 as the sum of the surface integrals over the various pieces of the
surface. For example, suppose 𝑆 is the orientable piecewise – smooth
closed surface bounded by the paraboloid 𝑧 = 1 − 𝑥 2 − 𝑦 2 (𝑆1 ) and
the plane = 1 (𝑆2 ). Then the flux of a vector field 𝐹 out of the surface 𝑆
is

flux = ∯(𝐹 ∙ 𝑛̂) 𝑑𝑆 = ∬(𝐹 ∙ 𝑛̂) 𝑑𝑆 + ∬(𝐹 ∙ 𝑛̂) 𝑑𝑆


𝑆 𝑆1 𝑆2

where we take 𝑆1 oriented upward and 𝑆2 oriented downward

Gauss Theorem (Divergence Theorem)

Let 𝐸 be a closed and bounded region in 3-space with a


piecewise-smooth boundary 𝑆 that is oriented outward.

Let
𝐹 (𝑥, 𝑦, 𝑧) = 𝑃(𝑥, 𝑦, 𝑧)𝐢 + 𝑄(𝑥, 𝑦, 𝑧)𝐣 + 𝑅(𝑥, 𝑦, 𝑧)𝐤
be a vector field for which 𝑃, 𝑄, and 𝑅 are continuous and have
continuous first partial derivatives in a region of 3-space
containing 𝐸. Then

∯(𝐹 ∙ 𝑛̂) 𝑑𝑆 = ∭ div 𝐹 𝑑𝑉 = ∭ ∇ ∙ 𝐹 𝑑𝑉


𝑆 𝐸 𝐸

Example 30 If ⃑𝑭 = 𝒙𝒚 𝐢 + 𝒚𝟐 𝒛 𝐣 + 𝒛𝟑 𝐤 , evaluate ∯𝑺 (𝑭 ⃑ · 𝒏 )𝒅𝑺, where 𝑺 is the unit cube defined


by 𝟎 ≤ 𝒙 ≤ 𝟏, 𝟎 ≤ 𝒚 ≤ 𝟏, and 𝟎 ≤ 𝒛 ≤ 𝟏.
Solution:
Rather than evaluate six surface integrals, we apply the divergence theorem. Since
div 𝐹 = ∇ · 𝐹 = 𝑦 + 2𝑦𝑧 + 3𝑧 2 , we have from

∯(𝐹 ∙ 𝑛̂) 𝑑𝑆 = ∭( 𝑦 + 2𝑦𝑧 + 3𝑧 2 ) 𝑑𝑉


𝑆 𝐷
1 1 1 1 1
2)
= ∫ ∫ ∫( 𝑦 + 2𝑦𝑧 + 3𝑧 𝑑𝑥 𝑑𝑦 𝑑𝑧 = ∫ ∫( 𝑦 + 2𝑦𝑧 + 3𝑧 2 ) 𝑑𝑦 𝑑𝑧
0 0 0 0 0

- 83 -
Chapter 3 Multiple Integrals
1 𝑦=1 1 𝑧=1
2 2
𝑦 1 1 𝑧
= ∫( + 𝑦 2 𝑧 + 3𝑦𝑧 2 ) 𝑑𝑧 = ∫ ( + 𝑧 + 3𝑧 2 ) 𝑑𝑧 = ( 𝑧 + + 𝑧 3 ) = 2.
2 𝑦=0
2 2 2 𝑧=0
0 0

Example 31 Let 𝑫 be the solid bounded by the sphere 𝒙𝟐 + 𝒚𝟐 + 𝒛𝟐 = 𝟏𝟔. Find the flux of the vector
⃑ (𝒙, 𝒚, 𝒛) = 〈𝒙, 𝒚, 𝒛〉 over the surface 𝑺.
field 𝑭
Solution:
div 𝐹 = ∇ · 𝐹 = 3, we have from
4
∯(𝐹 ∙ 𝑛̂) 𝑑𝑆 = ∭ 3 𝑑𝑉 = 3 × the volume of the sphere = 3 × × 𝜋 × (4)3 = 256𝜋
3
𝑆 𝐷

Example 32 Let 𝑸 be the solid bounded by the paraboloid 𝒛 = 𝟒 − 𝒙𝟐 − 𝒚𝟐 and the 𝒙𝒚-plane. Find
the flux of the vector field ⃑𝑭(𝒙, 𝒚, 𝒛) = 〈𝒙𝟑 , 𝒚𝟑 , 𝒛𝟑 〉 over the surface 𝑺.
Solution:
We show a sketch of the solid in opposite Figure. Notice that to
compute the flux directly, we must consider the two different portions
of 𝑆 (the surface of the paraboloid and its base in the 𝑥𝑦-plane)
separately. Alternatively, observe that the divergence of 𝐹 is given by
𝛻 · 𝐹 (𝑥, 𝑦, 𝑧) = 𝛻 · 〈𝑥 3 , 𝑦 3 , 𝑧 3 〉 = 3𝑥 2 + 3𝑦 2 + 3𝑧 2 .
From the Divergence Theorem, we now have that the flux of 𝐹 over 𝑆
is given by

∯(𝐹 ∙ 𝑛̂) 𝑑𝑆 = ∭(3𝑥 2 + 3𝑦 2 + 3𝑧 2 ) 𝑑𝑉


𝑆 𝐷
𝑧=1−𝑥 2 −𝑦 2

=∬ ∫ (3𝑥 2 + 3𝑦 2 + 3𝑧 2 ) 𝑑𝑧 𝑑𝐴
𝑅 𝑧=0

𝑧=1−𝑥 2 −𝑦 2
= ∬ (3𝑥 2 𝑧 + 3𝑦 2 𝑧 + 𝑧 3 )𝑧=0 𝑑𝐴
𝑅

= ∬ [(3𝑥 2 + 3𝑦 2 )(1 − 𝑥 2 − 𝑦 2 ) + (1 − 𝑥 2 − 𝑦 2 )3 ] 𝑑𝐴
𝑅
𝜃=2𝜋 𝑟=2

= ∫ ∫ [3𝑟 2 (1 − 𝑟 2 ) + (1 − 𝑟 2 )3 ] 𝑟𝑑𝑟𝑑𝜃 = 96𝜋.


𝜃=0 𝑟=0

𝟐
⃑ (𝒙, 𝒚, 𝒛) = 〈𝒙𝒚, 𝒚𝟐 + 𝒆𝒙𝒛 , 𝐬𝐢𝐧 𝒙𝒚〉 over 𝑺 is the
Example 33 Evaluate the flux of the vector field 𝑭
surface of the region 𝑬 bounded by the parabolic cylinder 𝒛 = 𝟏 − 𝒙𝟐 and the planes
𝒛 = 𝟎, 𝒚 = 𝟎 and 𝒚 + 𝒛 = 𝟐.
Solution:
The divergence of 𝐹 is
2
𝛻 · 𝐹 (𝑥, 𝑦, 𝑧) = 𝛻 · 〈𝑥𝑦, 𝑦 2 + 𝑒 𝑥𝑧 , sin 𝑥𝑦〉 = 𝑦 + 2𝑦 = 3𝑦.

- 84 -
Chapter 3 Multiple Integrals
Therefore, we use the Divergence Theorem to transform the
given surface integral into a triple integral. The easiest way to
evaluate the triple integral is to express as
𝐸 = {(𝑥, 𝑦, 𝑧)| − 1 ≤ 𝑥 ≤ 1, 0 ≤ 𝑧 ≤ 1 − 𝑥 2 , 0 ≤ 𝑦 < 2 − 𝑧}

Flux = ∯(𝐹 ∙ 𝑛̂) 𝑑𝑆 = ∭ 3𝑦 𝑑𝑉


𝑆 𝐷
𝑦=2−𝑧

=∬ ∫ 3𝑦 𝑑𝑦 𝑑𝐴
𝑅 𝑦=0
𝑦=2−𝑧
3 3
= ∬ ( 𝑦2) 𝑑𝐴 = ∬ (2 − 𝑧)2 𝑑𝐴
2 𝑦=0 2
𝑅 𝑅
1 1−𝑥 2 1 𝑧=1−𝑥 2 1
3 3 (2 − 𝑧)3 1
= ∫ ∫ (2 − 𝑧)2 𝑑𝑧𝑑𝑥 = ∫ (− ) 𝑑𝑥 = − ∫[(𝑥 2 + 1)3 − 8] 𝑑𝑥
2 2 3 𝑧=0
2
−1 0 −1 −1
184
= .
35

Sheet 7
1. Evaluate the following surface integral
(i) ∬𝑆 𝑥 2 𝑦𝑧 𝑑𝑆 where 𝑆 is that part of the plane 𝑧 = 1 + 2𝑥 + 3𝑦 that lies above the rectangle 0 ≤
𝑥 ≤ 3 and 0 ≤ 𝑦 ≤ 2.
(ii) ∬𝑆 𝑥𝑦 𝑑𝑆 where 𝑆 is the triangular region with vertices (1, 0, 0), (0, 2, 0) and (0, 0, 2).
(iii) ∬𝑆 𝑥 2 𝑦 2 𝑑𝑆 where 𝑆 is the cone 𝑧 = √𝑥 2 + 𝑦 2 that lies between the planes 𝑧 = 1 and 𝑧 = 3.

2. Evaluate the flux of the vector field 𝐹 = 〈𝑦, 𝑥, 𝑧4 〉 through the surface 𝑆 is the part of the cone 𝑧 = √𝑥2 + 𝑦2
beneath the plane 𝑧 = 1 with downward orientation.
3. Evaluate the flux of the water through the portion of the paraboloid 𝑧 = 1 − 𝑥 2 − 𝑦 2 above the plane
𝑧 = 0 if the velocity vector is 𝐹 = 〈𝑦, 𝑥, 𝑧〉 oriented with upward-pointing normal vectors.
4. If the heat flow of the temperature in a metal ball is 𝐹 (𝑥, 𝑦, 𝑧) = −𝐾 ∇𝑇 where 𝑇(𝑥, 𝑦, 𝑧) = 3(𝑥 2 + 𝑦 2 +
𝑧 2 ) is the temperature function and 𝐾 is the conductivity of the metal. Find the rate of heat flow across a
sphere 𝑆 of radius 𝑎 with center at the center of the ball.

5. Find the surface area of that part of the paraboloid 𝑧 = 1 + 𝑥 2 + 𝑦 2 that lies below the plane 𝑧 = 5.
6. Find the surface area of the part of the plane 𝑥 + 3𝑦 + 𝑧 = 6 in the first octant.
7. Find the surface area of the part of the sphere 𝑥 2 + 𝑦 2 + 𝑧 2 = 64 that lies above the cone 𝑧 = √𝑥 2 + 𝑦 2 .
8. Find the surface area of the part of the part of the sphere 𝑥 2 + 𝑦 2 + 𝑧 2 = 4 that lies above the plane 𝑧 = 1.
9. Use the Divergence Theorem to compute ∬𝐸 𝐹 ∙ 𝑛̂ 𝑑𝑆 where 𝑛̂ is the outward unit normal
(i) 𝐸 is bounded by 𝑧 = √𝑥 2 + 𝑦 2 and 𝑧 = 4, 𝐹 = 〈𝑦 3 , 𝑥 + 𝑧 2 , 𝑧 + 𝑦 2 〉.
(ii) 𝐸 is bounded by 𝑧 = 𝑥 2 + 𝑦 2 and 𝑧 = 4, 𝐹 = 〈𝑥 3 , 𝑦 3 − 𝑧, 𝑥𝑦 2 〉.
(iii)𝐸 is bounded by 𝑥 2 + 𝑦 2 = 4 𝑧 = 1 and 𝑧 = 8 − 𝑦, 𝐹 = 〈𝑦 2 𝑧, 2𝑦 − 𝑒 𝑧 , sin 𝑥〉.
(iv) 𝐸 is bounded by 𝑧 = −√4 − 𝑥 2 − 𝑦 2 and 𝑧 = 0, 𝐹 = 〈𝑥 3 , 𝑦 3 , 𝑧 3 〉.
- 85 -
Chapter 3 Multiple Integrals
(v) 𝐸 is bounded by the hemisphere 𝑧 = √1 − 𝑥2 − 𝑦2 and 𝑥𝑦-plane, 𝐹 = (𝑦 + 𝑒 𝑦𝑧 ) 𝐢 +
3
(sin 𝑧 − 𝑥𝑧) 𝐣 + (𝑥 2 + 𝑦 2 + 𝑧 2 )2 𝐤

Homework 7:
1. Compute the surface integral ∬𝑆 𝑥 2 𝑧 𝑑𝑆 where S is that part of the cone 𝑧 = √𝑥 2 + 𝑦 2 that lies between
the two planes 𝑧 = 1 and 𝑧 = 4.
2. Compute the surface integral ∬𝑆 (𝑥 + 𝑦) 𝑑𝑆 where S is that part of the plane 2𝑥 + 3𝑦 + 𝑧 = 3 lying in
the first octant.
3. Find the surface area of the cone 𝑧 = √𝑥 2 + 𝑦 2 below the plane 𝑧 = 1.
4. Find the surface area of the part of the cylinder 𝑥 2 + 𝑧 2 = 4 that lies directly above the rectangle 0 ≤
𝑥 ≤ 1, 0 ≤ 𝑦 ≤ 4 in the 𝑥𝑦 – plane.
5. Find the mass of a thin funnel in the shaped of a paraboloid𝑧 = 𝑥 2 + 𝑦 2 , 1 ≤ 𝑧 ≤ 4 if its density
function 𝜌(𝑥, 𝑦, 𝑧) = 10 − 𝑧.
6. Evaluate the flux of the water through the paraboloid cylinder 𝑦 = 𝑥 2 , 0 ≤ 𝑥 ≤ 2, 0 ≤ 𝑧 ≤ 3 if the
velocity vector is 𝐹 = 〈3𝑧 2 , 6, 6𝑥𝑧〉.
7. Evaluate the flux of the water through the unit sphere lies in the first octant if the velocity vector is 𝐹 =
𝑥 𝐣.
8. Use the divergence Theorem to find the flux of the vector 𝐹 = 〈𝑥 3 , 𝑦 3 , 𝑧 3 〉 across the surface of the
hemisphere 𝑧 = √1 − 𝑥 2 − 𝑦 2 and 𝑧 = 0.
9. Use the divergence Theorem to find the flux of the vector 𝐹 = 〈2𝑥, 3𝑦, 𝑧 2 〉 across the surface of the unit
cube has vertices (0,0,0), (0,0,1), (0,1,0), (1,0,0), (1,1,0), (1,0,1), (0,1,1) and (1,1,1).
2 2
10. Use the divergence Theorem to find the flux of the vector 𝐹 = 〈𝑥 3 + sin 𝑧 , 𝑥 2 𝑦 + cos 𝑧 , 𝑒 𝑥 +𝑦 〉 across
the surface of the paraboloid cylinder 𝑧 = 4 − 𝑥 2 , bounded by the planes 𝑦 + 𝑧 = 5, 𝑥𝑦-plane and 𝑥𝑧-
plane.

Challenging Problems:
1. Let 𝑥, 𝑦 and 𝑧 be measured in meters, and suppose that 𝐹 (𝑥, 𝑦, 𝑧) = 2𝑥 𝐢 − 3𝑦 𝐣 + 𝑧𝐤 is the velocity
vector (in 𝑚/𝑠) of a fluid particle at the point (𝑥, 𝑦, 𝑧) in a steady-state incompressible fluid flow. Find
the net mass of fluid that passes in the upward direction through the portion of the plane 𝑥 + 𝑦 + 𝑧 = 1
in the first octant in 1 sec if the fluid has a mass density of 806 𝑘𝑔/𝑚3 .

2. Let 𝐸 be a solid whose surface 𝑆 is oriented outward. If 𝐹 (𝑥, 𝑦, 𝑧) = 𝑓(𝑥, 𝑦, 𝑧)𝐢 + 𝑔(𝑥, 𝑦, 𝑧)𝐣 +
ℎ(𝑥, 𝑦, 𝑧)𝐤 where 𝑓, 𝑔, and ℎ have continuous first partial derivatives on some open set containing 𝐸,
and if 𝑛̂ is the outward unit normal on 𝑆, prove that

∯(curl 𝐹 ∙ 𝑛̂) 𝑑𝑆 = 0
𝑆
3. Let 𝐸 be a solid whose surface 𝑆 is oriented outward. If 𝐹 (𝑥, 𝑦, 𝑧) is a vector field has continuous first
partial derivatives on some open set containing 𝐸, 𝑓 and 𝑔 have continuous first partial derivatives and
if 𝑛̂ is the outward unit normal on 𝑆, prove that

∯(curl 𝐹 ∙ 𝑛̂) 𝑑𝑆 = ∭(𝑓(∇ ∙ ∇𝑔) + ∇𝑓 ∙ ∇𝑔) 𝑑𝑉


𝑆 𝐷

- 86 -
Chapter 3 Multiple Integrals

Stokes’ Theorem
Introduction
Green's Theorem of the preceding section has two vector forms. In this Section, we shall generalize
these forms to three dimensions.

Vector Form of Green's Theorem


If 𝐹 (𝑥, 𝑦) = 𝐹1 (𝑥, 𝑦) 𝐢 + 𝐹2 (𝑥, 𝑦)𝐣 is a two dimension at vector field, then
𝐢 𝐣 𝐤
𝜕 𝜕 𝜕 𝜕𝐹2 𝜕𝐹2
curl 𝐹 = | |=( − )𝐤
𝜕𝑥 𝜕𝑦 𝜕𝑧 𝜕𝑥 𝜕𝑥
𝐹1 𝐹2 0
Green's theorem can be written in vector notation as
𝜕𝐹2 𝜕𝐹2
∮ 𝐹 ∙ 𝑑𝑟 = ∮ 𝐹1 𝑑𝑥 + 𝐹2 𝑑𝑦 = ∬ ( − ) 𝑑𝐴 = ∬(curl ⃑𝐹) ∙ 𝐤 𝑑𝐴
𝜕𝑥 𝜕𝑥
𝑐 𝑐 𝑅 𝑅

Green's Theorem in 3-Space

The vector form of Green's theorem given in the previous relates a line
integral around a piecewise smooth simple closed curve 𝐶 forming the
boundary of a plane region 𝑅 to a double integral over 𝑅. Green's theorem
in 3-space relates a line integral around a piecewise-smooth simple closed
curve 𝐶 forming the boundary of a surface 𝑆 with a surface integral over
𝑆.

Suppose 𝑧 = 𝑓 (𝑥, 𝑦) is a continuous function whose graph is a piecewise-smooth orientable surface


over a region 𝑅 on the 𝑥𝑦-plane. Let 𝐶 form the boundary of 𝑆 and let the projection of 𝐶 onto the 𝑥𝑦-plane
form the boundary of 𝑅. The positive direction on 𝐶 is induced by the orientation of the surface S; the
positive direction on 𝐶 corresponds to the direction a person would have to walk on 𝐶 to have his or her
head point in the direction of the orientation of the surface while keeping the surface to the left. (See the
preceding figure).

- 87 -
Chapter 3 Multiple Integrals

More precisely, the positive orientation of 𝐶 is in accordance with


the right-hand rule: If the thumb of the right-hand points in the
direction of the orientation of the surface, then roughly the fingers
of the right-hand wrap around the surface in the positive direction.
Finally, let 𝐓 be a unit tangent vector to 𝐶 that points in the positive
direction. The three-dimensional form of Green's theorem, which
we now give, is called Stokes' theorem.

Stoke’s Theorem

Let 𝑆 be a piecewise-smooth orientable surface bounded by a


piecewise-smooth simple closed curve 𝐶. Let

𝐹 (𝑥, 𝑦, 𝑧) = 𝐹1 (𝑥, 𝑦, . 𝑧)𝐢 + 𝐹2 (𝑥, 𝑦, 𝑧)𝐣 + 𝐹3 (𝑥, 𝑦, 𝑧)𝐤

be a vector field for which 𝐹1 , 𝐹2 , and 𝐹3 are continuous and have


continuous first partial derivatives in a region of 3-space containing
𝑆. If 𝐶 is traversed in the positive direction, then

⃑ ) ∙ 𝐧̂ 𝑑𝑆
∮ 𝐹 ∙ 𝑑𝑟 = ∮ 𝐹1 𝑑𝑥 + 𝐹2 𝑑𝑦 + 𝐹3 𝑑𝑧 = ∬(curl 𝐹
𝑐 𝑐 𝑆
̂ is a unit normal to 𝑆 in the direction of the orientation of 𝑆.
where 𝐧

Example 34 Use Stokes’ Theorem to evaluate ∮𝒄 𝒛 𝒅𝒙 + 𝒙 𝒅𝒚 + 𝒚 𝒅𝒛, where 𝑪 is the trace of the
cylinder 𝒙𝟐 + 𝒚𝟐 = 𝟏 in the plane 𝒚 + 𝒛 = 𝟐. Orient 𝑪 counterclockwise as viewed
from above.
Solution:
If 𝐹 = 𝑧 𝐢 + 𝑥 𝐣 + 𝑦 𝐤, then
𝐢 𝐣 𝐤
𝜕 𝜕 𝜕
curl 𝐹 = | |=𝐢+𝐣+𝐤
𝜕𝑥 𝜕𝑦 𝜕𝑧
𝑧 𝑥 𝑦
The given orientation of 𝐶 corresponds to an upward orientation of the
surface 𝑆. Thus, if
𝑧 = 𝑓(𝑥, 𝑦) = 2 − 𝑦,
then the upper normal is
⃑ = 〈−𝑓𝑥 , −𝑓𝑦 , 1〉 = 〈0, 1,1〉.
𝑁
Hence,

̂ 𝑑𝑆 = ∬(curl 𝐹 ) ∙ ⃑N
∮ 𝐹 ∙ 𝑑𝑟 = ∬(curl 𝐹 ) ∙ 𝐧 ⃑ 𝑑𝐴 = ∬[〈1, 1, 1〉 ∙ 〈0, 1,1〉] 𝑑𝐴
𝑐 𝑆 𝑅 𝑅

= ∬ 2 𝑑𝐴 = 2 × area of the circle = 2 × 𝜋 × (1)2 = 2𝜋


𝑅
- 88 -
Chapter 3 Multiple Integrals

Example 35 Use Stokes’ Theorem to evaluate ∮𝒄 𝒚 𝒅𝒙 − 𝒙 𝒅𝒚 + 𝟎 𝒅𝒛, where 𝑪 is the trace of the
hemisphere 𝒙𝟐 + 𝒚𝟐 + 𝒛𝟐 = 𝟗, 𝒛 ≥ 𝟎 and 𝒙𝒚 −plane.
Solution:
If 𝐹 = 𝑦 𝐢 − 𝑥 𝐣 + 0 𝐤, then
𝐢 𝐣 𝐤
𝜕 𝜕 𝜕
curl 𝐹 = | | = 0𝐢 + 0𝐣 − 2𝐤
𝜕𝑥 𝜕𝑦 𝜕𝑧
𝑦 −𝑥 0
The given orientation of 𝐶 corresponds to an upward orientation
of the surface 𝑆. Thus, if
𝑧 = 𝑓(𝑥, 𝑦) = √9 − 𝑥 2 − 𝑦 2 ,
then the upper normal is
2𝑥 2𝑦
⃑ = 〈−𝑓𝑥 , −𝑓𝑦 , 1〉 = 〈
𝑁 , , 1〉 .
√9 − 𝑥 2 − 𝑦 2 √9 − 𝑥 2 − 𝑦 2
Hence,

̂ 𝑑𝑆 = ∬(curl 𝐹 ) ∙ ⃑N
∮ 𝐹 ∙ 𝑑𝑟 = ∬(curl 𝐹 ) ∙ 𝐧 ⃑ 𝑑𝐴 = ∬ −2 𝑑𝐴
𝑐 𝑆 𝑅 𝑅

= ∬ −2 𝑑𝐴 = −2 × area of the circle = −2 × 𝜋 × (3)2 = −18𝜋


𝑅
Remarks

The value of the surface integral in Stokes' Theorem is


determined solely by the integral around its boundary 𝐶.
Then for two different surfaces 𝑆1 and 𝑆2 with the same
orientation and with the same boundary 𝐶, we have

∮ 𝐹 ∙ 𝑑𝑟 = ∬(curl ⃑𝐹) ∙ 𝐧̂ 𝑑𝑆 = ∬(curl ⃑𝐹) ∙ 𝐧̂ 𝑑𝑆


𝑐 𝑆1 𝑆2
𝟐
Example 36 Use Stokes’ Theorem to evaluate ∬𝑺 (𝛁 × ⃑𝑭). 𝐧 ̂ 𝒅𝑺, where ⃑𝑭 = 〈𝒆𝒛 , 𝟒𝒛 − 𝒚, 𝟖𝒙 𝐬𝐢𝐧 𝒚〉
and 𝑺 is the portion of the paraboloid 𝒛 = 𝟒 − 𝒙𝟐 − 𝒚𝟐 above 𝒙𝒚 −plane, oriented so
that the unit normal vector point to the outside of the paraboloid.
Solution:
Notice that the boundary curve is simply the circle 𝑥 2 + 𝑦 2 = 4 lying
in the 𝑥𝑦-plane. Then,
𝑥 = 2 cos 𝑡 , 𝑦 = 2 sin 𝑡 , 𝑧 = 0, 0 ≤ 𝑡 ≤ 2𝜋
𝑟 = 〈2 cos 𝑡 , 2 sin 𝑡 , 0〉, ⃑⃑⃑
𝑟 ′ = 〈−2 sin 𝑡 , 2 cos 𝑡 , 0〉,
𝐹 (𝑟(𝑡)) = 〈1, −2 sin 𝑡 , 16 cos 𝑡 sin(2 sin 𝑡)〉
By Stokes’ Theorem, we then have
2𝜋

̂ 𝑑𝑆 = ∮ 𝐹 ∙ 𝑑𝑟 = ∫ (−2 sin 𝑡 − 4 sin 𝑡 cos 𝑡)𝑑𝑡


∬(curl 𝐹 ) ∙ 𝐧
𝑆 𝑐 0

= 2 cos 𝑡 − 2(sin 𝑡)2 |2𝜋


0 =0

- 89 -
Chapter 3 Multiple Integrals
𝟐 𝟐
Example 37 Use Stokes’ Theorem to evaluate ∮𝒄 (𝒙 − 𝒚)𝒅𝒙 + 𝟒𝒛𝒅𝒚 + 𝒙 𝒅𝒛, where 𝑪 is the trace
of the cone 𝒛 = √𝒙𝟐 + 𝒚𝟐 in the plane 𝒛 = 𝟐. Orient 𝑪 counterclockwise as viewed from
above.
Solution:
If 𝐹 = (𝑥 2 − 𝑦)𝐢 + 4𝑧 𝐣 + 𝑥 2 𝐤, then
𝐢 𝐣 𝐤
𝜕 𝜕 𝜕
curl 𝐹 = || | = −4 𝐢 − 𝟐𝑥 𝐣 + 𝐤
𝜕𝑥 𝜕𝑦 𝜕𝑧|
𝑥 2 − 𝑦 4𝑧 𝑥 2
The given orientation of 𝐶 corresponds to an upward orientation
of the surface 𝑆. Thus, if
𝑧 = 𝑓(𝑥, 𝑦) = 2,
then the upper normal is

⃑ = 〈−𝑓𝑥 , −𝑓𝑦 , 1〉 = 〈0, 0,1〉 .


𝑁
Hence,

⃑⃑ 𝑑𝐴 = ∬ 1 𝑑𝐴
̂ 𝑑𝑆 = ∬(curl 𝐹 ) ∙ N
∮ 𝐹 ∙ 𝑑𝑟 = ∬(curl 𝐹 ) ∙ 𝐧
𝑐 𝑆 𝑅 𝑅

= ∬ 1 𝑑𝐴 = area of the circle = 𝜋 × (2)2 = 4𝜋


𝑅

Example 38 Evaluate ∬𝑺 (𝛁 × ⃑𝑭). 𝐧


̂ 𝒅𝑺, where ⃑𝑭 = 〈𝒙𝒛, 𝒚𝒛, 𝒙𝒚〉 and 𝑺 is the portion of the sphere
𝒙𝟐 + 𝒚𝟐 + 𝒛𝟐 = 𝟒 that lies inside the cylinder 𝒙𝟐 + 𝒚𝟐 = 𝟏 above 𝒙𝒚 −plane, oriented
so that the unit normal vector point to the outside of the sphere.
Solution:
To find the boundary curve 𝐶, we solve the equations 𝑥 2 + 𝑦 2 +
𝑧 2 = 4 and 𝑥 2 + 𝑦 2 = 1. Subtracting, we get 𝑧 2 = 3 and so 𝑧 =
√3 (since 𝑧 ≥ 0). Thus is the circle given by the equations 𝑥 2 +
𝑦 2 = 1, 𝑧 = √3. A vector equation of 𝐶 is

𝑟 = 〈cos 𝑡 , sin 𝑡 , √3〉, ⃑⃑⃑


𝑟 ′ = 〈− sin 𝑡 , cos 𝑡 , 0〉,
𝐹 (𝑟(𝑡)) = 〈√3 cos 𝑡 , √3 sin 𝑡 , cos 𝑡 sin 𝑡〉

By Stokes’ Theorem, we then have


2𝜋

̂ 𝑑𝑆 = ∮ 𝐹 ∙ 𝑑𝑟 = ∫ (−√3 cos 𝑡 sin 𝑡 + √3 sin 𝑡 cos 𝑡)𝑑𝑡


∬(curl 𝐹 ) ∙ 𝐧
𝑆 𝑐 0
2𝜋

= ∫ (0)𝑑𝑡 = 0.
0

- 90 -
Chapter 3 Multiple Integrals
Example 39 Verify Stoke’s Theorem by computing both integrals, if ⃑𝑭 = 𝒙𝒛 𝐢 + 𝒙𝒚 𝐣 + 𝟑𝒙𝒛 𝐤 and
𝑺 is the portion of the plane 𝟐𝒙 + 𝒚 + 𝒛 = 2 in the first octant, oriented upward.
Solution:
Surface Integral:
If 𝐹 = 𝑥𝑧 𝐢 + 𝑥𝑦 𝐣 + 3𝑥𝑧 𝐤, then
𝐢 𝐣 𝐤
𝜕 𝜕 𝜕
curl 𝐹 = | | = 0𝐢 + (𝑥 − 3𝑧)𝐣 + 𝑦𝐤
𝜕𝑥 𝜕𝑦 𝜕𝑧
𝑥𝑧 𝑥𝑦 3𝑥𝑧
= 0𝐢 + (𝑥 − 3(2 − 2𝑥 − 𝑦)) 𝐣 + 𝑦𝐤
= (7𝑥 + 3𝑦 − 6) 𝐣 + 𝑦 𝐤
The given orientation of 𝐶 corresponds to an upward
orientation of the surface 𝑆. Thus, if
𝑧 = 𝑓(𝑥, 𝑦) = 2 − 2𝑥 − 𝑦,
then the upper normal is
⃑ = 〈−𝑓𝑥 , −𝑓𝑦 , 1〉 = 〈2, 1, 1〉 .
𝑁
Hence,

̂ 𝑑𝑆 = ∬(curl 𝐹 ) ∙ ⃑N
∬(curl 𝐹 ) ∙ 𝐧 ⃑ 𝑑𝐴 = ∬(7𝑥 + 4𝑦 − 6) 𝑑𝐴
𝑆 𝑅 𝑅
𝑥=1 𝑦=2−2𝑥

= ∫ ∫ (7𝑥 + 4𝑦 − 6) 𝑑𝐴 = −1
𝑥=0 𝑦=0

Line Integral:
1. Along 𝐶1
The equation of the first path 𝐶1 is 𝑧 = 0, 2𝑥 + 𝑦 = 2
𝑥 = 𝑡, 𝑦 = 2 − 2𝑡, 𝑧 = 0,
𝑟 = 〈𝑡, 2 − 2𝑡, 0〉, ⃑⃑⃑
𝑟 ′ = 〈1, −2, 0〉, 𝐹 (𝑟(𝑡)) = 〈0, 2𝑡 − 2𝑡 2 , 0〉
0
2 )𝑑𝑡 2
4 30 4 2
𝐼1 = ∫ 𝐹 ∙ 𝑑𝑟 = ∫(−4𝑡 + 4𝑡 = −2𝑡 + 𝑡 | = − (−2 + ) =
3 1 3 3
𝐶1 1

2. Along 𝐶2
The equation of the first path 𝐶2 is 𝑥 = 0, 𝑦 + 𝑧 = 2
𝑥 = 0, 𝑦 = 𝑡, 𝑧 = 2 − 𝑡,
𝑟 = 〈0, 𝑡, 2 − 𝑡〉, ⃑⃑⃑
𝑟 ′ = 〈0, 1, 2〉, 𝐹 (𝑟(𝑡)) = 〈0, 0, 0〉

𝐼2 = ∫ 𝐹 ∙ 𝑑𝑟 = 0
𝐶2

3. Along 𝐶3
The equation of the first path 𝐶3 is 𝑦 = 0, 2𝑥 + 𝑧 = 2
𝑦 = 0, 𝑥 = 𝑡, 𝑧 = 2 − 2𝑡,
- 91 -
Chapter 3 Multiple Integrals
𝑟 = 〈𝑡, 0, 2 − 2𝑡〉, ⃑⃑⃑
𝑟 ′ = 〈1, 0, −2〉, 𝐹 (𝑟(𝑡)) = 〈2𝑡 − 2𝑡 2 , 0, 6𝑡 − 6𝑡 2 〉

1 1
2 2 )𝑑𝑡 2 )𝑑𝑡 2
10 3 1
𝐼3 = ∫ 𝐹 ∙ 𝑑𝑟 = ∫(2𝑡 − 2𝑡 − 12𝑡 + 12𝑡 = ∫(−10𝑡 + 10𝑡 = −5𝑡 + 𝑡 |
3 0
𝐶3 0 0

10
= −5 +
3
2 10
𝐼 = 𝐼1 + 𝐼2 + 𝐼2 = +0−5+ = −1
3 3
Example 40 Verify Stoke’s Theorem by computing both integrals for 𝑭 ⃑ = 𝒚 𝐢 + 𝒛𝟐 𝐣 + 𝒙𝒛 𝐤 and
𝑺 is the portion of the hemisphere 𝒛 = √𝟖 − 𝒙𝟐 − 𝒚𝟐 cutting by the cylinder 𝒙𝟐 + 𝒚𝟐 =
𝟒, oriented upward.
Solution:
Surface Integral:
z

𝐢 𝐣 𝐤
𝜕 𝜕 𝜕
curl 𝐹 = |𝜕𝑥 𝜕𝑦 𝜕𝑧
| = −2𝑧 𝐢 − 𝑧 𝐣 − 𝐤
2
𝑦 𝑧 𝑥𝑧
The given orientation of 𝐶 corresponds to an upward orientation of
the surface 𝑆. Thus, if
𝑧 = 𝑓(𝑥, 𝑦) = 2, y
then the upper normal is
x
⃑ = 〈−𝑓𝑥 , −𝑓𝑦 , 1〉 = 〈0, 0, 1〉 .
𝑁
Hence,

⃑ 𝑑𝐴 = ∬ −1 𝑑𝐴 = −4𝜋
̂ 𝑑𝑆 = ∬(curl 𝐹 ) ∙ 𝑁
∬(curl 𝐹 ) ∙ 𝐧
𝑆 𝑅 𝑅

Line Integral:
The equation of the path 𝐶 is 𝑧 = 2, 𝑥 = 2 cos 𝑡 , 𝑦 = 2 sin 𝑡
𝑟 = 〈2 cos 𝑡 , 2 sin 𝑡 , 2〉, ⃑⃑⃑
𝑟 ′ = 〈−2 sin 𝑡 , 2 cos 𝑡 , 2〉, 𝐹 (𝑟(𝑡)) = 〈2 sin 𝑡 , 4, −4 cos 𝑡〉

2𝜋 2𝜋

𝐼 = ∮ 𝐹 ∙ 𝑑𝑟 = ∫ (−4 sin2 𝑡 + 8 cos 𝑡)𝑑𝑡 = ∫ (−2[1 − cos 2𝑡] + 8 cos 𝑡)𝑑𝑡


𝑐 0 0
𝜋
1
= [−2 (𝑡 − sin 2𝑡) + 8 sin 𝑡]| = −4𝜋
2 0

Sheet 8
1. Use Stokes’ Theorem to evaluate ∮𝑐 𝐹 ∙ 𝑑𝑟 of the vector field 𝐹 around the curve 𝐶 in the indicated
direction
(i) 𝐹 = 2𝑦 𝐢 + 3𝑥 𝐣 − 𝑧 2 𝐤, 𝐶: 𝑥 2 + 𝑦 2 = 9 in the 𝑥𝑦-plane, counterclockwise when viewed
from above.
- 92 -
Chapter 3 Multiple Integrals
2
(ii) 𝐹 = 𝑦 𝐢 + 𝑥𝑧 𝐣 + 𝑥 𝐤, 𝐶:The boundary of the triangle cut from the plane 𝑥 + 𝑦 + 𝑧 = 1 by
the first octant, counterclockwise when viewed from above.
(iii)𝐹 = 𝑥 2 𝑦 3 𝐢 + 𝐣 + 𝑧 𝐤, 𝐶:The intersection of the cylinder 𝑥 2 + 𝑦 2 = 4 and the hemisphere 𝑧 =
√16 − 𝑥 2 + 𝑦 2 , counterclockwise when viewed from above.
(iv) 𝐹 = 𝑦 3 𝐢 − 𝑥 3 𝐣 + 𝑧 3 𝐤, 𝐶:The trace of the cylinder 𝑥 2 + 𝑦 2 = 1 and the plane 𝑥 + 𝑦 + 𝑧 = 1,
counterclockwise when viewed from above.

2. Use Stokes' theorem to evaluate ∬𝑆 (∇ × 𝐹 ). 𝑛̂ 𝑑𝑆. Assume that the surface 𝑆 is oriented upward

(i) 𝐹 = 2𝑥𝑦 2 𝑧 𝐢 + 2𝑥 2 𝑦𝑧 𝐣 + (𝑥 2 𝑦 2 − 6𝑥) 𝐤, 𝑆 that portion of the plane 𝑧 = 𝑦 that lies inside the
cylinder 𝑥 2 + 𝑦 2 = 1.
(ii) 𝐹 = 3𝑥 2 𝐢 + 𝑥 3 𝑦 𝐣 + 3𝑥 2 𝑦 𝐤, 𝑆 that portion of the plane 𝑧 = 𝑥 that lies inside the rectangular
cylinder defined by the planes 𝑥 = 0, 𝑦 = 0, 𝑥 = 2, 𝑦 = 2.
(iii)𝐹 = 𝑥𝑦𝑧 𝐤, 𝑆 that portion of the paraboloid 𝑧 = 1 − 𝑥 2 − 𝑦 2 for 𝑧 ≥ 0.

3. Verify Stokes, theorem. Assume that the surface 𝑆 is oriented upward.


(i) 𝐹 = 5𝑦 𝐢 − 5𝑥 𝐣 + 3 𝐤, 𝑆 that portion of the plane 𝑧 = 1 within the cylinder 𝑥 2 + 𝑦 2 = 4.
(ii) 𝐹 = 2z 𝐢 − 3𝑥 𝐣 + 4𝑦 𝐤, 𝑆 𝑆 that portion of the paraboloid 𝑧 = 16 − 𝑥 2 − 𝑦 2 for 𝑧 ≥ 0.
(iii)𝐹 = 𝑧 𝐢 + 𝑥 𝐣 + 𝑦 𝐤, 𝑆 that portion of the plane 2𝑥 + 𝑦 + 2𝑧 = 6 in the first octant.
(iv) 𝐹 = 𝑥 𝐢 + 𝑦 𝐣 + 𝑧 𝐤, 𝑆 that portion of the sphere 𝑥 2 + 𝑦 2 + 𝑧 2 = 1 for 𝑧 ≥ 0.

Homework 8:

1. Use Stoke’s Theorem to evaluate ∮𝑐 𝐹 ∙ 𝑑𝑟 where 𝐶: 0 ≤ 𝑥 ≤ 2, 0 ≤ 𝑦 ≤ 1, 𝑧 = 𝑦 2 and 𝐹 =


𝑒 2𝑧 𝐢 + 𝑒 𝑧 sin 𝑦 𝐣 + 𝑒 𝑧 cos 𝑦 𝐤.
2. Verify Stoke’s Theorem by computing both integrals for 𝐹 = 2𝑧 𝐢 + 3𝑥 𝐣 + 5𝑦 𝐤 and 𝑆 is the
portion of the paraboloid 𝑧 = 4 − 𝑥 2 − 𝑦 2 , 𝑧 ≥ 0, oriented upward.
3. Verify Stoke’s Theorem by computing both integrals for 𝐹 = 𝑥 2 𝐢 + 𝑦 2 𝐣 + 𝑧 2 𝐤 and 𝑆 is the portion
of the cone 𝑧 = √𝑥 2 + 𝑦 2 below 𝑧 = 1, oriented upward.
4. Use Stoke’s Theorem to evaluate for ∮𝑐 𝐹 ∙ 𝑑𝑟 where 𝐶 is the intersection between the sphere 𝑥 2 +
𝑦 2 + 𝑧 2 = 1 and the cone 𝑧 = √𝑥 2 + 𝑦 2 and 𝐹 = (𝑧 + sin 𝑥) 𝐢 + (𝑥 + 𝑦 2 ) 𝐣 + (𝑦 + 𝑒 𝑧 ) 𝐤.

Challenging problems
2
1. Use Stokes’ Theorem to evaluate ∮𝑐 𝑧 2 𝑒 𝑥 𝑑𝑥 + 𝑥𝑦 2 𝑑𝑦 + tan−1 𝑦 𝑑𝑧 where 𝐶 is the circle 𝑥 2 + 𝑦 2 =
9, by finding a surface 𝑆 with 𝐶 as its boundary and such that the orientation of 𝐶 is counterclockwise
as viewed from above.
2. Let 𝐶 be a simple closed piecewise-smooth space curve that lies in a plane with normal vector 𝑛̂ =
〈𝑎, 𝑏, 𝑐〉 and has positive orientation with respect to 𝑛̂. Show that the plane area by 𝐶 is
1
∮ (𝑏𝑧 − 𝑐𝑦)𝑑𝑥 + (𝑐𝑥 − 𝑎𝑧)𝑑𝑦 + (𝑎𝑦 − 𝑏𝑥)𝑑𝑧.
2 𝑐
3. If 𝑎 is a constant vector, 𝑟 = 𝑥 𝐢 + 𝑦 𝐣 + 𝑧 𝐤, 𝑆 is an oriented, smooth surface with a simple closed,
smooth, positively oriented curve 𝐶, show that

∬ 2𝑎 . 𝑛̂ 𝑑𝑆 = ∮ (𝑎 × 𝑟) ∙ 𝑑𝑟
𝑐
𝑆

- 93 -
Chapter 4 Complex Numbers

Chapter 4
Complex Numbers

Introduction
One of the fundamental properties of the real numbers is that the square of a real number is always
nonnegative i.e., if 𝑥 is a real number, then 𝑥 2 ≥ 0. This implies, among other things, that certain quadratic
equations don't have real solutions. In particular, the equation 𝑥 2 = −1, has no solution in real numbers.
This doesn't mean however, that the equation cannot have a solution. By the 16th century, it became apparent
to various mathematicians that √−1would be very useful and this new number, and its multiples, entered
slowly into common use. As with all important constants, a special symbol was eventually designated to
represent √−1.

Definition 1
An imaginary number is a number of the form 𝒃𝒊, where 𝒃 is real and
.𝒊 = √−𝟏

With the set of imaginary numbers in hand, we can find a square root for every real number, positive
or negative. Indeed, following the usual algebraic rules, we have
2
(𝑏𝑖)2 = 𝑏 2 𝑖 2 = 𝑏 2 (√−1) = 𝑏 2 (−1) = −𝑏 2
and
3
(𝑏𝑖)3 = 𝑏 3 𝑖 3 = 𝑏 3 (√−1) = 𝑏 3 (−𝑖) = −𝑏 2 𝑖

By combining real and imaginary numbers, we can solve any quadratic equation.

Example 1 Solve the equation 𝒙𝟐 + 𝟐𝒙 + 𝟐 = 𝟎. Using the quadratic formula.


Solution:

We find the two solutions


−2 + √4 − 8 −2 − √4 − 8
𝑥1 = = −1 + 𝑖, 𝑥2 = = −1 − 𝑖
2 2
Note that the two solutions are neither real numbers, nor are they purely imaginary.
- 95 -
Chapter 4 Complex Numbers

Definition 2
A complex number is the number of the form 𝒛 = 𝒙 + 𝒊𝒚, where 𝒙 and 𝒚 are real numbers, and 𝒊 =
√−𝟏. The (real) numbers 𝒙 and 𝒚 are called the real and imaginary parts of 𝒛, respectively, and we
often use the notation
𝒙 = 𝐑𝐞(𝒛) and 𝒚 = 𝐈𝐦(𝒛)

If Re(𝑧) = 0, then 𝑧 is an imaginary number and if Im(𝑧) = 0, then 𝑧 is a real number. In other words,
the real numbers and the imaginary numbers are subsets of the complex numbers. The boldfaced letter ℝ
is used to denote the set of real numbers and the boldfaced letter ℂ is used to denote the set of all complex
numbers.

Definition 3
The complex conjugate of 𝒛 = 𝒂 + 𝒃𝒊 is the number 𝒛̅ = 𝒂 − 𝒃𝒊. We use a bar over the number to
denote the conjugate. .

For any 𝑧1 , 𝑧2 ∈ 𝐶

(1) 𝑧̅̅̅̅̅̅̅̅̅
1 ± 𝑧2 = 𝑧̅1 ± 𝑧̅2
(2) ̅̅̅̅̅̅̅̅
𝑧1 ∙ 𝑧2 = 𝑧̅1 ∙ 𝑧̅2
̅̅̅̅̅
𝑧 𝑧̅
(3) ( 1 ) = 1, 𝑧̅2 ≠ 0
𝑧2 𝑧̅2
(4) ̅̅̅̅
(𝑧̅) = 𝑧

Fundamental operations with complex numbers:


Complex numbers may be added and multiplied, just like real numbers, and the usual properties
(commutativity, associativity and distributivity) continue to hold.

Addition and subtraction:


To add two complex numbers, we add their real and imaginary components separately and use the
distributive rule 𝑏𝑖 + 𝑑𝑖 = (𝑏 + 𝑑)𝑖 for the imaginary parts. In other words,

(𝑎 + 𝑏𝑖) + (𝑐 + 𝑑𝑖) = (𝑎 + 𝑐) + (𝑏 + 𝑑)𝑖

Subtraction is defined similarly


(𝑎 + 𝑏𝑖) − (𝑐 + 𝑑𝑖) = (𝑎 − 𝑐) + (𝑏 − 𝑑)𝑖
Addition of complex numbers satisfies the usual rules of commutativity and associativity i.e., if 𝑢, 𝑣 and 𝑤
are complex numbers then

𝑢 + 𝑣 = 𝑣 + 𝑢 and (𝑢 + 𝑣) + 𝑤 = 𝑢 + (𝑣 + 𝑤)

Example 2 If 𝒛𝟏 = 𝟑 + 𝟐𝒊 and 𝒛𝟐 = 𝟒 − 𝟑𝒊 and 𝒛𝟑 = −𝟑 + 𝟓𝒊, find 𝒛𝟏 + 𝒛𝟐 and 𝒛𝟐 − 𝒛𝟑


Solution:
𝑧1 + 𝑧2 = (3 + 2𝑖) + (4 − 3𝑖) = (3 + 4) + (2 − 3)𝑖 = 7 − 𝑖
and
𝑧2 − 𝑧3 = (4 − 3𝑖) − (−3 + 5𝑖) = (4 + 3) + (−3 − 5)𝑖 = 7 − 8𝑖

- 96 -
Chapter 4 Complex Numbers
Multiplication:
To multiply the numbers 𝑎 + 𝑏𝑖 and 𝑐 + 𝑑𝑖, we use the familiar 'foil' rule from elementary algebra
while keeping in mind that 𝑖 2 = −1, and collecting real and imaginary terms:

(𝑎 + 𝑏𝑖) ∙ (𝑐 + 𝑑𝑖) = 𝑎𝑐 + 𝑎𝑑𝑖 + 𝑐𝑏𝑖 + 𝑏𝑑𝑖 2 = (𝑎𝑐 − 𝑏𝑑) + (𝑎𝑑 + 𝑏𝑐)𝑖

As with addition, the usual properties of commutativity and associativity hold for multiplication of complex
numbers, i.e., if 𝑢, 𝑣 and 𝑤 are complex numbers then

𝑢 ∙ 𝑣 = 𝑣 ∙ 𝑢 and 𝑤 ∙ (𝑢 + 𝑣) = 𝑤 ∙ 𝑢 + 𝑤 ∙ 𝑣

Example 3 If 𝒛𝟏 = 𝟑 + 𝟐𝒊 and 𝒛𝟐 = 𝟒 − 𝟑𝒊 and 𝒛𝟑 = −𝟑 + 𝟓𝒊, find 𝒛𝟏 ∙ 𝒛𝟐 and 𝒛𝟑 ∙ 𝒛𝟏


Solution:

𝑧1 ∙ 𝑧2 = (3 + 2𝑖)(4 − 3𝑖) = 12 − 9𝑖 + 8𝑖 − 6𝑖 2 = (12 − (−6)) + (−9 + 8)𝑖 = 18 − 𝑖

𝑧3 ∙ 𝑧1 = (−3 + 5𝑖)(3 + 2𝑖) = −9 − 6𝑖 + 15𝑖 + 10𝑖 2 = (−9 + (−10)) + (15 − 6)𝑖

= −19 + 9𝑖
Division:
To divide 𝑧1 = 𝑐 + 𝑑𝑖 by 𝑧2 = 𝑎 + 𝑏𝑖, we multiply by 𝑧̅2 = (𝑎 − 𝑏𝑖) both sides of the fraction, so
𝑧1 𝑧1 𝑧̅2 𝑐 + 𝑑𝑖 𝑎 − 𝑏𝑖 𝑎𝑐 + 𝑏𝑑 + (𝑎𝑑 − 𝑏𝑐)𝑖
= × = × =
𝑧2 𝑧2 𝑧̅2 𝑎 + 𝑏𝑖 𝑎 − 𝑏𝑖 𝑎2 + 𝑏 2
𝒛𝟏 𝒛𝟏
Example 4 If 𝒛𝟏 = 𝟑 + 𝟐𝒊 and 𝒛𝟐 = 𝟒 − 𝟑𝒊 and 𝒛𝟑 = −𝟑 + 𝟓𝒊, find and
𝒛𝟐 𝒛𝟑

Solution:
𝑧1 𝑧1 𝑧̅2 3 + 2𝑖 4 + 3𝑖 12 − 6 + (−9 − 8)𝑖 6 17
= × = × = = − 𝑖
𝑧2 𝑧2 𝑧̅2 4 − 3𝑖 4 + 3𝑖 25 25 25

𝑧1 𝑧1 𝑧̅3 3 + 2𝑖 −3 − 5𝑖 −9 + 10 + (−15 − 6)𝑖 1 21


= × = × = = − 𝑖
𝑧3 𝑧3 𝑧̅3 −3 + 5𝑖 −3 − 5𝑖 34 34 34

The geometry of complex numbers:


1. Complex numbers as points in the plane:
Every complex number has two real coordinates, namely its real and imaginary parts. It is natural
therefore to represent complex numbers as points in what is called the complex plane. In this representation,
the convention is to plot the real part of the complex number on the horizontal axis and the imaginary part
on the vertical axis. For this reason, the horizontal axis of the complex plane is called the real axis and the
vertical axis is called the imaginary axis. In this representation, the real numbers correspond to the points
on the real axis and the (purely) imaginary numbers correspond to the points on the imaginary axis. The real
and imaginary parts of a complex number are also called the rectangular coordinates of that number.

- 97 -
Chapter 4 Complex Numbers
Example 5 Express the complex number 𝟑 + 𝟒𝒊, −𝟐 + 𝟑𝒊 and −𝟒 − 𝟑𝒊 on the z-plane.
Solution:

Im

3  4i
 2  3i

Re

 4  3i

2. The modulus of a complex number:


The magnitude of a real number 𝑤 is given by its absolute value, |𝑤|, which may be described as the
distance of the number from the origin (𝑧 = 0). This idea is easily extended to all complex numbers.

Definition 4
The modulus, or absolute value, of a complex number 𝒛 = 𝒂 + 𝒃𝒊, is denoted by |𝒛| and defined to
be the distance in the complex plane between the point 𝒛 and the point 0. Equivalently, |𝒛| is the length
of the vector corresponding to 𝒛.

Comments:

a. The modulus of a complex number may be computed from its rectangular coordinates (the real and
imaginary parts) using the Pythagorean Theorem, i.e.,

|𝑎 + 𝑏𝑖| = √𝑎2 + 𝑏 2

b. It follows directly from the definition that |𝑧̅| = |𝑧|, since

|𝑎 − 𝑏𝑖| = √𝑎2 + (−𝑏)2 = √𝑎2 + 𝑏 2 = |𝑎 + 𝑏𝑖|

3. The argument of a complex number:


The modulus of a complex number defines its magnitude. We can also associate an angle to each nonzero
complex number.

Definition 5
The argument (or phase) of 𝒛 = 𝒂 + 𝒃𝒊 (𝒛 ≠ 𝟎) is the angle, 𝜽, between the positive real axis and
the line segment connecting 𝒛 to 𝟎, measured in the counterclockwise direction. The argument of 𝒛 is
denoted by 𝐚𝐫𝐠(𝒛).

Comments:

a. In the definition of the argument, for any complex number 𝑧, arg(𝑧) is only determined up to a multiple
of 2𝜋.
- 98 -
Chapter 4 Complex Numbers

b. Among the (infinitely many) possible choices for arg(𝑧), we'll usually choose the one that lies between
0 and 2𝜋. This value of arg(𝑧) is called the principal value of the argument.

The correct approach is to adjust the value of tan−1 (𝑏/𝑎) according to the quadrant of the complex plane
in which 𝑎 + 𝑏𝑖 lies.

1. If 𝑎 + 𝑏𝑖 is in the first quadrant, then the arctangent function produces the correct value, so no
adjustment is necessary. This is the case of 𝑧1 in the following Figure. I.e.,

𝑏
if 𝑎 > 0 and 𝑏 > 0, then arg(𝑎 + 𝑏𝑖) = tan−1
𝑎

𝜋 𝑏
2. If 𝑎 + 𝑏𝑖 is in the second quadrant, then − < tan−1 < 0, and the correct value of the argument is
2 𝑎
𝑏 𝑏
𝜋 + tan−1 . In the following Figure, this corresponds to the case of 𝑧2 , in which case tan−1 = −𝜃,
𝑎 −𝑎
but arg(𝑧2 ) = 𝜋 − 𝜃 I.e.,

𝑏 𝑏
if 𝑎 < 0 and 𝑏 > 0, then arg(𝑎 + 𝑏𝑖) = 𝜋 + tan−1 = 𝜋 − tan−1 | |
𝑎 𝑎

𝑏 𝜋
3. If 𝑎 + 𝑏𝑖 is in the third quadrant, then 0 < tan−1 < , and the correct value of the argument is 𝜋 +
𝑎 2
𝑏 −𝑏
tan−1 . In the following Figure, this corresponds to the case of 𝑧3 , in which case tan−1 = 𝜃, but
𝑎 −𝑎
arg(𝑧2 ) = 𝜋 + 𝜃 I.e.,

−𝑏 𝑏
if 𝑎 < 0 and 𝑏 < 0, then arg(𝑎 + 𝑏𝑖) = 𝜋 + tan−1 = 𝜋 + tan−1 | |
−𝑎 𝑎

𝜋 𝑏
4. If 𝑎 + 𝑏𝑖 is in the fourth quadrant, then − < tan−1 < 0, and the correct value of the argument is
2 𝑎
𝑏 −𝑏
2𝜋 + tan−1 . In the following Figure, this corresponds to the case of 𝑧4 , in which case tan−1 = −𝜃,
𝑎 𝑎
but arg(𝑧2 ) = 2𝜋 − 𝜃 or arg(𝑧2 ) = −𝜃, i.e.,

𝑏 𝑏
if 𝑎 > 0 and 𝑏 < 0, then arg(𝑎 + 𝑏𝑖) = 2𝜋 + tan−1 = 2𝜋 − tan−1 | |
𝑎 𝑎

𝑏
5. Finally, if 𝑎 = 0, then tan−1 is not defined, but the argument is easily determined without the
𝑎
𝜋
arctangent function. Specifically, If Re(𝑧) = 0 and Im(𝑧) > 0 then arg(𝑧) = , and if Re(𝑧) = 0
2
3𝜋
and Im(𝑧) < 0 then arg(𝑧) = . If 𝑏 = 0 and 𝑎 > 0, then arg(𝑧) = 0 and if 𝑏 = 0 and 𝑎 < 0 then
2
arg(𝑧) = 𝜋.

- 99 -
Chapter 4 Complex Numbers
Im

z2  a  ib z1  a  ib

b b
    tan 1   tan 1
a a
Re

b b
    tan 1    tan 1
a a
b
  2  tan 1
a
z3  a  ib z4  a  ib

Example 6 Find the moduli and arguments of 𝒖 = 𝟑 + 𝟒𝒊 and 𝒗 = 𝟏𝟐 − 𝟓𝒊.


Solution:

The moduli are easy to compute, even without a calculator:


|𝑢| = √9 + 16 = 5 and |𝑣| = √144 + 25 = 13.
Since 𝑢 lies in the first quadrant (both Re(𝑢) > 0 and Im(𝑢) > 0), it follows
4
arg(𝑢) = tan−1 ( ) = 53.13°
3
Likewise, since 𝑣 lies in the fourth quadrant (Re(𝑣) > 0 and Im(𝑣) < 0), it follows
−5
arg(𝑣) = tan−1 ( ) = −22.619° = 337.38°
12

The polar-coordinate representation of complex numbers:


Im
The modulus and argument of a complex number are
called the polar coordinates of the number, and they z  a  ib
determine that number completely, i.e., given the polar
r
coordinates of 𝑧, we can easily find Re(𝑧) and Im(𝑧) using r sin 
basic trigonometry.

Proposition 1. Re
r cos
If |𝑧| = 𝑟 and arg(𝑧) = 𝜃, then Re(𝑧) = 𝑟 cos 𝜃 and
Im(𝑧) = 𝑟 sin 𝜃. In other words, 𝑧 = 𝑟 cos 𝜃 + 𝑖 𝑟 sin 𝜃,
or more succinctly
𝑧 = 𝑟(cos 𝜃 + 𝑖 sin 𝜃)

- 100 -
Chapter 4 Complex Numbers
Exponential notation:

The polar coordinate representation of a complex number, 𝑧 = 𝑟(cos 𝜃 + 𝑖 sin 𝜃), can be simplified
by using Euler's formula:
𝑧 = 𝑟𝑒 𝑖𝜃
where
𝑒 𝑖𝜃 = cos 𝜃 + 𝑖 sin 𝜃
We can formally establish the previous by using the Maclaurin series for 𝑒 𝑥 and replacing 𝑥 by 𝑖𝜃 and
rearranging terms:
(𝑖𝜃)2 (𝑖𝜃)3 (𝑖𝜃)4 (𝑖𝜃)5
𝑒 𝑖𝜃 = 1 + 𝑖𝜃 + + + + +⋯
2! 3! 4! 5!
𝜃2 𝜃4 𝜃6 𝜃3 𝜃5 𝜃7
= (1 − + − + ⋯ ) + 𝑖 (𝜃 − + − + ⋯)
2! 4! 6! 3! 5! 7!
= cos 𝜃 + 𝑖 sin 𝜃.

Proposition 2.
For any number 𝜃 and any integer 𝑘, 𝑒 𝑖(𝜃+2𝑘𝜋) = 𝑒 𝑖𝜃 .

Proposition 3.
If 𝑧1 = 𝑒 𝑖𝜃1 and 𝑧2 = 𝑟2 𝑒 𝑖𝜃2 , then

𝑖𝜃1 𝑖𝜃2 𝑖(𝜃1 +𝜃2 )


𝑧1 (𝑟1 𝑒 𝑖𝜃1 ) 𝑟1 𝑖(𝜃 −𝜃 )
𝑧1 ∙ 𝑧2 = (𝑟1 𝑒 )(𝑟2 𝑒 ) = 𝑟1 𝑟2 𝑒 and = = 𝑒 1 2
𝑧2 (𝑟2 𝑒 𝑖𝜃2 ) 𝑟2

Example 7 Transform the following complex numbers in the form 𝒆𝒊𝜽


𝟏. 𝒛 = 𝟑 + 𝟒𝒊 𝟐. 𝒛 = −𝟐 + 𝟐𝒊 𝟑. 𝒛 = −𝟏 − 𝟓𝒊 𝟒. 𝒛 = 𝟑 − 𝟐𝒊
Solution:
1. 𝑧 = 3 + 4𝑖 lies in the 1st quadrant, 0 < θ < 90°
4
𝑟 = √32 + 42 = √25 = 5, 𝜃 = tan−1 = 53.13° = 0.29𝜋 ⇒ 𝑧 = 5𝑒 𝑖(0.29𝜋)
3

2. 𝑧 = −2 + 2𝑖 lies in the 2nd quadrant, 90° < θ < 180°


3𝜋
2 3𝜋
𝑟 = √22 + 22 = √8 = 2√2, 𝜃 = tan−1 − = −45° = 180° − 45° = ⇒ 𝑧 = 2√2𝑒 4 𝑖
2 4

3. 𝑧 = −1 − 5𝑖 lies in the 3rd quadrant, 180° < θ < 270°


5
𝑟 = √12 + 52 = √26, 𝜃 = tan−1 + 𝜋 = 180° + 78.69° = 258.69° = 1.43𝜋 ⇒ 𝑧 = √26𝑒1.43𝜋𝑖
1

4. 𝑧 = 3 − 2𝑖 in the 4th quadrant, 270° < θ < 360°


−2
𝑟 = √32 + 22 = √13, 𝜃 = tan−1 = −33.69° = 360° −33.69° = 326.3° ⇒ 𝑧 = √13𝑒1.81𝜋𝑖
3

De Moivre's Theorem:

If 𝑧 = 𝑥 + 𝑖𝑦 = 𝑟𝑒 𝑖𝜃 , then for any positive integer 𝑛


𝑛
𝑧 𝑛 = (𝑥 + 𝑖𝑦)𝑛 = (𝑟𝑒 𝑖𝜃 ) = 𝑟 𝑛 𝑒 𝑖𝑛𝜃 = 𝑟 𝑛 (cos 𝑛𝜃 + 𝑖 sin 𝑛𝜃)
- 101 -
Chapter 4 Complex Numbers

Example 8 Express the following complex numbers in the form 𝒂 + 𝒊𝒃


𝒂. (𝟔 + 𝟓𝒊)𝟑 𝒃. (𝟑 − 𝟖𝒊)𝟓 𝒄. (−𝟐 + 𝟕𝒊)𝟒 𝒅. (−𝟏𝟔 − 𝟗𝒊)𝟔
Solution:
a) (6 + 5𝑖)3
5
𝑧 = 𝑤 3 , 𝑤 = 6 + 5𝑖 ⇒ 𝑟 = √61, 𝜃 = tan−1 ( ) = 39.8°
6

𝑧 = 𝑤 3 = 𝑟 3 (cos 3𝜃 + 𝑖 sin 3𝜃) = 61√61(cos 119.4° + 𝑖 sin 119.4° )

b) (3 − 8𝑖)5
8
𝑧 = 𝑤 5 , 𝑤 = 3 − 8𝑖 ⇒ 𝑟 = 73, 𝜃 = tan−1 (− ) = −69.4° +360° = 290.6°
3

𝑧 = 𝑤 5 = 𝑟 5 (cos 5𝜃 + 𝑖 sin 5𝜃) = 732 √73(cos 1453° + 𝑖 sin 1453° )

c) (−2 + 7𝑖)4
7
𝑧 = 𝑤 4 , 𝑤 = −2 + 7𝑖 ⇒ 𝑟 = √53, 𝜃 = tan−1 (− ) = −74.05° +180° = 105.95°
2
4 4 (cos 2 (cos
𝑧=𝑤 =𝑟 4𝜃 + 𝑖 sin 4𝜃) = 53 423.8° + 𝑖 sin 423.8° )

d) (−16 − 9𝑖)6
9
𝑧 = 𝑤 6 , 𝑤 = −16 − 9𝑖 ⇒ 𝑟 = √337, 𝜃 = tan−1 ( ) = 29.35° +180° = 209.35°
16

𝑧 = 𝑤 6 = 𝑟 6 (cos 6𝜃 + 𝑖 sin 6𝜃) = 3373 (cos 1256.1° + 𝑖 sin 1256.1° )

Roots of complex Numbers:


A complex number 𝑤 is called the 𝑛th root of another complex number 𝑧 = 𝑥 + 𝑖𝑦 if 𝑤 𝑛 = 𝑧, and we
write
1
𝑛
1 𝜃 + 2𝑘𝜋 𝜃 + 2𝑘𝜋
𝑤 𝑘 = 𝑧 𝑛 = √𝑧 = 𝑟 𝑛 (cos + 𝑖 sin ), 𝑘 = 0,1, 2, 3, … , 𝑛 − 1
𝑛 𝑛

Example 9 Determine the square roots of the following complex numbers


𝒂. 𝟏 + 𝒊 𝒃. 𝟑 − 𝟒𝒊
Solution:
a) 1 + 𝑖
𝜋
𝑧 = 1 + 𝑖 = √2 ∠tan−1 (1) = √2 ∠45° = √2𝑒 4 𝑖
45° +2𝑘𝜋
1 ( 2 )𝑖 4 45° + 2𝑘𝜋 45° + 2𝑘𝜋
𝑧2 = √𝑧 = √√2𝑒 = √2 [cos ( ) + 𝑖 sin ( )] , 𝑘 = 0,1
2 2
4 45° 45°
for 𝑘 = 0 ⇒ 𝑤1 = √𝑧 = √2 [cos ( ) + 𝑖 sin ( )] = 1.098 + 0.455𝑖
2 2

4 45° +360° 45° +360°


for 𝑘 = 1 ⇒ 𝑤2 = √𝑧 = √2 [cos ( ) + 𝑖 sin ( )] = −1.098 − 0.455𝑖
2 2

b) 3 − 4𝑖
4
𝑧 = 3 − 4𝑖 = 5 ∠tan−1 (− ) = 5 ∠306.86°
3

- 102 -
Chapter 4 Complex Numbers
306.86° +2𝑘𝜋 ° °
1 (
2
)𝑖 306.86 + 2𝑘𝜋 306.86 + 2𝑘𝜋
𝑧 2 = √𝑧 = √5𝑒 = √5 [cos ( ) + 𝑖 sin ( )] , 𝑘 = 0,1
2 2
306.86° 306.86°
for 𝑘 = 0 ⇒ 𝑧1 = √5 [cos ( ) + 𝑖 sin ( )] = −2 + 𝑖
2 2

306.86° +360° 306.86° +360°


for 𝑘 = 1 ⇒ 𝑧2 = √5 [cos ( ) + 𝑖 sin ( )] = 2 − 𝑖
2 2

Example 10 Find the cubic roots of the equation – 𝟖𝒊


Solution:
3 3𝜋 3 3𝜋
𝑧 = √−8𝑖 = √8𝑒 2 𝑖 = 2 √𝑒 2 𝑖
3
If 𝑧 3 = −8𝑖,

270° +2𝑘𝜋 270° +2𝑘𝜋


𝑧 = 2 [cos ( ) + 𝑖 sin ( )] , 𝑘 = 0, 1, 2
3 3

270° 270°
for 𝑘 = 0 ⇒ 𝑧1 = 2 [cos ( ) + 𝑖 sin ( )] = 2𝑖,
3 3
630° 630°
for 𝑘 = 1 ⇒ 𝑧2 = 2 [cos ( ) + 𝑖 sin ( )] = −√3 − 𝑖,
3 3

990° 990°
for 𝑘 = 2 ⇒ 𝑧3 = 2 [cos ( ) + 𝑖 sin ( )] = √3 − 𝑖 .
3 3

Example 11 Find all roots of the equation 𝐳 𝟒 − 𝟏 = −𝒊


Solution:
4 𝜋 4 𝜋
𝑧 = √1 − 𝑖 = √√2𝑒 − 4 𝑖 = 2 √𝑒 − 4 𝑖
4
If 𝑧 4 = 1 − 𝑖,

8 −45° +2𝑘𝜋 −45° +2𝑘𝜋


𝑧 = √2 [cos ( ) + 𝑖 sin ( )] , 𝑘 = 0, 1, 2, 3
4 4

8 −45° −45°
for 𝑘 = 0 ⇒ 𝑧1 = √2 [cos ( ) + 𝑖 sin ( )] ≈ 1.069  − 0.212𝑖,
4 4

8 315° 315°
for 𝑘 = 1 ⇒ 𝑧2 = √2 [cos ( ) + 𝑖 sin ( )] ≈ 0.212 + 1.069𝑖,
4 4

8 675° 675°
for 𝑘 = 2 ⇒ 𝑧3 = √2 [cos ( ) + 𝑖 sin ( )] ≈ −1.069 + 0.212𝑖,
4 4

8 1395° 1395°
for 𝑘 = 3 ⇒ 𝑧4 = √2 [cos ( ) + 𝑖 sin ( )] ≈ 0.212 − 1.069𝑖.
4 4

Example 12 Find all roots of the equation 𝐳 𝟑 = 𝒊


Solution:
3 𝜋 3 𝜋
𝑧 = √𝑖 = √𝑒 2 𝑖 = √𝑒 2 𝑖
3
If 𝑧 3 = 𝑖,

90° +2𝑘𝜋 90° +2𝑘𝜋


𝑧 = [cos ( ) + 𝑖 sin ( )] , 𝑘 = 0, 1, 2
3 3

𝜋 𝜋 √3 1
for 𝑘 = 0 ⇒ 𝑧1 = [cos ( ) + 𝑖 sin ( )] = + 𝑖,
6 6 2 2

5𝜋 5𝜋 √3 1
for 𝑘 = 1 ⇒ 𝑧2 = [cos ( ) + 𝑖 sin ( )] = − + 𝑖,
6 6 2 2
3𝜋 3𝜋
for 𝑘 = 2 ⇒ 𝑧3 = [cos ( ) + 𝑖 sin ( )] = −𝑖 .
2 2

- 103 -
Chapter 4 Complex Numbers

Sheet 9
1. Write the given number in the form 𝑎 + 𝑖𝑏
(i) 3𝑖 5 − 𝑖 4 + 7𝑖 3 − 10𝑖 2 − 9 (ii) 3(4 − 𝑖) − 3(5 + 2𝑖) (iii) 𝑖(4 − 𝑖) + 4𝑖(1 + 2𝑖)
2−4𝑖
(iv) (2 − 3𝑖)(4 + 𝑖) (v) (2 + 3𝑖)2 (vi)
3+5𝑖
(1+𝑖)(1−2𝑖) 1
(vii) (viii) (3 + 6𝑖) + (4 − 𝑖 )(3 + 5𝑖 ) +
(2+𝑖)(4−3𝑖) 2−𝑖
2. Let 𝑧 = 𝑥 + 𝑖𝑦. Find the indicated expression
(i) Re(1/𝑧) (ii) Re(𝑧 2 ) (iii) Im(2𝑧 + 4𝑧̅ − 4𝑖)
(iv) Im(𝑧̅ 2 + 𝑧 2 ) (v) |𝑧 − 1 − 3𝑖| (vi) |𝑧 + 5𝑧̅|
3. Find a complex number 𝑧 satisfying the given equation
(i) 2𝑧 = 𝑖(2 + 9𝑖) (ii) 𝑧 − 2𝑧̅ + 7 − 6𝑖 = 0 (iii) 𝑧̅ 2 = 4𝑧
4. Express the following complex numbers in the polar form
1 1+2𝑖
(i) 4 + 4√3𝑖 (ii) (iii)
1+𝑖 1−3𝑖
5. Compute the indicated power
9 4
(i) (1 + √3𝑖) (ii) (2 − 2𝑖)5 (iii) (−√2 + √6𝑖)
6. Find all solutions of the given equation
(i) 𝑧 4 + 1 = 0 (ii) 𝑧 8 − 2𝑧 4 + 1 = 0 (iii) 𝑧 3 = 1 − 𝑖
Homework 8:
1. Evaluate the following expressions
a) (3 + 2𝑖) − (8 − 5𝑖)
b) (4 − 2𝑖)(1 − 5𝑖)
c) (−2 − 4𝑖) / 𝑖
d) (− 3 + 2𝑖)/(3 − 6𝑖)
− 3+2𝑖 10
e) ( )
3−6𝑖

2. If (𝑥 + 𝑦𝑖)/𝑖 = (7 + 9𝑖), where 𝑥 and 𝑦 are real, what is the value of (𝑥 + 𝑦𝑖)(𝑥 − 𝑦𝑖)?
3. Determine all complex number 𝑧 that satisfy the equation 𝑧 + 3 𝑧̅ = 5 − 6𝑖.
4. Find all complex numbers of the form 𝑧 = 𝑎 + 𝑏𝑖 , where 𝑎 and 𝑏 are real numbers such that 𝑧𝑧̅ = 25
and 𝑎 + 𝑏 = 7.
5. The complex number 2 + 4𝑖 is one of the root to the quadratic equation 𝑥 2 + 𝑏𝑥 + 𝑐 = 0, where 𝑏 and
𝑐 are real numbers. Find the values of 𝑏 and 𝑐.
6. Find all complex numbers 𝑧 such that 𝑧 2 = −1 + 2 √6𝑖.
Challenging problems:
1. Given that the complex number 𝑧 = −2 + 7𝑖 is a root to the equation 𝑧 3 + 6𝑧 2 + 61𝑧 + 106 = 0
find the real root to the equation.
3𝜋 3𝜋
1+sin 8 +𝑖 cos 8 𝜋 𝜋
2. Prove that 3𝜋 3𝜋 = sin + 𝑖 cos
1+sin −𝑖 cos 8 8
8 8
1 1
3. If 𝑥 + = −1, then find 𝑥 36 + ?
𝑥 𝑥 36
1
4. If 𝑧 = cos 𝜃 + 𝑖 sin 𝜃, prove that 𝑧 𝑛 + = 2 cos 𝑛𝜃.
𝑧𝑛
𝑎−𝑖𝑏 𝑎2 +𝑏2
5. If 𝑥 − 𝑖𝑦 = √ , prove that (𝑥 2 + 𝑦 2 )2 = .
𝑐−𝑖𝑑 𝑐 2 +𝑑 2

- 104 -
Chapter 4 Complex Numbers
Functions of a complex variables:

We shall define a function 𝑓 from a set 𝐴 into a set 𝐵 as a rule which assigns a unique element of 𝐵 to
each element 𝐴. If 𝑧 and 𝑤 are variables on 𝐴 and 𝐵 respectively, the expression 𝑤 = 𝑓(𝑧) is used to indicate
that 𝑓 is a function from 𝐴 to 𝐵. If 𝑤 = 𝑓(𝑧) and 𝑧 = 𝑥 + 𝑖𝑦 then 𝑤 can be written in the form 𝑢(𝑥, 𝑦) +
𝑖𝑣(𝑥, 𝑦) where 𝑢 and 𝑣 are real valued function of 𝑥 and 𝑦.

Example 13 Express the function 𝒇(𝒛) = 𝒛𝟑 in the form 𝒘 = 𝒖(𝒙, 𝒚) + 𝒊𝒗(𝒙, 𝒚)


Solution:
Since 𝑓(𝑧) = 𝑧 3 and 𝑧 = 𝑥 + 𝑖𝑦

𝑓(𝑧) = (𝑥 + 𝑖𝑦)3 = (𝑥 + 𝑖𝑦)(𝑥 + 𝑖𝑦)2 = (𝑥 + 𝑖𝑦)[𝑥 2 − 𝑦 2 + 2𝑖𝑥𝑦]

= [𝑥 3 − 𝑥𝑦 2 − 2𝑥𝑦 2 ] + 𝑖[𝑥 2 𝑦 − 𝑦 3 + 2𝑥 2 𝑦] = [𝑥 3 − 3𝑥𝑦 2 ] + 𝑖[−𝑦 3 + 3𝑥 2 𝑦]

Hence, 𝑓(𝑧) = [𝑥 3 − 3𝑥𝑦 2 ] + 𝑖[−𝑦 3 + 3𝑥 2 𝑦]

Elementary functions:

An elementary function is defined as a function or the inverse of a function that is generated by


constants and the independent variable 𝑧 by means of finite sequence of elementary operations.
Example of elementary functions:
(1) The general polynomial function

𝑓(𝑧) = 𝑎𝑛 𝑧 𝑛 + 𝑎𝑛−1 𝑧 𝑛−1 + ⋯ + 𝑎1 𝑧 + 𝑎0

(2) The rational function

𝑃𝑛 (𝑧)
𝑓(𝑧) =
𝑄𝑚 (𝑧)

where 𝑃𝑛 (𝑧) and 𝑄𝑛 (𝑧) are general polynomial functions.

(3) The exponential function 𝑓(𝑧) = 𝑒 𝑧 .

(4) The trigonometric functions such that as sin 𝑧 , cos 𝑧 , tan 𝑧 …


- 105 -
Chapter 4 Complex Numbers
(5) The hyperbolic functions such that as sinh 𝑧 , cosh 𝑧 , tanh 𝑧 …

(6) The logarithmic function 𝑓(𝑧) = log 𝑧.

(7) The square root function 𝑓(𝑧) = √𝑧.

In the following, we introduce some important elementary functions and study their important properties.

The exponential function 𝒇(𝒛) = 𝒆𝒛 :


The exponential function 𝑓(𝑧) = 𝑒 𝑧 is defined for all 𝑧 = 𝑥 + 𝑖𝑦 by:
𝑒 𝑧 = 𝑒 𝑥+𝑖𝑦 = 𝑒 𝑥 [cos 𝑦 + 𝑖 sin 𝑦]
Basic properties of the exponential function:
Using the above definition it can be shown quite easily that the exponential function satisfies the following
properties:
1
(1) 𝑒 𝑖𝑦 = cos 𝑦 + 𝑖 sin 𝑦 (6) 𝑒 −𝑧 =
𝑒𝑧

𝑒 𝑧1
(2) 𝑒 𝑧 = 𝑒 𝑥 𝑒 𝑖𝑦 (7) = 𝑒 𝑧1 −𝑧2
𝑒 𝑧2

(3) 𝑒 0 = 1 (8) (𝑒 𝑧 )𝑝 = 𝑒 𝑝𝑧 , 𝑝 is an integer

(4) |𝑒 𝑧 | = 𝑒 𝑥 (9) 𝑒 𝑧 = 𝑒 𝑧+𝑖2𝑘𝜋

(5) 𝑒 𝑧1 ∙ 𝑒 𝑧2 = 𝑒 𝑧1 +𝑧2 (10) 𝑒 𝑧̅ = ̅̅̅


𝑒𝑧

The logarithmic function 𝒇(𝒛) = 𝐥𝐧 𝒛:


The logarithmic function 𝑓(𝑧) = ln 𝑧0 of a complex number 𝑧0 is any number 𝑤0 which satisfies 𝑧0 =
𝑒 𝑤0 . If 𝑤0 is a logarithm of 𝑧0 , then the logarithmic function 𝑤 = ln 𝑧 defined on any domain for which the
equation 𝑧 = 𝑒 𝑤 has a solution.
If 𝑧 = 𝑥 + 𝑖𝑦 = 𝑟𝑒 𝑖𝜃
𝑤 = ln 𝑧 = log e 𝑟 + 𝑖(𝜃 + 2𝑘𝜋), 𝑘 = 0, ±1, ±2, ±3, …
where log 𝑒 𝑟 = ln 𝑟.
The principle value of w = Ln 𝑧 when 𝑘 = 0,
𝑤 = Ln 𝑧 = log e 𝑟 + 𝑖𝜃
Example 14 Determine all values of 𝐥𝐧 𝒛
𝒂. 𝒛 = 𝟏 + 𝒊 𝒃. 𝒛 = 𝟑 − 𝟒𝒊
Solution:
𝜋
(a) 𝑧 = 1 + 𝑖 = √2𝑒 4 𝑖
𝜋
ln 𝑧 = ln(1 + 𝑖) = log e √2 + 𝑖 ( + 2𝑘𝜋) , 𝑘 = 0, ±1, ±2, ±3, …
4
(b) 𝑧 = 3 − 4𝑖 = 5𝑒 −0.29 𝜋 𝑖
ln 𝑧 = ln(3 − 4𝑖) = log e 5 + 𝑖(−0.29𝜋 + 2𝑘𝜋), 𝑘 = 0, ±1, ±2, ±3, …
- 106 -
Chapter 4 Complex Numbers
Example 15 find all solution of 𝒛 the following equations
𝒂. 𝒆𝒛 = −𝟐 𝒃. 𝒆𝒛 = −𝟑 + 𝟒𝒊
Solution:
In order to solve the equation, we assume 𝑤 = 𝑒 𝑧 ⇔ 𝑧 = ln 𝑤
(a) 𝑤 = −2 = 2𝑒 𝜋 𝑖
𝑤 = 𝑒𝑧 ⇔ 𝑧 = ln 𝑤 = ln(−2) = log e 2 + 𝑖(𝜋 + 2𝑘𝜋), 𝑘 = 0, ±1, ±2, ±3, …
(b) 𝑤 = −3 + 4𝑖 = 5𝑒 0.7𝜋 𝑖
𝑤 = 𝑒𝑧 ⇔ 𝑧 = ln 𝑤 = ln(−3 + 4𝑖) = log e 5 + 𝑖(0.7𝜋 + 2𝑘𝜋), 𝑘 = 0, ±1, ±2, ±3, …

Example 16 find the principle value of 𝒛 satisfies the following equations


𝒂. 𝒆𝒛 = −𝒊 𝒃. 𝒆𝒛 = 𝟏 + √𝟑 𝒊
Solution:
In order to solve the equation, we assume 𝑤 = 𝑒 𝑧 ⇔ 𝑧 = Ln 𝑤
3𝜋
𝑖
(c) 𝑤 = −𝑖 = 𝑒 2
3𝜋
𝑤 = 𝑒𝑧 ⇔ 𝑧 = Ln 𝑤 = Ln(−𝑖) = log e 1 + 𝑖 ( )
2
𝜋
(d) 𝑤 = 1 + √3𝑖 = 2𝑒 3 𝑖
𝜋
𝑤 = 𝑒𝑧 ⇔ 𝑧 = Ln 𝑤 = Ln(1 + √3𝑖) = log e 2 + 𝑖 ( )
3

𝒛
The general exponential form 𝐳𝟏𝟐 (Power Complex):
The general power of a complex number 𝑧 = 𝑥 + 𝑖𝑦 is defined by the formula
𝑐
𝑧 𝑐 = 𝑒 ln 𝑧 = 𝑒 𝑐 ln 𝑧 , (𝑐 is a complex number, 𝑧 ≠ 0)

In general 𝑧 𝑐 is a multivalued function.

Example 17 find the principle value of


𝒂. 𝒊𝒊 𝒃. (𝟏 + 𝒊)𝟐−𝒊
Solution:
(a)
𝜋 𝜋
𝑖(𝑙𝑜𝑔𝑒 1+𝑖( 2 ))
𝑖 𝑖 = 𝑒 𝑖 𝐿𝑛 𝑖 = 𝑒 = 𝑒 −2
(b)
𝜋 𝜋 𝜋
(2−𝑖)(𝑙𝑜𝑔𝑒 √2+𝑖( ))
(1 + 𝑖)2−𝑖 = 𝑒 (2−𝑖) 𝐿𝑛(1+𝑖) = 𝑒 4 = 𝑒 2 𝑙𝑜𝑔2 √2+ 4 +𝑖(− 𝑙𝑜𝑔𝑒 √2+ 2 )
𝜋 𝜋 𝜋 𝜋 𝜋
= 𝑒 2 𝑙𝑜𝑔2 √2 + 4 𝑒 𝑖(− 𝑙𝑜𝑔𝑒 √2+ 2 ) = 2𝑒 4 [𝑐𝑜𝑠 (− 𝑙𝑜𝑔𝑒 √2 + ) + 𝑖 𝑠𝑖𝑛 (− 𝑙𝑜𝑔𝑒 √2 + )]
2 2
𝜋
= 2𝑒 4 [sin(− log e √2) + 𝑖 cos(− log e √2)]

- 107 -
Chapter 4 Complex Numbers
The Trigonometric Functions:

The trigonometric function for a complex variable 𝑧 is defined as follows:

𝑒 𝑖𝑧 − 𝑒 −𝑖𝑧 𝑒 𝑖𝑧 + 𝑒 −𝑖𝑧
sin 𝑧 = , cos 𝑧 =
2𝑖 2

From the above definition of the trigonometric functions we can easily prove the following:

(1) sin(𝑧1 ± 𝑧2 ) = sin 𝑧1 cos 𝑧2 ± cos 𝑧1 sin 𝑧2 .

(2) cos(𝑧1 ± 𝑧2 ) = cos 𝑧1 cos 𝑧2 ∓ sin 𝑧1 sin 𝑧2 .

(3) cos2 𝑧 + sin2 𝑧 = 1.

(4) sin 𝑥 = 0 ⇒ when 𝑥 = 𝑛𝜋, 𝑛 = 0, ±1, ±2, …


𝜋
(5) cos 𝑥 = 0 ⇒ when 𝑥 = (2𝑛 + 1) , 𝑛 = 0, ±1, ±2, …
2

𝜋
(6) sin 𝑥 = (−1)𝑛 ⇒ when 𝑥 = (2𝑛 + 1) , 𝑛 = 0, ±1, ±2, … ,
2
𝜋
sin 𝑥 = 1, when 𝑥 = (2𝑛 + 1) , 𝑛 = 0, ±2, ±4, … = even.
2
𝜋
sin 𝑥 = −1, when 𝑥 = (2𝑛 + 1) , 𝑛 = ±1, ±3, … = odd.
2

(7) cos 𝑥 = (−1)𝑛 ⇒ when 𝑥 = 𝑛𝜋, 𝑛 = 0, ±1, ±2, … ,


cos 𝑥 = 1, when 𝑥 = 𝑛𝜋, 𝑛 = 0, ±2, ±4, … = even.
cos 𝑥 = −1, when 𝑥 = 𝑛𝜋, 𝑛 = ±1, ±3, … = odd.
The Hyperbolic Functions:

The hyperbolic function for a complex variable 𝑧 is defined as follows:

𝑒 𝑧 − 𝑒 −𝑧 𝑒 𝑧 + 𝑒 −𝑧
sin 𝑧 = , cos 𝑧 =
2 2

From the above definition of the trigonometric functions we can easily prove the following:

(1) sinh(𝑧1 ± 𝑧2 ) = sinh 𝑧1 cosh 𝑧2 ± cosh 𝑧1 sinh 𝑧2 .

(2) cosh(𝑧1 ± 𝑧2 ) = cosh 𝑧1 cosh 𝑧2 ± sinh 𝑧1 sinh 𝑧2 .

(3) cosh2 𝑧 − sinh2 𝑧 = 1.

(4) sin 𝑖𝑧 = 𝑖 sinh 𝑧

(5) sinh 𝑖𝑧 = 𝑖 sin 𝑧

(6) cos 𝑖𝑧 = cosh 𝑧

(7) cosh 𝑖𝑧 = cos 𝑧

- 108 -
Chapter 4 Complex Numbers
Example 18 Solve the following equations
𝒂. 𝐜𝐨𝐬 𝒛 = 𝟓 𝒃. 𝐬𝐢𝐧𝐡 𝒛 = 𝟎
Solution:
(a) cos 𝑧 = 5
Method 1
Assume 𝑧 = 𝑥 + 𝑖𝑦
cos 𝑧 = 5 ⇒ cos(𝑥 + 𝑖𝑦) = 5

cos 𝑥 cos 𝑖𝑦 − sin 𝑥 sin 𝑖𝑦 = 5 ⇒ cos 𝑥 cosh 𝑦 − 𝑖 sin 𝑥 sinh 𝑦 = 5

sin 𝑥 sinh 𝑦 = 0 cos 𝑥 cosh 𝑦 = 5

sin 𝑥 = 0 ⇒ 𝑥 = 𝑛𝜋, 𝑛 = 0, ±1, ±2, … ⇒ 𝑥 = 𝑛𝜋, 𝑛 = 0, ±1, ±2, …


cos 𝑛𝜋 cosh 𝑦 = 5 ⇒ (−1)𝑛 cosh 𝑦 = 5
∵ cosh 𝑦 is positive function ⇒ (−1)𝑛 = 1,
if 𝑛 = even
cosh 𝑦 = 5, 𝑛 = 0, ±2, ±4, … = even

or 𝑦 = ± cosh−1 5 , 𝑛 = 0, ±2, ±4, … = even

sinh 𝑦 = 0 ⇒ 𝑦=0 ⇒ 𝑦 = 0 ⇒ cos 𝑥 cosh 0 = 5 ⇒ cos 𝑥 = 5


This case is rejected because −1 ≤ cos 𝑥 ≤ 1

𝑧 = (𝑛𝜋) + 𝑖(± cosh−1 5), 𝑛 = 0, ±2, ±4, … = even

Method 2
𝑒 𝑖𝑧 + 𝑒 −𝑖𝑧
cos 𝑧 = 5 ⇒ = 5 ⇒ 𝑒 𝑖𝑧 + 𝑒 −𝑖𝑧 = 10 ⇒ 𝑒 𝑖𝑧 − 10 + 𝑒 −𝑖𝑧 = 0
2
Then,
2 10 ± √100 − 4 10 ± √96
(𝑒 𝑖𝑧 ) − 10(𝑒 𝑖𝑧 ) + 1 = 0 ⇒ 𝑒 𝑖𝑧 = = = 5 ± 2√6
2 2

𝑒 𝑖𝑧 = 5 + 2√6 𝑒 𝑖𝑧 = 5 − 2√6

𝑖𝑧 = ln(5 + 2√6) 𝑖𝑧 = ln(5 − 2√6)

𝑖𝑧 = ln(5 + 2√6) + 𝑖(2𝑘𝜋), 𝑘 = 0, ±1, ±2, … 𝑖𝑧 = ln(5 − 2√6) + 𝑖(2𝑘𝜋), 𝑘 = 0, ±1, ±2, …

𝑧 = 2𝑘𝜋 − 𝑖 ln(5 + 2√6), 𝑘 = 0, ±1, ±2, … 𝑧 = 2𝑘𝜋 − 𝑖 ln(5 − 2√6) , 𝑘 = 0, ±1, ±2, …

- 109 -
Chapter 4 Complex Numbers
(b) sinh 𝑧 = 0
Method 1
𝑒 𝑧 − 𝑒 −𝑧
sinh 𝑧 = 0 ⇒ = 0 ⇒ 𝑒 𝑧 − 𝑒 −𝑧 = 0 ⇒ (𝑒 𝑧 )2 − 1 = 0
2
Then,
𝑒 2𝑧 = 1 ⇒ 2𝑧 = ln(1) = log e 1 + 𝑖(0 + 2𝑘𝜋), 𝑘 = 0, ±1, ±2, …

𝑧 = 𝑖(𝑘𝜋), 𝑘 = 0, ±1, ±2, …

Method 2
Assume 𝑧 = 𝑥 + 𝑖𝑦
sinh 𝑧 = 0 ⇒ sinh(𝑥 + 𝑖𝑦) = 0

cosh 𝑥 sinh 𝑖𝑦 + sinh 𝑥 cosh 𝑖𝑦 = 0 ⇒ 𝑖 cosh 𝑥 sin 𝑦 + sinh 𝑥 cos 𝑦 = 0

cosh 𝑥 sin 𝑦 = 0 sinh 𝑥 cos 𝑦 = 0

sin 𝑦 = 0 ⇒ 𝑦 = 𝑛𝜋, 𝑛 = 0, ±1, ±2, … ⇒ 𝑦 = 𝑛𝜋, 𝑛 = 0, ±1, ±2, …


or sinh 𝑥 cos 𝑛𝜋 = 0 ⇒ (−1)𝑛 sinh 𝑥 = 0
cosh 𝑥 = 0 sinh 𝑥 = 0 ⇒ 𝑥=0
This case is rejected because cosh 𝑥 ≥ 1

𝑧 = 0 + 𝑖(𝑛𝜋), 𝑛 = 0, ±1, ±2, …

Analytic functions:

The Cauchy-Riemann equations

Before introducing the definition of analytic functions let us start with the following:
A function 𝑓(𝑧) = 𝑢 + 𝑖𝑣 is said to satisfy the Cauchy-Riemann equations at the point (𝑥0 , 𝑦0 ) if
𝜕𝑢(𝑥0 , 𝑦0 ) 𝜕𝑣(𝑥0 , 𝑦0 ) 𝜕𝑢(𝑥0 , 𝑦0 ) 𝜕𝑣(𝑥0 , 𝑦0 )
= , =−
𝜕𝑥 𝜕𝑦 𝜕𝑦 𝜕𝑥
Theorem 1
If 𝑓(𝑧) is a differentiable at 𝑧 = 𝑧0 , then it must satisfy the Cauchy-Reimann equation at this point.
Moreover
𝑓 ′ (𝑧0 ) = 𝑢𝑥 (𝑥0 , 𝑦0 ) + 𝑖𝑣𝑥 (𝑥0 , 𝑦0 ) = 𝑣𝑦 (𝑥0 , 𝑦0 ) − 𝑖𝑢𝑦 (𝑥0 , 𝑦0 ).

Proof:
Suppose the 𝑓 is differentiable at 𝑧 = 𝑧0 . Then,

- 110 -
Chapter 4 Complex Numbers
𝑓(𝑧) − 𝑓(𝑧0 ) 𝑢(𝑥, 𝑦) + 𝑖𝑣(𝑥, 𝑦) − 𝑢(𝑥0 , 𝑦0 ) − 𝑖𝑣(𝑥0 , 𝑦0 )
𝑓 ′ (𝑧0 ) = lim = 𝑥→𝑥
lim
𝑧→𝑧0 𝑧 − 𝑧0 𝑦→𝑦
0 𝑥 + 𝑖𝑦 − 𝑥0 − 𝑖𝑦0
0

Since, this limit exist, then it has the same value along any path from 𝑧 to 𝑧0 . On the path 𝑦 = 𝑦0 have

𝑢(𝑥, 𝑦0 ) − 𝑢(𝑥0 , 𝑦0 ) 𝑣(𝑥, 𝑦0 ) − 𝑣(𝑥0 , 𝑦0 )


𝑓 ′ (𝑧0 ) = lim [ +𝑖 ]
𝑦→𝑦0 𝑥 − 𝑥0 𝑥 − 𝑥0
𝜕𝑢(𝑥0 , 𝑦0 ) 𝜕𝑣(𝑥0 , 𝑦0 )
= +𝑖 − − − − − −(1)
𝜕𝑥 𝜕𝑥
Similarity, on the path 𝑥 = 𝑥0 we get

𝑢(𝑥0 , 𝑦) − 𝑢(𝑥0 , 𝑦0 ) 𝑣(𝑥0 , 𝑦) − 𝑣(𝑥0 , 𝑦0 )


𝑓 ′ (𝑧0 ) = lim [ + ]
𝑥→𝑥0 𝑖(𝑦 − 𝑦0 ) 𝑦 − 𝑦0
𝜕𝑣(𝑥0 , 𝑦0 ) 𝜕𝑢(𝑥0 , 𝑦0 )
= −𝑖 − − − − − −(2)
𝜕𝑦 𝜕𝑦
From (1) and (2), we conclude that
𝑓 ′ (𝑧0 ) = 𝑢𝑥 (𝑥0 , 𝑦0 ) + 𝑖𝑣𝑥 (𝑥0 , 𝑦0 ) = 𝑣𝑦 (𝑥0 , 𝑦0 ) − 𝑖𝑢𝑦 (𝑥0 , 𝑦0 )
and that
𝜕𝑢(𝑥0 , 𝑦0 ) 𝜕𝑣(𝑥0 , 𝑦0 ) 𝜕𝑢(𝑥0 , 𝑦0 ) 𝜕𝑣(𝑥0 , 𝑦0 )
= , =−
𝜕𝑥 𝜕𝑦 𝜕𝑦 𝜕𝑥
Theorem 2 (Sufficient condition for the existence of the derivative)

If all partial derivatives 𝑢𝑥 , 𝑢𝑦 , 𝑣𝑥 and 𝑣𝑦 are continuous at 𝑧 = 𝑧0 and the Cauchy-Reimann equation are
satisfied at 𝑧 = 𝑧0 , then the function 𝑓(𝑧) is differentiable at this point.
Analytic function:
Definition 6
A complex function 𝒇(𝒛) is said to be analytic, or regular at the point 𝒛𝟎 if it is defined and
differentiable at 𝒛𝟎 and at every point 𝒛 in some neighborhood of 𝒛𝟎 .

Note that:
1. It is clear that, if the Cauchy-Riemann equations are satisfied everywhere. Since the partial derivatives
are everywhere continuous, the derivative of the function exists everywhere in the 𝑧-plane. Hence, the
function is everywhere analytic.

2. If 𝑓 is analytic at 𝑧0 , then 𝑓(𝑧) is also differentiable at 𝑧0 , but the converse need not be true. That is,
there do exist complex functions 𝑓(𝑧) such that 𝑓(𝑧) is differentiable at 𝑧0 but not analytic at 𝑧0 .

3. A function 𝑓 is analytic in a domain 𝐷 if 𝑓(𝑧) is analytic at every point in 𝐷. If 𝑓(𝑧) is analytic at every
point in the complex plane, then 𝑓(𝑧) is called an entire function.

- 111 -
Chapter 4 Complex Numbers
Example 19 Show that the function 𝒇(𝒛) = 𝒛 + 𝟑 is analytic function.
Solution:
𝑓(𝑧) = 𝑧 + 3 = (𝑥 + 3) + 𝑖𝑦
Hence,
𝑢 = 𝑥 + 3, 𝑣=𝑦 ⇒ 𝑢𝑥 = 1, 𝑢𝑦 = 0, 𝑣𝑥 = 0, 𝑣𝑦 = 1
It is clear that 𝑢𝑥 , 𝑢𝑦 , 𝑣𝑥 and 𝑣𝑦 are all continuous in the whole 𝑧-plane and 𝑢𝑥 = 𝑣𝑦 and 𝑢𝑦 = −𝑣𝑥 .
Therefore, 𝑓(𝑧) = 𝑧 + 3 is differentiable for all 𝑧, i.e. 𝑓(𝑧) is an analytic function. In this example, 𝑓(𝑧) is
an entire function.

Example 20 Show that the function 𝒇(𝒛) = |𝒛|𝟐 is differentiable but not analytic function at 𝒛 = 𝟎.
Solution:
𝑓(𝑧) = |𝑧|2 = 𝑥 2 + 𝑦 2
Hence,
𝑢 = 𝑥2 + 𝑦2, 𝑣=0 ⇒ 𝑢𝑥 = 2𝑥, 𝑢𝑦 = 2𝑦, 𝑣𝑥 = 0, 𝑣𝑦 = 0
These derivatives are continuous and satisfy the Cauchy-Riemann equations at 𝑧 = 0. Consequently, 𝑓(𝑧)
is differentiable at 𝑧 = 0.
If 𝑧 ≠ 0, the Cauchy-Riemann equations are not satisfied. This means that there is no neighborhood about
𝑧 = 0 throughout which the function is differentiable. Thus, 𝑓(𝑧) = |𝑧|2 is not analytic at 𝑧 = 0.

Example 21 Determined whether the following functions are analytic


(a) 𝒇(𝒛) = 𝒆−𝒚 (𝐜𝐨𝐬 𝒙 + 𝒊 𝐬𝐢𝐧 𝒙) (b) 𝒇(𝒛) = 𝒛 − 𝒛̅ (c) 𝒇(𝒛) = 𝐜𝐨𝐬𝐡 (𝒊𝒛).
Solution:
(a) 𝑓(𝑧) = 𝑒 −𝑦 (cos 𝑥 + 𝑖 sin 𝑥) = 𝑒 −𝑦 cos 𝑥 + 𝑖 𝑒 −𝑦 sin 𝑥
Hence,
𝑢 = 𝑒 −𝑦 cos 𝑥 , 𝑣 = 𝑒 −𝑦 sin 𝑥
𝑢𝑥 = −𝑒 −𝑦 sin 𝑥 , 𝑢𝑦 = −𝑒 −𝑦 cos 𝑥 , 𝑣𝑥 = 𝑒 −𝑦 cos 𝑥 , 𝑣𝑦 = −𝑒 −𝑦 sin 𝑥
It is clear that 𝑢𝑥 , 𝑢𝑦 , 𝑣𝑥 and 𝑣𝑦 are all continuous in the whole 𝑧-plane and 𝑢𝑥 = 𝑣𝑦 and 𝑢𝑦 = −𝑣𝑥 .
Thus, 𝑓(𝑧) is an analytic function.

(b) 𝑓(𝑧) = 𝑧 − 𝑧̅ = 2𝑥 = 2𝑥 + 𝑖 (0)


Hence,
𝑢 = 2𝑥, 𝑣 = 0 ⇒ 𝑢𝑥 = 2, 𝑢𝑦 = 0, 𝑣𝑥 = 0, 𝑣𝑦 = 0
It is clear that 𝑢𝑥 , 𝑢𝑦 , 𝑣𝑥 and 𝑣𝑦 are all continuous in the whole 𝑧-plane but 𝑢𝑥 ≠ 𝑣𝑦 . Thus, 𝑓(𝑧) is
not an analytic function.
(c) 𝑓(𝑧) = cosh(𝑖𝑧) = cosh(𝑖𝑥 − 𝑦) = cosh 𝑖𝑥 cosh(−𝑦) + sinh 𝑖𝑥 sinh(−𝑦)
𝑓(𝑧) = cos 𝑥 cosh 𝑦 − 𝑖 sin 𝑥 sinh 𝑦
Hence,
𝑢 = cos 𝑥 cosh 𝑦 , 𝑣 = − sin 𝑥 sinh 𝑦
𝑢𝑥 = − sin 𝑥 cosh 𝑦 , 𝑢𝑦 = cos 𝑥 sinh 𝑦, 𝑣𝑥 = − cos 𝑥 sinh 𝑦 , 𝑣𝑦 = − sin 𝑥 cosh 𝑦
It is clear that 𝑢𝑥 , 𝑢𝑦 , 𝑣𝑥 and 𝑣𝑦 are all continuous in the whole 𝑧-plane and 𝑢𝑥 = 𝑣𝑦 and 𝑢𝑦 = −𝑣𝑥 .
Thus, 𝑓(𝑧) is an analytic function.
- 112 -
Chapter 4 Complex Numbers
Harmonic function:
Definition 6
A real – valued function 𝒈(𝒙, 𝒚) of real variable 𝒙 and 𝒚 is said to be harmonic function on set 𝑺 if it
satisfies Laplace's equation
𝝏𝒈 𝝏𝒈
𝟐 + 𝝏𝒙 𝟐 = 𝟎,
𝝏𝒚
𝐟𝐨𝐫 𝐚𝐥𝐥 (𝒙, 𝒚) ∈ 𝑺

Theorem 1
If the real and imaginary parts of an analytic function 𝑓(𝑧) have continuous second order partial derivatives
with respect to 𝑥 and 𝑦 in the domain 𝐷 of 𝑓(𝑧), and 𝑢(𝑥, 𝑦) and 𝑣(𝑥, 𝑦) are harmonic in 𝐷. They are called
the conjugate harmonic functions. i.e.
𝜕𝑢 𝜕𝑢
+ = 0, for all (𝑥, 𝑦) ∈ 𝑆
𝜕𝑥 2 𝜕𝑦 2
and
𝜕𝑣 𝜕𝑣
2
+ 2 = 0, for all (𝑥, 𝑦) ∈ 𝑆
𝜕𝑥 𝜕𝑦
Proof:
Since 𝑓(𝑧) is analytic in 𝐷, the Cauchy-Riemann equations are satisfied in 𝐷, i.e.

𝜕𝑢(𝑥0 , 𝑦0 ) 𝜕𝑣(𝑥0 , 𝑦0 ) 𝜕𝑢(𝑥0 , 𝑦0 ) 𝜕𝑣(𝑥0 , 𝑦0 )


= and =− in 𝐷
𝜕𝑥 𝜕𝑦 𝜕𝑦 𝜕𝑥
Since the second partial derivatives are continuous, we can differentiate with respect to 𝑥 the above equation
to get
𝜕 2 𝑢(𝑥0 , 𝑦0 ) 𝜕 2 𝑣(𝑥0 , 𝑦0 ) 𝜕 2 𝑢(𝑥0 , 𝑦0 ) 𝜕 2 𝑣(𝑥0 , 𝑦0 )
= and = −
𝜕𝑥 2 𝜕𝑥𝜕𝑦 𝜕𝑦 2 𝜕𝑦𝜕𝑥
By continuity of this derivatives, we have that
𝜕 2 𝑢(𝑥0 , 𝑦0 ) 𝜕 2 𝑢(𝑥0 , 𝑦0 ) 𝜕 2 𝑣(𝑥0 , 𝑦0 ) 𝜕 2 𝑣(𝑥0 , 𝑦0 )
= and = in 𝐷
𝜕𝑥𝜕𝑦 𝜕𝑦𝜕𝑥 𝜕𝑥𝜕𝑦 𝜕𝑦𝜕𝑥
Therefore, by adding we get
𝜕 2 𝑢(𝑥0 , 𝑦0 ) 𝜕 2 𝑢(𝑥0 , 𝑦0 )
+ = 0, in 𝐷
𝜕𝑥 2 𝜕𝑦 2
This shows that 𝑢 is harmonic in 𝐷. Also, by differentiating the both sides of the Cauchy-Riemann equations
with respect to 𝑦 and 𝑥, we get
𝜕 2 𝑢(𝑥0 , 𝑦0 ) 𝜕 2 𝑣(𝑥0 , 𝑦0 ) 𝜕 2 𝑢(𝑥0 , 𝑦0 ) 𝜕 2 𝑣(𝑥0 , 𝑦0 )
= and =−
𝜕𝑦𝜕𝑥 𝜕𝑦 2 𝜕𝑥𝜕𝑦 𝜕𝑥 2
Therefore, by adding we get
𝜕 2 𝑣(𝑥0 , 𝑦0 ) 𝜕 2 𝑣(𝑥0 , 𝑦0 )
+ = 0, in 𝐷
𝜕𝑥 2 𝜕𝑦 2
Hence, 𝑣 is harmonic in 𝐷.

- 113 -
Chapter 4 Complex Numbers
Example 22 Determined whether the following functions are harmonic, if the function is harmonic
find its harmonic conjugate 𝒗(𝒙, 𝒚) then express 𝒇(𝒛) as a function of the complex
variable 𝒛
(𝐚)𝒖(𝒙, 𝒚) = 𝒙𝟐 − 𝒚𝟐 (𝐛) 𝒖(𝒙, 𝒚) = 𝐜𝐨𝐬 𝒙 𝐜𝐨𝐬𝐡 𝒚 (𝐜) 𝒖(𝒙, 𝒚) = −𝒆−𝒙 𝐬𝐢𝐧 𝒚
Solution:
(a) 𝑢(𝑥, 𝑦) = 𝑥 2 − 𝑦 2

𝑢𝑥 = 2𝑥, 𝑢𝑦 = −2𝑦, 𝑢𝑥𝑥 = 2, 𝑢𝑦𝑦 = −2


Hence,
𝜕2𝑢 𝜕2𝑢
+ = 2 − 2 = 0, for all (𝑥, 𝑦)
𝜕𝑥 2 𝜕𝑦 2
Thus, 𝑓(𝑧) is a harmonic function. Since the harmonic conjugate 𝑣(𝑥, 𝑦) is the imaginary part of an
analytic function whose real part is 𝑢(𝑥, 𝑦), it follows from the Cauchy-Riemann equations that
𝜕𝑢 𝜕𝑣 𝜕𝑢 𝜕𝑣
= and =−
𝜕𝑥 𝜕𝑦 𝜕𝑦 𝜕𝑥
Using the first equation, we get
𝜕𝑣
= 2𝑥 ⇒ ∫ ⇒ 𝑣(𝑥, 𝑦) = 2𝑥𝑦 + 𝑐(𝑥)
𝜕𝑦
This implies that
𝜕𝑣
= 2𝑦 + 𝑐′(𝑥),
𝜕𝑥
then, using the second equation of the Cauchy – Riemann equations, we get

2𝑦 + 𝑐 ′ (𝑥) = 2𝑦 ⇒ 𝑐 ′ (𝑥) = 0 ⇒ ∫ ⇒ 𝑐(𝑥) = 𝑘


where 𝑘 is any constant. Therefore
𝑣(𝑥, 𝑦) = 2𝑥𝑦 + 𝑘
and
𝑓(𝑧) = 𝑢(𝑥, 𝑦) + 𝑖𝑣(𝑥, 𝑦) = 𝑥 2 − 𝑦 2 + 𝑖[2𝑥𝑦 + 𝑘]
To express 𝑓(𝑧) in terms of 𝑧, we can replace each 𝑥 by 𝑧 and each 𝑦 by 0 in 𝑓(𝑧). Then,
𝑓(𝑧) = 𝑧 2 + 𝑖𝑘

(b) 𝑢(𝑥, 𝑦) = cos 𝑥 cosh 𝑦

𝑢𝑥 = − sin 𝑥 cosh 𝑦 , 𝑢𝑦 = cos 𝑥 sinh 𝑦, 𝑢𝑥𝑥 = −cos 𝑥 cosh 𝑦 , 𝑢𝑦𝑦 = cos 𝑥 cosh 𝑦
Hence,
𝜕2𝑢 𝜕2𝑢
+ = − cos 𝑥 cosh 𝑦 + cos 𝑥 cosh 𝑦 = 0, for all (𝑥, 𝑦)
𝜕𝑥 2 𝜕𝑦 2
Thus, 𝑓(𝑧) is a harmonic function. Since the harmonic conjugate 𝑣(𝑥, 𝑦) is the imaginary part of an
analytic function whose real part is 𝑢(𝑥, 𝑦), it follows from the Cauchy-Riemann equations that
𝜕𝑢 𝜕𝑣 𝜕𝑢 𝜕𝑣
= and =−
𝜕𝑥 𝜕𝑦 𝜕𝑦 𝜕𝑥
Using the first equation, we get

- 114 -
Chapter 4 Complex Numbers
𝜕𝑣
= − sin 𝑥 cosh 𝑦 ⇒ ∫ ⇒ 𝑣(𝑥, 𝑦) = − sin 𝑥 sinh 𝑦 + 𝑐(𝑥)
𝜕𝑦
This implies that
𝜕𝑣
= − cos 𝑥 sinh 𝑦 + 𝑐′(𝑥),
𝜕𝑥
then, using the second equation of the Cauchy – Riemann equations, we get

− cos 𝑥 sinh 𝑦 + 𝑐 ′ (𝑥) = − cos 𝑥 sinh 𝑦 ⇒ 𝑐 ′ (𝑥) = 0 ⇒ ∫ ⇒ 𝑐(𝑥) = 𝑘


where 𝑘 is any constant. Therefore
𝑣(𝑥, 𝑦) = − cos 𝑥 sinh 𝑦 + 𝑘
and
𝑓(𝑧) = 𝑢(𝑥, 𝑦) + 𝑖𝑣(𝑥, 𝑦) = cos 𝑥 cosh 𝑦 + 𝑖[− sin 𝑥 sinh 𝑦 + 𝑘]
To express 𝑓(𝑧) in terms of 𝑧, we can replace each 𝑥 by 𝑧 and each 𝑦 by 0 in 𝑓(𝑧). Then,

𝑓(𝑧) = cos 𝑧 + 𝑖𝑘

(c) 𝑢(𝑥, 𝑦) = −𝑒 −𝑥 sin 𝑦

𝑢𝑥 = 𝑒 −𝑥 sin 𝑦 , 𝑢𝑦 = −𝑒 −𝑥 cos 𝑦, 𝑢𝑥𝑥 = −𝑒 −𝑥 sin 𝑦 , 𝑢𝑦𝑦 = 𝑒 −𝑥 sin 𝑦


Hence,
𝜕2𝑢 𝜕2𝑢
+ = −𝑒 −𝑥 sin 𝑦 + 𝑒 −𝑥 sin 𝑦 = 0, for all (𝑥, 𝑦)
𝜕𝑥 2 𝜕𝑦 2
Thus, 𝑓(𝑧) is a harmonic function. Since the harmonic conjugate 𝑣(𝑥, 𝑦) is the imaginary part of an
analytic function whose real part is 𝑢(𝑥, 𝑦), it follows from the Cauchy-Riemann equations that
𝜕𝑢 𝜕𝑣 𝜕𝑢 𝜕𝑣
= and =−
𝜕𝑥 𝜕𝑦 𝜕𝑦 𝜕𝑥
Using the first equation, we get
𝜕𝑣
= 𝑒−𝑥 sin 𝑦 ⇒ ∫ ⇒ 𝑣(𝑥, 𝑦) = −𝑒−𝑥 cos 𝑦 + 𝑐(𝑥)
𝜕𝑦
This implies that
𝜕𝑣
= 𝑒−𝑥 cos 𝑦 + 𝑐′(𝑥),
𝜕𝑥
then, using the second equation of the Cauchy – Riemann equations, we get

𝑒 −𝑥 cos 𝑦 + 𝑐′(𝑥) = 𝑒 −𝑥 cos 𝑦 ⇒ 𝑐 ′ (𝑥) = 0 ⇒ ∫ ⇒ 𝑐(𝑥) = 𝑘


where 𝑘 is any constant. Therefore
𝑣(𝑥, 𝑦) = −𝑒 −𝑥 cos 𝑦 + 𝑘
and
𝑓(𝑧) = 𝑢(𝑥, 𝑦) + 𝑖𝑣(𝑥, 𝑦) = −𝑒 −𝑥 sin 𝑦 + 𝑖[−𝑒 −𝑥 cos 𝑦 + 𝑘]
To express 𝑓(𝑧) in terms of 𝑧, we can replace each 𝑥 by 𝑧 and each 𝑦 by 0 in 𝑓(𝑧). Then,
𝑓(𝑧) = [−𝑖𝑒 −𝑧 + 𝑘]

- 115 -
Chapter 4 Complex Numbers
Sheet 10

1. Express 𝑒 𝑧 in the form 𝑎 + 𝑖𝑏


𝜋
(i) 𝑧 = 1.7 + 4.2𝑖 (ii) 𝑧 = −1 + 𝑖 (iii) 𝑧 = − 0.23 − 𝑖
4
3𝜋
(iv) 𝑧 = 𝜋 + 𝜋𝑖 (v) 𝑧 = 5𝑖 (vi) 𝑧 = −𝜋 + 𝑖
2

2. Express the given function in the form 𝑓(𝑧) = 𝑢 + 𝑖𝑣


2
(i) 𝑓(𝑧) = 𝑒 −𝑖𝑧 (ii) 𝑓(𝑧) = 𝑒 2𝑧̅ (iii) 𝑓(𝑧) = 𝑒 𝑧 (iv) 𝑓(𝑧) = 𝑒1/𝑧

3. Express ln 𝑧 in the form 𝑎 + 𝑖𝑏


(i) 𝑧 = −5 (ii) 𝑧 = −2 + 2𝑖 (iii) 𝑧 = √2 + √6𝑖
(iv) 𝑧 = −𝑒𝑖 (v) 𝑧 = 1 + 𝑖 (vi) 𝑧 = −√3 + 𝑖

4. Express the principle value of ln 𝑧 in the form 𝑎 + 𝑖𝑏


5
(i) 𝑧 = −6 − 6𝑖 (ii) 𝑧 = −12 + 5𝑖 (iii) 𝑧 = (1 + √3𝑖)
(iv) 𝑧 = −𝑒 3 (v) 𝑧 = 3 − 4𝑖 (vi) 𝑧 = (1 + 𝑖)4

5. Find all values of 𝑧 satisfying the given equation


(i) 𝑒 𝑧 = 4𝑖 (ii) 𝑒1/𝑧 = −1 (iii) 𝑒 𝑧−1 = −𝑖𝑒 2 (iv) 𝑒 2𝑧 + 𝑒 𝑧 + 1 = 0

6. Find all values of the given quantity


3𝑖
(i) (−𝑖)4𝑖 (ii) 3𝑖/𝜋 (iii) (1 + 𝑖)1+𝑖 (iv) (1 + √3𝑖)

7. Find the principal value of the given quantity. Express answers in the form 𝑎 + 𝑖𝑏
(i) (−1)−2𝑖/𝜋 (ii) (1 − 𝑖)2𝑖

8. Find all values of 𝑧 satisfying the given equation


(i) sin 𝑧 = 2 (ii) cos 𝑧 = −3𝑖 (iii) sinh 𝑧 = −1
(iv) cosh 𝑧 = −𝑖 (v) cos 𝑧 = sin 𝑧 (vi) cos 𝑧 = 𝑖 sin 𝑧

9. Determine whether the following functions are analytic


(i) 𝑓(𝑧) = 𝑖𝑧𝑧̅ (ii) 𝑓(𝑧) = 𝑒 −2𝑥 (cos 2𝑦 − 𝑖 sin 2𝑦) (iii) 𝑓(𝑧) = 𝑒 𝑥 (cos 𝑦 − 𝑖 sin 𝑦)
(iv) 𝑓(𝑧) = ln|𝑧| + 𝑖 Arg 𝑧 (v) 𝑓(𝑧) = cos 𝑧 cosh 𝑦 − 𝑖 sin 𝑥 sinh 𝑦 (vi) 𝑓(𝑧) = Re(𝑧 2 ) − 𝑖 Im(𝑧 2 )
10. Find real constants 𝑎, 𝑏 , 𝑐 , and 𝑑 so that the given function is analytic
(i) 𝑓(𝑧) = 3𝑥 − 𝑦 + 5 + 𝑖(𝑎𝑥 + 𝑏𝑦 − 3) (ii) 𝑓(𝑧) = 𝑥 2 + 𝑎𝑥𝑦 + 𝑏𝑦 2 + 𝑖(𝑐𝑥 2 + 𝑑𝑥𝑦 + 𝑦 2 )

11. Determine whether the following functions are harmonic? If the answer is yes, find a corresponding
analytic function 𝑓(𝑧) = 𝑢(𝑥, 𝑦) + 𝑖𝑣(𝑥, 𝑦)
𝑥
(i) 𝑢 = 𝑥 2 + 𝑦 2 (ii) 𝑢 = 𝑥𝑦 (iii) 𝑢 = sin 𝑥 cosh 𝑦 (iv) 𝑢 =
𝑥 2 +𝑦 2
(v) 𝑣 = 𝑒 𝑥 sin 2𝑦 (vi) 𝑣 = (2𝑥 + 1)𝑦 (vii) 𝑢 = 𝑥 3 − 3𝑥𝑦 2 (viii) 𝑣 = 𝑥𝑦

12. Determine 𝑎 and 𝑏 so that the given function is harmonic and find a harmonic conjugate
(i) 𝑢 = 𝑒 𝜋𝑥 cos 𝑎𝑦 (ii) 𝑢 = cos 𝑎𝑥 cosh 2𝑦 (iii) 𝑢 = 𝑎𝑥 3 + 𝑏𝑥𝑦 (iv) 𝑢 = cosh 𝑎𝑥 cos 𝑦

- 116 -
Chapter 4 Complex Numbers
Homework 10:
1. Find all 𝑛𝑡ℎ roots of the following complex numbers 𝑧: Express your answers in the form 𝑎 + 𝑖𝑏
(i) 𝑛 = 3, 𝑧 = −8𝑖.
(ii) 𝑛 = 4, 𝑧 = −2 + √12𝑖.
2𝑖
(iii)𝑛 = 3, 𝑧 = .
1+𝑖
3𝜋
2. Solve for 𝑧 the equation ln(𝑧 + 2𝑖) = 1 + 𝑖.
4

3. Show that when 𝑛 = 0, ±1, ±2, …


𝜋 𝑖
(i) (1 + 𝑖)𝑖 = 𝑒 (− 4 +2𝑛𝜋) 𝑒 (2 ln 2) .
(ii) (−1)1/𝜋 = 𝑒 (2𝑛+1)𝑖 .
4. Find the principal value of (1 − 𝑖)4𝑖
5. Find all the complex roots of the equation cos 𝑧 = 3.
6. Show that
1
1 1
(i) The set of values ln (𝑖 2 ) is (𝑛 + ) 𝜋𝑖 (𝑛 = 0, ±1, ±2, ±3, … ) and the same is true of ln 𝑖.
4 2

(ii) The set of values ln(𝑖 2 ) is not the same as the set of values of 2 ln 𝑖.
7. Find all solutions 𝑧 of the following (express your answers in the form 𝑥 + 𝑖𝑦):
(i) ln 𝑧 = 4𝑖.
(ii) 𝑧 𝑖 = 𝑖.
8. Determine which of the following functions are analytic
(i) 𝑧 = 2𝑥𝑦 + 𝑖(𝑥 2 + 𝑦 2 ).
(ii) 𝑧 = 𝑥 3 + 𝑖(1 − 𝑦)3 .
(iii)𝑧 = 𝑒 𝑦 𝑒 𝑖𝑥 .
9. Verify that the following functions 𝑢 are harmonic, and in each case give a conjugate harmonic function
𝑣 (i.e. 𝑣 such that 𝑢 + 𝑖𝑣 is analytic)
(i) 𝑢(𝑥, 𝑦) = 3𝑥 2 𝑦 + 2𝑥 2 − 𝑦 3 − 2𝑦 2 .
(ii) 𝑢(𝑥, 𝑦) = ln(𝑥 2 + 𝑦 2 ).

Challenging problems:
(i) Prove that sin(ln(𝑖)−𝑖 ) = 1.
(ii) Show that |cos 𝑧|2 = cos2 𝑥 + sinh2 𝑦.
1 1 𝜃 𝜋 𝜃
(iii)Prove that Ln ( ) = log e ( csc ) + 𝑖 ( − )
1−𝑒 𝑖𝜃 2 2 2 2

- 117 -

You might also like